Thread Rating:
  • 0 Vote(s) - 0 Average
  • 1
  • 2
  • 3
  • 4
  • 5
Important material for step 1. - bussa
#11
55. MOA of anstrazole......
56.MOA of sulfonylurea......
57.neutrophil migration is determine by what ........
58. s/ e of malaria drugs........
59.case of aseptic meningitis what's the finding on CSF.........
60h/o head injury 10 year back, recurrent rhinorrhea, what's finding on CSF......
61.gulf player weired mole on lower leg.......... what's the risk factor ...
62.case of neurofibromatosis........
63.clean wound on fig............ what's the healing process....
64. h/o alcohol taking ......patient is hypoglycemic 50% dextrose is given but patient still vomitting ..........what' s the next step of management ..........
65. what's effect of alcohol on gluconeogenesisi.
66.celiac sprue , duodenal biopsy done ........what's the other lab finding........... antibody to what.....
67. case of rota virus.........
68. case of measles .......
69. case of cerebral malaria........
70. case of streptococcus pyogne ......PSGN......
71. case of s.typhi.........
72.case of meningitis ........
72 . case of vWF defeciency.......
73.case of follicular lymphoma.........
74.case of AML.......
75. h/o radiation exposure in neck........what cancer would develope..........
76. embryo derivative of thymus which pouch.....
77. h/o autoimmuno disease ....remove the thymus.....
78. h/o die in utero in 22 weeks ..........what's defect on cytogenetics........they give placenta picture ........
79. case of turner syndrome.......
80. prevalance is 1/ 39000 what's the heterogygote frequency of child .......
81. weired pedegree .......i still don't know what they asking ......
82. epithelial lining of ureter........
83. mutation HMCII.........
84. Hb 5, polysegmented neutrophil,patient is dyspnea.......what's the next step of management ....... give boold or vit B12........
85. Hb7 / microcytic anemia ...... what's the next step of management .......measure ferritin or give iron.....
86.h/o poor nutration , fatty liver what 's the case .......
87. what make unconjugate bilirubin water soluble.....
88.stone on ureter what's effect on GFR......
89.left renal a. stenosis....what's effect on renin level.........
90 blcok the PTH recepter on kidney what's effect on PO4+, ca absorption on GI/ 1,25 dihydrocholicalciferol, . 91. hyperthyrodism........what's TSH/T4/T3........
92. grave's disease .......antibody to what .......
93. anorexa nervosa what's the FSH/ estrogen level............
94. estrogen produce by which cell ...........
95. spiderangioma is due to.....what excess on blood......
96.seminoma , lymph drian in where ....
97. h/ o hernia in early age ........what defect .......
98. left testicular vein drain in to.........
99. h/ o abortion on 12 week , beta_hCG increase ......what produce beta_ HCG........
100. MOA of Aspirin......
101. MOA of iburfen....
102. conjoined baby......what is the number of placenta / amniotic .......
103. h/o sudden headache , bood in CSF......... what's the the cause ........
104. case of temporal arteritis.......
105. pathological finding on PAN.......
106.pathology change after 48 hour in MI........
107 DOC acute gout.......
107.color of fluid in acute gout .......
108.NSAID resistance acute gout.........what's the next DOC......
109.recurrent kindey stone ........ which amino acid should add in diet....
110.suger burn smell in urine ........ which amin acid metabolism defect ........
111.what's the case of anemia in lead poisoning......
112 ????+ glycin -------.> hema
113.h/o insulin producing tumor ,glucose is 20 ..........what's the effect on fasting ............they give whole glycolysis chart .........
114.warfarin toxicity.........next step of management .........
115 second messanger on hemorragic shock........
116. how many glucose need to produce one fatty acid ......
117.h/o epigastric pain , does not responce to pain medicine ...........what's the next step of management ........
118. site for peritoneal dialysis.......
119. case of BPH.......
120. case of cor pulmonale ........
121. size of partical that can remove by mucocilliary mechanism........
122. h/o pacemaker replacement .........does PR interval always same......
123.h/o chines, pharyngeal carcinoma...........what's the case ..... tobao or EBV.........
124. pathology change on acute rejection ......
125. h/ o DM they give both kideny an ask what' s the pathology ..........papillary necrosis.......
126. case of transitionl cell carcinoma........ they give 6 fig of urinary system with different shape and size and ask which one is due to trasitiona cell carcinoma...........
127. case of delirium .......
128 case of schizhophrenia.......
129. case of displacement ........
130. case of VSD......
131. nurse give h/o hematuria and back pain..........there is no assotiation between pain and hemature later she threat to sue to doctor...........choices is facticious /antisocial.............
133. patient came to ur office with his dog...........what's ur responce before he enter the examination room.........
134.h/o diarrhea an vominting what's the MOA .......they give new bug something .................hydrophillia..........
135. case of H. influenza........
136. 8 year kid h/ o asthma.......... patient do not want to quite the smoking ............so, what's the next step of advise regarding to smoking.........
140. h/o anurea after riding bicycle (saddle injury ) where is the lesion........
141. drugs work in dista tubule.......
142. damage of pituitary stalk ........what increse.......
143. antipsychotic drugs work in which receptor .......
144. moa of BUSPIRON........
145. h/o hemorrhoids............what's the DOC ........
146.S/o TCA.........
147 h/ o depression after surgery what's the DOC.......
148. case of Osteogenic imperfecta...........
149.case of mycoplasma pneumonia..........
150. h/ o seizure multipal calcification on CT ...... subcutenious oval shape bump.........so, what's the bug.......
151. 75 year old man lower back pain .......osteoblastic errosion ....what's the case of pain.............

1. visual problem and prolactin???hypothalamus or infundibulum,,,choose infundi

Hyperprolactinaemia caused by compression of the pituitary stalk (infundibulum)

Hyperprolactinaemia may be caused by either disinhibition (e.g. compression of the pituitary stalk or reduced dopamine levels) or excess production from a prolactinoma (a pituitary gland adenoma tumour).

2. for terminationg pr synthesis,,,,atp,gtp.cgmp,catp
for terminatin protein synthesis energy req in the form of atp ?
no its gtp

The mRNA Signal
STOP Codons: UAA, UAG, or UGA, There are no tRNAs that recognize the STOP codons UAA, UAG, or UGA.
----Soluble Protein Release Factors
RF1 responds to UAA or UAG
RF2 responds to UAA or UGA
RF3, a GTPase (like EF-Tu and binds in a similar A-site location)
RF1/RF2 interact with RF3-GTP, have a similar shape as EF-Tu-GTP-aa-tRNA or EF-G, and bind in a similar ribosomal site (A-site). In a manner similar to EF-G, GTP hydrolysis drives the movement of the terminal mRNA codon into the P-site, moving the last tRNA into the E-site and off. At the same time, the polypeptide chain is released after hydrolysis of the tRNA-peptide bond.
In eukaryotes, only a single release factor, eRF, is necessary. It recognizes all three STOP codons and interacts with GTP.
A mutation resulting in a premature STOP codon is called a nonsense mutation.
Elongation consists of three distinct steps to add one amino acid
Requires three elongation factors: EF-Tu/EF-Ts and EF-G
Requires two GTPs per cycle (4 phospharyl bonds)
Occurs many times per polypeptide
The elongation cycle is similar in prokaryotes and eukaryotes.
Fast: 15-20 amino acids added per second
Accurate: 1 mistake every ~10,000 amino acids
Termination results in the release of the polypeptide chain
Requires one of the three STOP codons: UAA, UAG, or UGA.
Requires RF1 or RF2, and RF3 in prokaryotes (eRF in eukaryotes)
Requires one GTP
Each step of protein synthesis (initiation, elongation and termination) requires GTP
3. in aat defic in emphysema,,,,defect in lower lobe

1/pansystolic murmur(ASD.VSD. PDA)
2/transposition OF GREAT VESSELS(h/o 1day old baby cyanotic...)
3/staright easy qs abt deliruim
4/h/o holo systolic murmur....ans MIT REGURGITATION
5/a baby born and died (mother has h/o oligohydroamnios) ....in autopsy .....kidneyagenesis...what other finding?ans pulmonary hypoplasia.
6/ aortic regurgitation....it was an easy qs.
7/h/o autoimune hemolytic anemia........postive combs test was there.it was straight qs actually
8/h/o 3 pain one sexual prob thou it was big history ......ans somatization disorder
9/a women she always in hurry,worry abt lot of things abt her parents abt kids and ....call her hubb.... that he safely reached his office....ans generalized anxiety disorder
10/ h/o ADHD ASKING ABT MEDICATION....I CHOOSE MEtAMPHATEMNIE.
11/MOA ibuprofen
12/megalablastic anemia....actually it was h/o a patient who was on chemo agent so they were asking abt which vit defciency coz ........so ichoose megalablastic anemia
13/ h/o alcholic ......Rx B1
14/alziemer pat .....describing abt wasting it was long history thou....ans kwashoirkor
15/14 yr old kid ...dose nt want a read ,but he is good in science geography, play games ,he was active drug screen negative,,but he just dont want a read.....there r chosie but i choose 'normal 'coz teen agers,they dont want aread,and other choicees dosent match with history either.
16/ effernt arteriol constricted.....so i choose FF inc GFR inc but renal plasma filo decreease .u can chk in FA the same thing is thr.
17/there is h/o of grandfather ....thr her grand daughter she was giving all the history sho din let granda to ans doc qs ,so wat ur response i choose....tell lady to go out so doc can directly ask qs to the patient.
18/ hypertensive patient....on salt restriction.....he came for follw up...he said doc i don like taste of my food....wat ur response....thr r cpl of reponse but i choose..... give him pocket guide of salt restiction menu....
19/ h/o immigrant kid KOREA n...cant speek english....translator hired....thr r some h/o coin lesions on the bak of kid...wat ur response.....i choose....ask transalator to ask mother abt coin practice ...the qs abt folk medicine.
20/ h/o ASD(IN HISTORY fixe split s2...)
21/h/o photosenstivity.......side effect...doxycyclin.
22/ thr is a history....pat was on methotrexate...prednisone...they wr asking abt wat shld we need to ad more...... i choose cox 2 inhibitor coz... prdnisone increaase expression of cox 2 so patient can have inflammatory side effects thats the reason i choose cox 2 i dont know its right ans or not.
23/h/o nulligravida.......risk of...ans endometrial CA
24/in clinical study ...if u find a pt with MI wat u will find in ECG....i choose st segment depreesion,,,i look for st segment elevation but i din find so i end with this ans.
25/h/o married couple they were tring to have baby since 2 months,ther sexual practise normal,,they both have orgasm normal, but they fight abt the baby prob so coz of it women is depress but at work she is normal.......a sexaul arousal prob,b dysthymia, c some sexual prob i don remmebr.but i choose dysthymia .i don no its ryt or rong.
26cohort
26/methoterexate
27/h/o plantar fascitis ....pt was casher ...tenderness sole of foot from heel to front..
28/h/o long trem pain prob....had gone thru lots of dignostic measures but stil have same prob.....came to doc saying i thing u r fooling me...wat ur response......a. u can switch other doc,,b i will refer to pscycgh...c, i am doing alll appropriate measures in order... this one i choose
29/2 qs abt acid base
30/ patient was on high alltitude.....now at sea level wat will be PO2,PCO2,and PH.
31/INSULIN qs.....h/o post prandial it was in grapl
32/secretin in graph
33/pt have prob in swallowing ....had surgerythoracic....mass in mediastinum ,,,removed...wat will intact after surgery.a.lower esophageal sphincter response..some more choices i don remmeber.
34/a pt hit by a truck...rear.......?a.direct injury.d hyperextension injury,,c hypoextension ..i choose direct ..
35/8 yr bouy had accident ....sitting front....got multiple fracture,,,no hope of survival....wat ur respone to parents.a. it must be moral 4 u guys.
thr some more but stupid ans,,,,i choose the one which has assurance i dont remmber the choice .
36/77 yr old lady was on dialysis since 10 yrs she was tired of it want no more... but she was asking that am i have painful death wat ur response...............u will have some fluid retention but we r here we l do our best possible to alleviate ur problem.
37/chlamadia
38/oxidase pos....psedomonas
39/latral pontine syndrome
40/clozapine toxi
41/occulomotor 2 qs
42/hemisection cord 2 qs
43/UMN lesion
44/upper quadrantopnia....middle cerebral artery
45/picI...arrow was on choroidal artery
46/cerebral artery
47/antihistamine 2 qs
48/teenager boy who was normal,play with friends often,but now mother complaing that he sleeps althe time.not playing with pals,no dru abuse wat RX a chlodiazepoxide b flostine.c resperidone
49/ ANS MEDUIM CHAIN (MCAD)///NO KETONE BODIES
50/ urine methylmalonic acid......
51/tb patient orange yellow urine....sweat...eyes...ans rifampin
52/qs 19 yr old house....urine inc ALA
51/cholyStyramin MOA
52/LDL receptor easy history
53/46 yr old male crying that i will die early like my father?wat ur response/....i choose tell me abt ur father ....how he died.
54/dabetic pt not taking care....follow up vist wat ur response......ii choose tell me wat u know abt dibetes
55/another diabetic patient ,,taking good care of diabetes,,exersice daily....wat ur advise in end
a.pres vit E.,b fish oil.,c check ur feet every day.
56/kidney stone pat ....wat advice u will give
57/obese diabetic pt wat treatment....a,insulin....,b,glyburide,....cmetformin
58/obese pt ...gall stone...u find in ultrasound...wat wiill be next mangement.a.uricodxycolic acid.b,open stomech remove stone.,c endoscopic removal
59/chrons Rx..salfasalzine
60/chemo pt nausea vomiting...rx ondansatren'
61/moa thiazoide
62./in er pt have mi wat tx... nitrogylycerine. b beta blocker. c dgixin
63/broad ligament of uterus
64/alcholic......acut pancreatitis
65/marfan pt....cystic medil necrosis
66/substance abuse........all nihgt dancing....ans majuana
67/xray fracture arrow on navicular bone(which i did rong i m upset i knew tat but still picked wrong)
68/xray/....sholder......arrow on biceps brachi
69/wiskot aldrich
70/LB4
71/side effect of EFAVERINZE(rash history)
72/bcl2.....folicular lymphoma
73/blast crisis........cml
74/obese pt sleep apnea.......dignosiss?how
75/ca mechanism ON SR
76/colon CA ....MISMATCH REPAIR
77/membranous glomerulonephritis
78/pt ANA positive... ig G,C3 DEPOSIT....good pature..acute gn
79/women ...4 wt loss.....which drug.....amphatemine
80/ADH SOME GRAPH ON IT
81/sarcodosis...noncaseating granuloma
82/primaquin moa
83/MIF (MULLERIN INHIBITORY FACTOR)MUTATION.......
84/qs abt renin ....graph
85/no spleen....risk.... strep pnemon..
86/77 yr pt which vaccine......i choose hinfulenza vaccine&
pnemo vaccine

87/seminoma
88/staph /strep....ow diff ....ans catalase
89/p value was less then 1.....a.rejection of true null hypothesis.,b tejection of false null hypothesis
90/tuberssclerosis
91/compititive inhibitor graph
92/h'/o paranoid
93/plasmid bacteria.........antibiotic resistance
94/dorsal plantar arch
95/cor pulmo.......S3
96/benzo......MOA
97/polyuria ...which drug....lithium
98/pt feeding milk......dry vagina.....y?a.prolactin,more choice i don rember
99/rx vit k in warfarin tox
100/factor 2 def.......whr prob.......liver,heart,lung,breast
101/GnRH.....RX IN....
102/estrogen......ingraph
103/sildenafil side effect
104/allupurinol moa
105/cycloserine moa
105/copd pt .......in graph
106/asking to calculate stroke volume
107/fibrocystic change......with cycle
108/3 qs on lober pnmo
109//acute pancreatitis....coz....gall stone
110/ketone bodies...............beta hydroxybutyrate
111/slide of bone formation
112/ribosomal rna synthesis
113/osteogenesis imperfecta........procollagen prob
114/qs abt st jhon wart medication
115/asprin toxi....rx
116interferon moa
117/calculate mean...in graph describing kids having uti.
118/thr was control grp versus disease in grph ...which one have 5 yr more survival then control.
119/elongation step in tranlation
120/moa ondinavir
121/HTLV1
122/injury at L5
123/HARDY WEIN BRG.....1/1000...SOME THIN LIKE TAT
124/WERD PIC ASKING ABT WHR BACTERIA IS INFECTING.
125/CALCULATE TOATL BODY WATER
126/XRAY/KNIFE...NEAR RT VENTRICAL OF HEART
127/HOMEO BOX MUTATION...INSTEAD HEAD LEG ...SOME WEIRD THING
128/ALLOTYPE DIAGRAM
129/INH TOX


Report Abuse









Report Abuse


* Re:good for those whos exam is near
#1472987
rizowana - 09/20/08 22:18

some useful links

http://library.med.utah.edu/WebPath/ORGAN.html


http://professional.diabetes.org/Multimedia_List.aspx

here is another one.....

http://www.wiley.com/legacy/college/boye...bolism.htm


Report Abuse


* Re:good for those whos exam is near
#1473009
rizowana - 09/20/08 22:47

brachial plexus or sciatic plexus injuries
knee/ankle joint x rays with ligaments pointed out.
ct scan of the thorax and abdomen
visual pathway defects
embryology clefts arches and pouches.
in genetics read the flow chart at the end of single gene disorders in kaplan lecture notes.
sensitivity specificy and predictive values
aldosterone cortisol hormone synthesis pathway defects
catalase coagulase and beta hemolysis reaction differences in streptococci differences in neisseria and differences based on motility and fermentation in enterococci
aids defining complex which diseases dominate in relation to cd4 count
important interferons and the their origin
binding sites on immunoglobulin for macrophages and complement
complement defeciencies
coagulation pathways.
autonomic drugs classification
schizophrenic sub types
autosomal trisomies and turners and klinefelter's
amino acid metabolism defects
glycogen metabolism(enzyme def branching debranching)
fatty acid metabolism def..lcad enzyme def
urea cycle
citric acid cycle intermediates which use vitamins and co factors
zero and first order kinetics potency and efficacy


Hox transcription factors and their elusive mammalian gene targets
T Svingen and K F Tonissen

BACK TO ARTICLETable 1. Current list of mammalian Hox gene targets
Figure and tables index
Hox protein +/- Target Species Reference
Hoxa2 - Six2 Mouse Kutejova et al (2005)
Hoxa5 + p53 Mouse Raman et al (2000a)
HOXA5 Human
HOXA5 + Progesterone receptor Human Raman et al (2000b)
HOXA5 + Pleiotrophin Human Chen et al (2005)
HOXA5 + IGFBP-1 Human Foucher et al (2002); Gao et al (2002)
HOXA10
HOXB4
Hoxa9 - Osteopontin Mouse Shi et al (1999, 2001)
Hoxc8
HOXA9 + EphB4 Human Bruhl et al (2004)
HOXA10 + p21 Human Bromleigh and Freedman (2000)
HOXA10 + 3-Integrin Human Daftary et al (2002)
HOXA10 - EMX2 Human Troy et al (2003)
Hoxa10 + IGFBP-1 Baboon Kim et al (2003)
Hoxa13, Hoxd13 + EphA7 Mouse Salsi and Zappavigna (2006)
HOXB1 + COL5A2 Human Penkov et al (2000)
Hoxb3 + TTF-1 Rat Guazzi et al (1994)
Hoxb5 + SPI3 Mouse Safaei (1997)
Hoxb5 + Flk1 Mouse Wu et al (2003)
HOXB7 + BFGF Human Carè et al (1996)
Hoxb8 - N-CAM Mouse Jones et al (1992)
Hoxb9 + N-CAM Mouse Jones et al (1992)
Hoxc8 ? mgl-1 Mouse Tomotsune et al (1993)
Hoxc13 - Keratins Mouse Tkatchenko et al (2001)
Hoxd10, b6, b7, b9, c8 + Renin Mouse Pan et al (2004)

The list contains likely Hox gene targets and the Hox protein responsible for the trans-regulatory effect. The (+/-) symbols represent either a positive or negative regulatory effect on the target gene and (?) symbol indicates an unknown effect. Also, note that although there is some experimental evidence to suggest all are likely direct gene targets, not all have been exclusively verified of being so through in vivo experiments. The corresponding reference(s) for each gene target is also shown.


PAX genes are a family of transcription factors essential to the genesis of a variety of tissues and organs. Mutations leading to a loss of function in PAX3 are found in people with Waardenburg Syndrome indicating that PAX3 is necessary for the correct formation of caudal neural crest derivatives and for the migration of myoblasts into the limbs. Mutations of PAX3 leading to gain of function when fused with the FKHR gene (another transcription factor) are associated with alveolar rhabdomyosarcomas.

Pituitary-specific transcription factor (Pit-1) binding site in the human renin gene 5'-flanking DNA stimulates promoter activity in placental cell primary cultures and pituitary lactosomatotropic cell lines.

A Pit-1 binding site in the human renin gene promoter stimulates activity in pituitary, placental and juxtaglomerular cells.


The mouse Hoxd13(spdh) mutation, a polyalanine expansion similar to human type II associated with synpolydactyly (SPD), disrupts the function but not the expression of other Hoxd genes.

Hox10 or Hox11 paralogous group are disrupted provide evidence that these Hox genes are involved in global patterning of the axial and appendicular skeleton.

In the absence of Hox10 function, no lumbar vertebrae are formed. Instead, ribs project from all posterior vertebrae, extending caudally from the last thoracic vertebrae to beyond the sacral region.

In the absence of Hox11 function, sacral vertebrae are not formed and instead these vertebrae assume a lumbar identity.

Report Abuse



Report Abuse


* Re:good for those whos exam is near
#1473015
hysty - 09/20/08 22:58

how long would it take to do all these..???
Report Abuse


* Re:good for those whos exam is near
#1473107
mle99possible - 09/21/08 00:24

God bless you for all your contribution to this forum. Your kindness and generosity, selflessness everyone on this forum came to know. Each of us is praying for you, you will excel in the exam for sure for the hardwork you are doing. Bye.

HISTOLOGY

Medical Histology Index
http://www.bu.edu/histology/m/index.htm


System Tissue Epithelium Subtype:
1) Circulatory blood vessels----------------------------------- Simple squamous endothelium
2) Lymphatic lymph vessel------------------------------------- Simple squamous endothelium
3) digestive attached Lips (external portion) -------------- Stratified squamous, keratinized
digestive attached Lips (internal portion) --------- Stratified squamous, non-keratinized
digestive ducts of submandibular glands---------------------------- Stratified columnar
digestive attached gingiva ---------------------------------Stratified squamous, keratinized
digestive dorsum of tongue------------------------------- Stratified squamous, keratinized
digestive hard palate--------------------------------------- Stratified squamous, keratinized
digestive esophagus -----------------------------------Stratified squamous, non-keratinised
digestive stomach ---------------------------------------------Simple columnar, non-ciliated
digestive small intestine------------------------------------- Simple columnar, non-ciliated
digestive large intestine --------------------------------------Simple columnar, non-ciliated
digestive rectum ---------------------------------------Stratified squamous, non-keratinised
digestive anus-----------------------------------------------Stratified squamous, keratinised
digestive gallbladder----------------------------------------- Simple columnar, non-ciliated

4) Endocrine thyroid follicles--------------------------------------------------- Simple cuboidal

5) Nervous ependyma------------------------------------------------------------ Simple cuboidal

6) Integumentary skin ---------------dead superficial layer Stratified squamous, keratinized
integumentary sweat gland ducts -----------------------------------------Stratified cuboidal
integumentary mesothelium of body cavities (pleura, pericardial)--Simple squamous
7) Reproductive - female ovaries -----------Simple cuboidal germinal epithelium (female)
reproductive - female Fallopian tubes-------------------------- -Simple columnar, ciliated
reproductive - female uterus --------------------------------------Simple columnar, ciliated
reproductive - female endometrium ---------------------------------------Sim
Reply
#12
reproductive - female endometrium ---------------------------------------Simple columnar
reproductive - female cervix (endocervix) ---------------------------------Simple columnar
reproductive - female cervix (ectocervix) ----------Stratified squamous, non-keratinised
reproductive - female vagina -------------------------Stratified squamous, non-keratinised
reproductive - female labia majora ----------------------Stratified squamous, keratinised
reproductive - male tubuli recti------------ Simple cuboidal germinal epithelium (male)
reproductive - male rete testis ------------------------------------------------Simple cuboidal
reproductive - male ductuli efferentes-------------------------- Pseudostratified columnar
reproductive - male epididymis --------------Pseudostratified columnar, with stereocilia
reproductive - male vas deferens --------------------------------Pseudostratified columnar
reproductive - male ejaculatory duct---------------------------------------Simple columnar
reproductive - male (gland) bulbourethral glands ----------------------Simple columnar
reproductive - male (gland) seminal vesicle-------------------- Pseudostratified columnar
9) Respiratory Paranasalsinuses ------ ----Pseudostratified columnar, ciliated epithelium
respiratory Nasopharynx ----------------------------Stratified squamous, non-keratinised
respiratory oropharynx ------------------------------Stratified squamous, non-keratinised
respiratory Anterior epiglottis ----------------------Stratified squamous, non-keratinised
respiratory Upper ½ of posterior epiglottis--------Stratified squamous, non-keratinised
respiratory True cords (vocal cords) ---------------Stratified squamous, non-keratinised
respiratory Laryngeal vestibule---------------------Stratified squamous, non-keratinised
respiratory larynx (mostly) --Pseudostratified columnar, ciliated respiratory epithelium
respiratory trachea -----------Pseudostratified columnar, ciliated respiratory epithelium
respiratory bronchi -----------Pseudostratified columnar, ciliated respiratory epithelium
respiratory respiratory bronchioles---------------------------------------- Simple cuboidal
10) Sensory cornea---------------- Stratified squamous, non-keratinised corneal epithelium
sensory nose --------------------------------Pseudostratified columnar olfactory epithelium
11) Urinary kidney - proximal convoluted tubule ---------------Simple columnar, ciliated
urinary kidney - ascending thin limb------------------------------------- Simple squamous
urinary kidney - distal convoluted tubule-----------------Simple columnar, non-ciliated
urinary kidney - collecting duct---------------------------------------------- Simple cuboidal
urinary renal pelvis ---------------------------------------------------Transitional urothelium
urinary ureter---------------------------------------------------------- Transitional urothelium
urinary urinary bladder ---------------------------------------------Transitional urothelium
urinary prostatic urethra-------------------------------------------- Transitional urothelium
urinary membranous urethra-------------------- Pseudostratified columnar, non-ciliated
urinary penile urethra----------------------------- Pseudostratified columnar, non-ciliated
urinary external urethral orifice ----------------------------------------Stratified squamous

Report Abuse


* Re:good for those whos exam is near
#1473254
rizowana - 09/21/08 02:16

okt3 - 07/26/08 11:23

Those are posted by 2c2bd, go through and compare with yours, nobody has the right answers to NBME, the best way is to take at least 2 forms online(if you can$$) and see where you stand.
GL

NBME 5 ANSWERS
2confused2bdoctor - 06/05/08 13:15

Still some doubt on answers..ask around if any doubts..
Block 1
1 A
2D
3E
4A
5B
6A
7B
8B
9A
10D
11B
12B
13B
14B
15A
16A
17B
18E
19F
20C
21B
22D
23B
24C
25A
26E
27A
28D
29E
30B
31A
32D
33E
34C
35D
36E
37B
38G
39A
40A
41E
42A
43A
44C
45F
46B
47D
48D
49B
50C
Block 2
1E
2D
3F
4B
5D
6B
7D
8B
9B
10B
11E
12E
13A
14B
15B
16E
17F
18C
19C
20A
21D
22F
23F
24C
25A
26E
27D
28C
29D
30E
31A
32A
33B
34E
35A
36B
37E
38E
39F
40E
41A
42D
43A
44E
45D
46E
47D
48F
49A
50E
block 3
1C
2E
3D
4A
5E
6A
7A
8D
9D
10C
11C
12D
13D
14B
15C
16A
17E
18C
19B
20C
21E
22E
23B
24E
25B
26C
27A
28C
29C
30B
31D
32D
33D
34E
35E
36C
37B
38C
39D
40E
41D
42B
43B
44E
45D
46H
47A
48D
49E
50C

block 4
1A
2C
3E
4C
5D
6F
7D
8B
9C
10B
11E
12E
13F
14E
15C
16B
17D
18A
19A
20D
21A
22D
23C
24C
25A
26D
27C
28F
29F
30A
31C
32C
33C
34C
35G
36B
37C
38E
39D
40A
41E
42D
43D
44D
45C
46D
47A
48D
49E
50C
NBME 6 answers
answers.
1. C
2. I
3. C
4. A
5. D
6. A
7. A
8. C
9. D
10. H
11. A
12. B
13. F
14. E
15. D
16. D
17. A
18. A
19. A
20. E
21. E
22. C
23. D
24. A
25. C
26. A
27. C
28. D
29. D
30. A
31. A
32. C
33. D
34. C
35. B
36. B
37. A
38. C
39. F
40. D
41. B
42. E
43. D
44. D
45. B
46. A
47. B
48. E
49. C
50. D

block 2
1. C
2. E
3. C
4. E
5. B
6. C
7. C
8. A
9. B
10. E
11. A
12. C
13. B
14. E
15. A
16. B or D?
17. B
18. G
19. D?
20. A
21. B
22. D orF?
23. A
24. C
25. D
26. A
27. B
28. B?
29. C?
30. E
31. B
32. B
33. E
34. B
35. C
36. A
37. B
38. D
39. A
40. D
41. A
42. B
43. E
44. E
45. B
46. F
47. A
48. A
49. E
50. A

block 3 (these are from dr. lek, i really appreciate your work)
1 B
2 C
3 E
4 B
5 B ?
6 B
7 D
8 E (corrected after)
9 B
10 A
11 E (have no idea)
12 F
13 C ?
14 B
15 C
16 B
17 B ?
18 B
19 D
20 B ?
21 D
22 B
23 C
24 B
25 A (corrected after)
26 E
27 C
28 C ?
29 A
30 D ?
31 E
32 D
33 C ?
34 C ?
35 A
36 H
37 D
38 E (corrected after)
39 D
40 A
41 B
42 A ??
43 A
44 E
45 C
46 E
47 E
48 B ?
49 H
50 D ??


block 4
1. E
2. A
3. C
4. C
5. D
6. B
7. A
8. A
9. D
10. D
11. C
12. C
13. C
14. A
15. D
16. A
17. F
18. E
19. D
20. H
21. E
22. B
23. B
24. B
25. C
26. A
27. B
28. D
29. A
30. B
31. A
32. F
33. D
34. B
35. C
36. A
37. B
38. F
39. D
40. C
41. E
42. B
43. A
44. D
45. B
46. D
47. C
48. D
49. D
50. I

Report Abuse

Report Abuse


* Re:good for those whos exam is near
#1473284
rizowana - 09/21/08 02:42

kryptik - 08/09/08 12:13

here is a link to the COMPLETE physiology that people have been looking for a while now, it has the complete renal, reproduction, endocrine...etc. If you are missing those you just download those topics to complete your collection, if not go ahead and download the entire set. Remember its a TORRENT so you have to be patient and also seed so others can also get it. I am only providing a link i am not the uploader all credit goes to my friend the_punisher for realeasing this.
Those of you who dont know what torrent is download utorrent from http://www.utorrent.com/ to be able to download the torrent.

http://mmovp.net/forum/attachment.php?at...1218245287

credit goes to the_punisher

Report Abuse

* Re:good for those whos exam is near
#1473289
himagnaghosh - 09/21/08 02:48

hi..u seem to be quite a help on this forum .
can u guide me on nbme....which nbme form is closest to the usmle step 1 real exam?
thanks in advance.......
good luck
my email id is himagnaghosh
Report Abuse


* Re:good for those whos exam is near
#1473313
rizowana - 09/21/08 03:02

nbme 3/5/6
Report Abuse

* Re:good for those whos exam is near
#1473318
rizowana - 09/21/08 03:05

Its over!! Exam Experience
#326256
md83 - 08/07/08 09:39

Hello Everyone
Thanks to all of you for your informative posts on here. I had promised that after finishing my exam I will share my experince. Guys this exam is DOABLE. There are so many gimmies but then you have some that are just out there...and those you try to narrow down to two and click and move on.

My exam was a pretty mix of everything with a bit more pharm and path. The path, I felt like I knew the answers, but they take even easy cases and make them a bit more challenging. So if the answer doesnt scream out in front of you, quickly go re-read the questions and choose your answers carefully. My pharm was fairly stright foward. Every single pharm questions couldbe answered from first aid alone (even though I did kaplan pharm too). In my opnion the kaplan biochem and pharm books are the best things ever. Pharm is mostly basic MOA and major SE...I had an interesting question on this lady whose BMI was a 42 and they named the MOA of two drugs and you had to chosoe the two drugs...They wanted to know what drugs would help her loose weight and it was sibutramine and orlistat..I had some antibiotics (two questions on tetracylin one moa and one SE in a preg women--bone defects in the fetus). Two chemo drugs....one was moa of paxcitaxel and the other one was why would you use mesna (prevent hemorragic cystitis). But anyhow, as you can all see PHARM Is really stright foward. Just know your main drugs interactions...

Path....I had used goljan audios and unfortunately didnt have time to go over the RR book even though I had it....So i wish I had gone over the RR book...but the whole exam feels like path...I had a lot of smoking related diseases...focus on risks of certain cancers and exposures. Some nutrition....Zinc def would cause changes in taste and smell....

Beh and biostat not too bad...some questions were--what would you tell the patient....stat, mainly graphs and finding PPV, NPV...power of study

Physio--I thought some of the questions were tricky and hard...but its mostly graphs...so try skimming through the graphs in kaplan's book...Dr. Kudrath is great too (the guy that does kaplan physio).

Micro/Immuno....Micro was okay...most of the questions were stright ID...Immuno was so so...know the formation of a granuloma of TB in detail!!! Know all your major ILs and what they do...I had a question you give IL2 to this renal transplant patient....and what would it cause? hypercholestorliemia...hypertention...and some other odd choices and I had NO idea so I just chose something and moved on.

Anatomy/Neuro...it was okay...mostly neuro and images so please look at images for neuro, it will help you a lot. Look at blood supply and where different nuclie and tracts are..Brachial plexus...horner syndrome..I had a question...pt has LAD and RCA blocked....which artery would be best to graft? I put internal mammilary...

FINALLY. I CANT EMPHASIZE the importance of USMLEWORLD. I can't comment on kaplan qbank because I just used their books and not their Qbank ( i am sure its good too)..But world is a great learning tool. The layout/format of the exam is just like world....So when you're sitting in the prometric center, you feel like you're just taking another practice set of uworld at home (not sure if thats a good thing or not but it helped calm me lol). I had about 5 questions from world...

Reply
#13
good site for review ct head

elbamaritza - 07/12/08 17:06

i like this site a lot
http://www.med-ed.virginia.edu/courses/r...index.html

(posted by 2confused)

#1378971
elbamaritza - 07/12/08 17:07

and also this one
http://www.meddean.luc.edu/lumen/MedEd/R...sons_f.htm

Report Abuse

* Re:good for those whos exam is near
#1473410
rizowana - 09/21/08 04:52


#313847
radiologist007 - 06/26/08 04:32

hi 2 every1,

i m sorry for posting my message for the first time. i am a silent visitor of this forum.I am an old post-graduate, a full timer Radiologist. i have a 2 year old naughty daughter. rarely got enough time 2 read along with the job. still managed 2 get at 3-4 hours a day, 6 days a week 2 read. i prepared for 3 months 4 step 1. 2 months reading and 1 month solving UW questions.

1 week before, slowed down my reading.travelled 2 mumbai by train.1 day before enjoyed the ambient atmosphere at Lonavla. slept for 6 hours night prior 2 exam.

reached 1 hour before with lot of Tea and snacks.

started by skipping Tutorials.

gave my blocks in this manner: first 2 blocks - break (5 min) - 3rd block - break (5 min) - 4th block - break (10 min) - 5th block - break (10 min)- 6 th block - break (15 min) - 7 block - feedback (8th block = 5 min ) - done.

it is an exhausting process. u need enough of energy, concentration and ability to sit constantly in front of a Computer for 8 hours.

i did the materials only once.

kaplans lecture notes (except biochem and Behavioral) + First aid + Goljan lectures + slides + UW (skipped renal and part of cardio).

i felt exam to be 40 % experimental. in those questons, i came to the situation of 2 close options remaining which were very difficult to cross.

2 blocks were easy. 5 were just Ok.

dont know about the results - whether +ve or -ve? and if positive, about the percentile?

i will recall the questions and will put in subsequent posts.

wishing u all a very best in ur coming exam.

please pray 4 me ...............c u soon
Report Abuse





* Re:gave step 1 on 23rd june
#1361213
radiologist007 - 06/26/08 04:50

¢ Few radiology images- 3 Q on EDH, SAH, fracture humerus, CT pseodocyst of pancreas,
¢ Few slides “ leishmania, N.gonorrhea, staph, chlamydia, giardia, kaposis sarcoma, heinz bodies, Osteogenesis imperfecta, Medial collateral ligament injury on MRI knee
¢ Specimens - 6th nerve frontal view brainstem, invasive choriocarcinoma, ca pancreas with invasion, hypertrophy heart, bicuspid valve,
¢ Standard deviation “ CI
¢ Tuberous sclerosis
¢ ADPKD
¢ Median nerve injury “ 2 Q
¢ Common perinela nerve injury
¢ Colchicine
¢ Newer antilipidaemics
¢ COX pathway
¢ Nonketotic hyperosmolar coma
¢ WPW syndrome
¢ Smoking-emphysema
¢ MEN syndrome type 1
¢ Crigglar najjar
¢ Allantois
¢ BPH
¢ Hypercalcaemia in malignancy
¢ Fibroid uterus
¢ G.vaginalis
¢ Topical antifungal
¢ Methotrexate “ S phase action

Report Abuse



* Re:gave step 1 on 23rd june
#1361214
radiologist007 - 06/26/08 04:58

¢ Central DI
¢ OA
¢ Ehler danlos
¢ Turners syndrome
¢ Pseudohermaphroditism”
¢ Consolidation
¢ Pneumothorax
¢ Renal stone
¢ PDA
¢ Asthma
¢ Anticonvulsant
¢ Post #, growth factor
¢ Fe def anaemia
¢ Lead posioning
¢ Action of ADH at..
¢ Effect of a ACE inhibitor on efferent arteriole
¢ Obstrcutive uropathy
¢ Wilms tumour
¢ MIF
¢ Collagen vascular disease

Report Abuse



* Re:gave step 1 on 23rd june
#1361215
radiologist007 - 06/26/08 05:06

¢ C diphth toxin and Cl.tetani “ action
¢ Streptococci pyogens
¢ Shigella
¢ Bacillus anthrax
¢ Yersinia pestis
¢ cAMP
¢ cGMP
¢ AKT
¢ Mycoplasma pneum
¢ Resistance in HIV
¢ EBV
¢ CMV
¢ HSV-2 “drug of choice
¢ Postmenopausal Osteporosis -curve of BMD

Report Abuse




Report Abuse


* Re:good for those whos exam is near
#1473412
rizowana - 09/21/08 05:14

#303997
blackice - 05/21/08 16:42

Here are my ans for blk1. some i was able to verify - please check and repost if u find mistakes. good luck
Block 1
1. C?
2. I
3. C
4. A
5. D
6. A
7. A
8. C?
9. D
10. H
11. A
12. B
13. F
14. E
15. D?
16. D
17. A
18. A
19. A
20. E
21. E
22. C
23. D
24. A
25. C
26. A
27. C
28. D
29. D
30. A
31. A?
32. C
33. D
34. C
35. B
36. B?
37. A
38. C
39. F
40. D?
41. B
42. E
43. D
44. D
45. B
46. A
47. B
48. E?
49. C
50. D

Report Abuse





* Re:Form SIX
#1320998
blackice - 05/21/08 17:57

Block 2
1. C
2. E
3. C
4. E
5. B
6. C
7. C
8. A
9. B
10. E
11. A
12. C
13. B
14. E
15. A
16. B or D?
17. B
18. G
19. D?
20. A
21. B
22. D orF?
23. A
24. C
25. D
26. A
27. B
28. B?
29. C?
30. E
31. B
32. B
33. E
34. B
35. C
36. A
37. B
38. D
39. A
40. D
41. A
42. B
43. E
44. E
45. B
46. F
47. A
48. A
49. E
50. A

Report Abuse



* Re:Form SIX
Report Abuse

* Re:good for those whos exam is near
#1473457
rizowana - 09/21/08 07:33

Questions:

p53: inhibit Rb phospohrylation
MS murmur,
androgen insensitivity symdrome,
4-5 blood questions ( CLL, Iron deficiency anemia, vWD,)
Treatment of Aplastic anemia,
8-10 immunology questions
which cell is responsible for positive PPD,
which cell release histamine,
MOA of vincristine, 5FU,
drug to maintain patency of ductus arteriosus,
8 - 10 experimental questins usually confused between 2 options,
drug in HT with prostatic hyperplasia,
loss of skin elasticity in elderly,
BZD and flumazenil graph (Actin of X drug decreased by flumazenil identify drug X),
Contraindication of lung transplantation(optins are Severe depression, Continued smoking, Obsessive Compulsive disorder),
Treatment of recurrent ulcer, H.Pylori,
MOA of C.botulism food poisoning,
Why less amount of salmonella is required to produce gastroenteritis( I choose acholrhydria ),
Know vitamins very well especially B12, folic acid, A , D K
3-4 questin on OC pills,
inhibin - they will show u a diagram and will ask where is inhibin,
2-3 questions on diuretics,
radial nerve injury ( X-ray with # mid shaft humerus)
Compartment sundrome ( Median nerve compression)
Hemiballimus : contralateral subtlalamic nuclei
treatment of absence seizures,

I think atleast 40 of my question shave diagrams ( Xrays, CT, MRI. Electron microscopy). In some question they will give clue but in some u have to just identify the structure and will ask question related to it.



treatment of pseudomonas ( I choose ticarcillin + tazobactam but i m no sure)
MOA of 5 fluoro uracil
S. pneunomiae ( optochin sensitive)
Type 1 and type 2 muscle fibre ( name of 5-6 muscles which one is Type 1 Pectoralis major,latissmus dorsi, paraspinalis, splenius cpaitus I just choose randomly)
IL which cause fever ( IL-1)
Picture of monocyte: i choose precursor of tissue macrophage
Edema of right side of face and right arm which vein obstruction
ash leaf patch : tuberous sclerosis
HMP shunt : i chose reduced product of NADP+
Treatment of aplastic anemia ( picture of hypocellular bonemarrow)
Pompes disease cardiomegaly which enzyme
Anaerobic glycolysis ( pyruvate to lactate)
Difference between RNA polymerase and peptidyl transferase very wierd choices i dont remember
smear of CLL exacly that of web path.
Macune Albright syndrome ( women has disease what are the chances in offspring )
Pressure difference between right and left arm,
Night terror ( Will not able to recall the event after awaking),
Characteristics feature of autism ( I choose communication impairment. There is also MR but i dont know what is the answer),
anti social personality disorder,
Borderline personality,
Mechanism of resistance of Ampicillin ( Penicillinase ),
Picture of nephron where Potassium sparing diuretic will act.



Report Abuse


* Re:good for those whos exam is near
#1473475
rizowana - 09/21/08 08:29

HOX genes share a common domain which is the HOMEODOMAIN, as a result, the HOX genes are referred to as homeotic genes. There are 11 HOXA genes, and 9 HOXB, HOXC, and HOXD genes; or a total of 38 HOX genes - 39 HOX genes. The DNA sequence that codes the homeodomain is a highly conserved 180-base pair sequence termed the HOMEOBOX. The homeodomain is a helix-turn-helix structural motif that is common to DNA binding proteins, such as transcription factors.

HOX A1- inner ear malformations
(HOX A-1 overexpression- up regulation of bcl-2 (anti-apoptotic factor)

HOX A2: transformation of the pharyngeal arch 2 into the pharyngeal arch 1 with duplication of the malleus and the incus.

HOX B2: facial nerve palsy, motor nucleus deficiency of cranial nerve of pharyngeal arch 2

HOX A13 causes hand foot and genital syndrome: malformation of the thumb and great toe, bicornuate uterus, ectopic ureter openings and hypospadias.

Vertebral
Mutation of HOX C8 : A null mutation of HOX C-8 lead to transformation of the first lumbar vertebra into a 14th thoracic vertebra, and the eight rib became attached to the sternum.

Mutation of HOX 10 - no lumbar vertebrae are formed. Instead, ribs project from all posterior vertebrae, extending caudally from the last thoracic vertebrae to beyond the sacral region.

Mutation of HOX11 - sacral vertebrae are not formed and instead these vertebrae assume a lumbar identity
Reply
#14
HOX-D 13 : is associated with synpolydactyly (fusion of fingers or toes)
SHH-Polydactyly
BMP-Syndactyly

Hox A-9 : AML
Hox-11 : T-cell ALL

PAX3 “ necessary for correct formation of caudal neural crest derivatives and migration of myoblasts into the limbs
Mutation causing loss of function- Waardenburg syndrome - a group of genetic conditions that can cause hearing loss and changes in coloring (pigmentation) of the hair, skin, and eyes.
Mutation causing gain of function- Alveolar Rhabdomyosarcomas:-Fusion of PAX-3 with FKHR gene (another transcription factor)

PIT-1 “ -( Pituitary-specific transcription factor ) Mutation cause Combined Pit hormone deficiency

Pbx-1: Pre-B cell ALL

GBX and NKX 3.1: Prostrate CA



PAX 6 - aniridia in humans, misplaced eye in Drosophila

TCOF 1 - Treacher Collins syndrome

SOX 10 “ Hirschsprung disease

HOX A-9, 10, 11 - epithelial ovarian cancer
HOX B-5 congenital lung abnormality



Chapter 11: Development of the Limbs
1. An infant exhibits amelia (total absence of a limb). Findings of experimental studies in birds and mice are consistent with the possibility that the limb bud AER failed to express which of the following factors?
A. HOX gene transcription factor
B. Bone morphogenetic substance
C. LIM gene transcription factor
D. WNT gene factor
E. Fibroblast growth factor

2. Recent studies support the likelihood that syndactyly caused by failure of apoptosis between the digital rays may result from disruption of expression of which of the following factors?
A. Bone morphogenetic substance
B. Fibroblast growth factor
C. WNT gene factor
D. Insulin-like growth factor
E. LIM gene transcription factor

3. An infant is born with absence of the ulna and radius and malformations of the carpals and hand. Recent studies show that unique limb segment identities are encoded by HOX genes as follows (scapula - HOX9; humerus - HOX9HOX10; ulna,radius - HOX9HOX10HOX11; proximal carpals - HOX9HOX10HOX11HOX12; distal carpals, metacarpals, phalanges - HOX9HOX10HOX11HOX12HOX13). Therefore, disruption of expression of which of the the following HOX gene families would most likely account for this anomaly?
A. HOX9
B. HOX10
C. HOX11
D. HOX12
E. HOX13

4. An infant born with teratogen-induced phocomelia was probably subjected to which of the following teratogenic substances during the indicated sensitive period.
substance sensitive period
A.
B.
C.
D.
E.
alcohol
thalidomide
phenytoin
cadmium
retinoic acid
4-8 weeks
2-3 weeks
4-8 weeks
2-3 weeks
4-8 weeks


5. In cases of mirror polydactyly, which of the following factors, produced within the early limb bud, is both expressed by and restricted to the newly formed caudal orthotopic ZPA and the cranial ectopic ZPA?
A. BMP4
B. FGF4
C. SHH
D. IGF1
Report Abuse

ans

1. E
2. A
3. C
4. E
5. C









Report Abuse


* Re:good for those whos exam is near
#1473484
rizowana - 09/21/08 08:50

egyptwithin - 07/20/08 23:21

CARDIOLOGY
1) Patient™s pain relieved by sitting and leaning forward “ pericarditis!
2) Showed an EKG with 2nd degree AV block. Exact picture from NBME exam.
3) Scenario with a person having bicuspid valve problem “ Left heart failure.
4) Patient with angina. Treat w/ Nitroglycerin “ asked about cGMP mechanism.
5) Dehydration stimulates a mechanism involving increased renin activity.
6) Patient w/ ANCA antibodies but steroid tx not working “ Churg Strauss
7) Patient w/ sinusitis, hematuria, and steroid tx works “ Wegener™s
8) Aortic aneurysm associated anatomically to the tunica media.
9) Carcinoid syndrome indicated by high levels of 5HIAA
10) Physiologically, cardiac tamponade will have decreased cardiac output
11) Rheumatic fever has the possibility of causing complications like arthritis
12) Aortic stenosis is AGE RELATED!
13) You had to identify the location of mitral stenosis on a photo of a chest
14) You would see an overriding aorta in Tetralogy of Fallot.
15) Here™s a really weird one: you had to know the association between rectal temperature and blood flow when drinking ice water (plotted on a graph)
16) Calcium-channel blockers will treat tachycardia
17) Cyanide poisoning is associated with nitroprusside
18) You had to know the mechanism of Sildinafil (i.e. Viagra)
19) A patient had a œheavy heart “ most likely CHF and edema
20) Loop diuretics cause hypokalemia, which is assoc. with torsades de pointes
21) Mydriasis = alpha-1 agonist (options were a-1, a-2, b-1, b-2, m-1, m-2)
22) Cholecystarimine = prevents cholesterol absorption
23) Fibrates = defective because decreased chylomicron degradation
24) You had to pick the appropriate graph that showed relationship between Epinephrine & Prazosin, and then followed by a beta-blocker being given.
25) Treatment for glaucoma = only appropriate choice was Pilocarpine
26) The most anterior structure in the superior mediastinum from the answer choices given “ Trachea
27) What is the function of the peptide found in the atrium “ secretes sodium
RESPIRATORY
28) Patchy consolidation “ how is it associated to Pseudomonas?!?
29) Tracheal shift in relation to tension pneumothorax
30) V/Q mismatch is associated to - Pulmonary emboli
31) œStippling seen in lead poisoning
32) If you work for 30 yrs w/ occupational exposure, and at the same time smoke, what is the most appropriate treatment for this patient? -Stop smoking!
33) #1 cause of death in smoking “ heart disease (lungs is #1 for cancer, not death)
34) If patient has low FEV, how do you treat? Beta-agnonist
35) Dynein arm defect seen in what pulmonary problem “ Bronchiectasis
36) Pancoast tumor is associated to sympathetic trunk lesion
37) Treatment for bilateral hilar lymphadenopathy = corticosteroids
38) Function of clara cells “ protect against toxins
39) They show an oxygen dissociation curve and ask which side it shifts in relation to pH. Keep an eye on how the curve looks when it shifts, because there were different designs of curves in both directions.
40) Dipalmitoyl-phosphatidylcholine is a deficiency usually seen in what age group?
Newborns.
41) Affected in vasalva maneuvar “ Levator Ani
42) What arachidonic product is associated to hives? Leukotrine

RENAL
43) A child presents w/ red cell casts = Post-Strep Glomulonephritis
44) Person has one large kidney and one shrunken kidney “ Chronic Renal Disease
45) A woman presents w/ fever and kidney problem “ Pyelonephritis (wbc™s)
46) They show a photo of a cystic kidney & ask for its genetic assoc “ autosomal
dominant.
47) They show a diagram of a kidney & ask site of most water “ collecting duct
48) They show the same diagram & ask site that is most hypertonic
49) Function of heparin sulfate “ prevents albumin filtration
50) They show a photo with a mass at the upper poles = renal cell carcinoma
51) JG cells associated to increased renin
52) A patient had a kidney transplant and undergoes chemotherapy “ you would give him xanthine oxidase inhibitor to decrease his odds of leukemia
53) Another weird one: what happens to renin-angiotenisin-aldosterone levels when you eat a black licorice?
54) Allantois remnant = urachus
55) Weird question: What are the urine levels of calcium and phosphate after treating the patient for PTH, if the person was originally hypocalcemic?!?
56) A patient with a mass in testes likely has obstruction in the renal vein

HEMATOLOGY
57) Beta-thalassemia will show 5% increase in HbA2.
58) Megaloblastic anemia has impaired DNA purine (or pyrimidine) synthesis?
59) Megaloblastic anemia is associated to increased homocysteine levels
60) CLL occurs in patients over age 60
61) Follicular lymphoma™s association to BCL-2
62) Segmented neutrophils “ what does their presence indicate?
63) Fragmented RBC™s “ are seen in what disease? TTP.
64) You had to ID granulocytes in a photo
65) Factor 5 Leiden “ associated to Protein C defect
66) Factor 5 Leiden “ occurs due to inheritence
67) Tyrosine kinase mechanism “ associated to nerve cells
68) Where does erythropoesis take place in 40 y/o? Answer is œend of long bone (other choices were spleen, kidney, liver, pancreas)
69) Decreased LTD4 /LTDF4 is due to decreased lipoxygenase
70) Aspirin™s effects on blood = increases bleeding time by inhibiting TXA2
71) Mechanism of Cisplatin = close answers were œacts on DNA or œacts on DNApolymerase?
72) Treatment for hemorrhagic cystitis “ MESNA.
73) Longterm use of prednisone leads to what complication “ vertebral fracture.

CNS
74) Patient presents w/ TIA, what is the next best treatment option after aspirin?
75) You had to identify alpha-synuclein accumulations on a photo
76) Upper quatratinopia is assocated to the temporal lobe
77) Radial muscle is associated to Gq mechanism
78) A lesion to what structure causes plantar reflex to increase? Internal Capsule.
79) Intranuclear ophtalmoplegia is associated to MLF lesion
80) Can™t move any part of their face except their eyeballs “ Pons lesion
81) Rapid intake of sodium can lead to lesion in what structure “ Pons
82) Teenage chick comes in with migraines? Due to menses
83) Guy comes in with bitemporal headache? œTension is the cause of it.
84) Identify photo of epidural hematoma
85) Weird one: they show a photo of nerve cell, does the action potential propagate towards the cell body, away from it, or both directions at the same time?
86) What nerve cell innervates chewing? I saw no CN5 options in the choices.
87) What glial cells are seen in HIV? Microglia
88) Confabulations in a drunkard are due to a lesion in the amygdala
89) What is the precursor of GABA? Glutamate
90) Patient is dizz with movement but no ear problems “ Benign Positional Vertigo
91) An oldy lady with presbycusis is accompanied by an adult daughter, would you communicate with her in writing or via talking to her daughter? Tough choice!
92) Lesion in anterior cerebral artery affects the lower extremities
93) A person has nystagmus, can™t remember his wife, and stumbles a lot? He has alcoholic intoxication.
94) Dorsal root ganglions are neural crest derivatives
95) You are given a picture of 3 hertz spike waves “ they ask for the appropriate
treatment in this condition? Ethosuxomide.
96) They ask which drug increases potassium levels “ Succinylcholine
97) A person comes in with pinpoint pupils and is unconscious “ treat w/ Naloxone.
98) Mechanism of methylphenidate = increases serotinin levels (alpha-1 not in choices)

Musculoskeletal
99) Mechanism of third-degree burns: they go down the dermis and decrease fibroblasts.
100) Mites are seen in scabies
101) They show a picture of a person with skin patches “ Kaposi™s Sarcoma
102) Squamous cell carcinoma is identified by cytokeratin
103) Sunscreen™s mechanism = does it œblock UV or does it have a œbarrier function?
104) You had to identify Gout in a photo, pathologically
105) You had to know in a separate question stem that uric acid crystals seen in Gout.
106) SLE is associated to diffuse proliferative glomerulonephritis
107) There is perifascicular fiber atrophy in dermatomositis
108) There is an osteoclast defect in osteopetrosis
109) You had to identify osteoclasts in a photo
110) There is increased levels of phosphate in osteomalacia
111) Pain is perceived by free nerve endings
112) Stem indicated shoulder dislocation, then showed a picture and pointed to a nerve “ they must have been pointing to the Axillary nerve since its assoc w/ it.
113) You had to identify CN6 on a picture
114) A person comes in with a broken elbow, you had to identify an artery on an xray and it turned out to be the Bracial Artery
115) If you had trouble with anterior and lateral leg sensation its due to common peroneal nerve
116) Person is hit on the side of the knee with no redness = lateral meniscus injury
117) What is the treatment of rheumatoid arthritis = aspirin or etanercept (both are in the answer choices).
118) Xanthine oxidase = associated to leukemia (asked this twice!)

BEHAVIORAL
119) Factitious disorder is associated to lower levels of C-peptide
120) Factitious disorder is the underlying cause of a patient™s hypoglycemia (in another question stem)
121) You had to know the structure of insulin, such as the œpeptide portion
122) A person has visual dreams before sleeping = indicates narcolepsy
123) Weird question: there is a twin boy and a twin girl. The boy is tanner stage 3 and below normal height. The girl is tanner stage 5 and of normal height. What should do as their physician? Choices: do tests on the boy, do tests on the girl, do tests on both of them, offer reassurance, or tell them to come back after 6 months?
124) Another weird one: there is a boy age 16 who is tanner stage 5. He likes playing with girls since age 5, he feels like he™s in the wrong body, and is otherwise a normal XY, what to do? Offer reassurance.
125) There is a female who displays a œsplitting personality
126) There is another female who displays a œborderline personality
127) There is a guy who works at the veterinary and lives in an old home. He has symptoms at work, is okay at home, is okay at vacation, and exercises 3 times a week. What™s the primary cause of his symptoms: animal tatter, exercise, or hold home?
128) A married sixteen year old needs no parental consent!
129) There is a question in which the answer aws œcohort study model
130) A female presents with fine hair on her neck and back “ indicates anorexia
131) A question which had the answer of agoraphobia
132) If you shift the well/diseased curve from epidemiology to the right, that indicates increased false negatives & decreased false positives.
133) Serotonin syndrome is due to SSRI.
134) Symptom of lithium overdose is tremors.
135) You had to calculate 4 years survival time = you multiply year 1 x 2 x 3 x 4, which equals the 4th year survival time. It™s similar to an NBME question.
136) There is a confidence interval question at 95% = 2.5 % on bottom end of
guassian distribution.
137) If P < 0.05 there is a 5% chance of error
138) Rephrased and asked again, if P < 0.05 there is a minute random chance of
error.

GASTROINTESTINAL
139) Achalasia = presents with decreased myenteric plexus
140) Cirrhosis is associated to esophageal varices
141) Duodenal ulcers are treated w/ antibiotic therapy!
142) Increased bilirubin indicates obstruction in biliary system
143) Anti-HAV IgM = indicates recent Hepatitis A infection
144) Anti-HbS antigen only = indicates immunity to Hepatitis B
145) Cirrhosis associated to portal hypertension (asked twice)
146) How is cirrhosis associated to gynecomastia?
147) Pancreatic carcinoma mechanism “ how does it spread and where does it
spread to on its way to the back?
148) Diabetics have a problem where in there kidneys? Glomerulus
149) Person presents with granulomas, fissures, ulcers at ileum “ has Crohn™s.
150) Kid has blood in his feces “ you would see œgastric mucosa
151) When is the first time to start worrying about a polyp , villous or tubularvillous?
152) What artery supplies the ileum? Superior mesenteric artery.
153) Weird one: how are sigma cells associated to antrum of the stomach?
154) Secretin increases CCK
155) Again asked later of its location, found in duodenum
156) Asked about identify symptoms of insulinoma
157) Person comes in with cold intolerance and weight gain “ hypothyroidism
158) Patient showed symptoms of chvotsek™s sign, be able to identify them
159) Growth hormone causes visual defects
160) There is increased aldosterone levels in Conn™s Syndrome
161) There is increased corticotropin if adrenal mass is presented in stem
162) Why are there higher levels of T3 than T4 in scenario of this question stem? Increased conversion to T3
163) Female pseudohermaphrodite associated to congenital adrenal hyperplasia
164) What is the best way to give birth control to someone who can™t make trips to the doctor because they live too far? Via injection.
165) Flutamide = increases free testosterone
166) Propothiouracil is OK to give to pregnant women

REPRODUCTIVE
167) A picture of PID is given, asked what organism involved? Gonorrhea
168) Why is there more chance of endometrial cancer in obese women? Fat will secrete more estrogen.
169) Classic symtpoms of abruptio placentae and asked to diagnose it
170) Once a hydatidiform mole is removed, what will the placenta lack? Fetal
vessals.
171) A person has high blood pressure and previous polyhydramnios, best answer choice is pre-eclampsia.
172) There is a guy with bilateral swelling underneath both nipples: one side is rigid and the other side is motile. Does he have gynecomastia or male breast cancer? These are the two answers I had it narrowed down to.
173) What does increased estradiol cause in males?
174) What is the location of nodular hyperplasia? Peri-urethral zone
175) Incomplete fusion of paramesonephric ducts associated w/ bicorneate uterus
176) MIH causes fallopian tube regression
177) They show an FSH graph (same as in First Aid but ONLY of FSH hormone) = have to ID proper graph of FSH out of 5 different looking ones.
178) DHT™s association to the penis?
179) If person suffers a urethral laceration, fluid leaks to anterior abdominal wall
180) If there is swelling in cells after MI/ischemia “ due to ATP depletion.
181) Why are elderly more affected by benzos than younger people are?

IMMUNOLOGY
182) IL-12, INF-Y, IL-2 involved in TB
183) ID Protease Inhibitor function on a diagram
184) Anaphylactic reaction involves mast cells
185) LPS is associated to TNF-alpha
186) What is the mechanism for IgA in mammary glands? Pinocytosis
187) Clathrin = this is what™s defective if improper endocytosis occurs
188) If there is decreased beta-2 microglobulin, then there is decreased MHC-1
189) NK cells use perforin as part of their mechanism
190) Decreased NADPH Oxidase results in defect in engulfing or killing?
191) Thymoma is associated to Ach defect

MICROBIOLOGY
192) What is an effect of Chlamydia? Conjuctival scarring
193) There was a comparison between Gram œ-œ bacteria causing diarrhea
194) Enterotoxigenic E Coli causes traveler™s diarrhea on a cruise
195) Person comes in with a cat bite = Pasteurella
196) Boy recurrently infected by Strep Pyogene, why? M-protein variant
197) Different types of Strep is distinguished how? Via hemolysis
198) Nasopharynx is most likely to show what bacteria? Strep Pneumonia
199) Child has white vesicles in mouth “ most likely Coxsackie virus
200) Silver stain = used to identify Pneumocystis Jerivicki
201) Person presents w/ Leishmania = treat w/ Stilbogluconate
202) ID a picture of Giardia Lamblia
203) If a person has a renal transplant, treat him for CMV virus
204) Scenario involving specialized transduction
205) Scenario involving conjugation via direct contact
206) Cholera toxin mechanism = increases camp
207) Diptheria toxin mechanism = inhibits protein synthesis
208) Neisseria vaccine is capsular
209) Scenario in which you had to diagnose patient w/ bacterial vaginosis

BIOCHEMISTRY
210) Patient presents with a bulky stool. The only fat vitamin option in the answer choices was Vitamin E deficiency.
211) Vitamin C deficiency causes decreased proline/hydroxylation
212) Fructose 2, 6 Bisphosphate allows formation of PFK
213) Valine deficiency in patient = causes maple syrup scent in child
214) You would take less sucrose if there is decreased fructose in the patient
215) Inhibiting ALA dehydrase will cause ALA accumulation
216) Asterixis will lead to ammonium problems
217) Asbestosis is associated to people who work in a shipyard
218) Dilated blood vellses + chromosome abnormality = lead to DNA repair problem
219) Degenerate codon = 2 amino acids for 1 codon
220) Question which asked the difference between prokaryotes and eukaryotes? Prokaryotes don™t have introns.

GENETICS
221) In cystic fibrosis, chloride epithelial transport is affected
222) Patient presents with low AFP levels, high beta-HCG levels, and endocardial cushion defect “ he has Down™s syndrome.
223) Imprinting is associated to chromosome 15
224) Female that constantly laughs, & male that is obese “ associated by imprinting 225) High levels of testosterone and LH, indicates polycystic ovarian disease
226) There is a diagram in which you have to figure out its œmitochondrial inheritance

EVERYTHING ELSE
227) There is a graph in which you have to figure out it™s a œcompetitive inhibitor
228) There is a graph in which you have to compare drugs via their VMAX
229) There is a scenario where a young boy is affected by lipofuschin granules
230) There is an acid-base graph indicating increased HCO3 excretion
231) Asks the mechanism of how Amphotericin B acts on fungi
232) Patient presents with symptoms of malaria, asks what another symptom may be: blackwater fever.
233) What would be another drug you could add after 3 drugs given already, in œdrug-resistance TB? Ethambutol
234) Aspirin acts at cyclo-oxygenase
235) When you lose K+/Cl- is person hypertonic or hypotonic? Had it between these two choices.
236) What causes complete blindness in diabetes?
237) Spinal cord section that causes loss of lower leg sensation?
238) There is a photo of the diencephalon, you are asked to ID the hypothalamus
239) Photo of spina cystica (you were asked to ID what the photo is)
240) Photo of maxillary sinus (you were simply asked to ID the photo again)
241) You were asked to ID a photo of pehmphigus vulgaris (clue was œnetted IgG)
242) Asked to compare two drugs (drugs A and drug B). You had to tell them if drug B had a higher/lower/equal half-life AND higher/lower/equal bioavailability in comparison to drug A.
243) Had to simply tell what is involved in elongation in protein syntehsis “ EF-1.
244) Site that™s not transcriptionally active in a photo “ heterochromatin
245) Scenario involving a virus, answer was œreverse transcriptase
246) Had to tell how edema flows to the lungs in CHF
247) Had to know albumin levels are decreased in a scenario of edema
248) The cause of a lump over the crycoid “ thyroglossal duct.
249) What is the underlying disease that causes dilation of arteries in head and neck only? Is it due to bronchogenic cancer, DVT, or some other choices “ don™t remember them all.
250) Photo of prion vacuolization, simply had to identify it.
251) Showed a shaded part of a spinal cord and had to know its associated to vitamin B12 deficiency.
252) Showed a shaded part of a spinal cord and had to know its associated to inability to discern pain and temperature.

Report Abuse





Page 1 of 1





* Step 1 * Re: 2008 remembered questions Smile

Report Abuse


* Re:good for those whos exam is near
#1473508
rizowana - 09/21/08 09:44

BS and ethics is my weakest link

it could contain some mistakes, any correction is welcome

MEDICAL ETHICS 137 cases:

1.A 25- year old mother refused immunization for her 2-month old son. The social worker spoke to the mother. (Important for Board examination)
Next step in management: immunization should be given for the benefit of the child.
Correction: IMMUNIZATION CAN BE WITH HELD FROM THE CHILD IF PARENTS ARE AGAINST IT.BECUZ LACK OF IMMUNIZATION POSES VERY LITTLE THREAT AND IT IS NEITHER LIFE OR LIMB SAVING.
HOWEVER IF GLOBAL CAMPAIGN ON IMMUNIZATION REFUSED APPROPRIATE HEALTH AUTHORITIES SHUD BE NOTIFIED {BREAK CONFIDENTIALETY BECAUSE POSING RISK TO OTHERS}

2. A 30-year old mother refused surgery for suspected appendicitis for her 6-year old daughter. The social worker spoke to the mother. (Important for Board examination)
Next step in management: surgical removal of the appendix should be preformed for the benefit of the child. (RULE: PARENTS CANNOT WITHHOLD LIMB OR LIFE SAVING PROCEDURE TO THEIR CHILD)

3. A 16-year old boy was diagnosed with osteosarcoma of the right thigh. The surgeon recommended amputation. The boy refuses amputation. He is doing very well otherwise. He is aware that death is certain without surgery. (Important)
Next step in management: amputation should NOT be preformed.
Adolescent patients or adults who are competent in making decisions have an absolute right to determine what shall be done with their own bodies. However, most pediatric patients are not competent to make their own decisions. Please remember, children (15 years or older) are usually able to give a genuinely informed consent. Therefore physicians may respond to their request, except in a case of irreversible sterilization.

4. A 17-year old girl is a Jehovah™s Witness. She refuses a lifesaving blood transfusion. She is aware of the consequences. She spoke to the social worker. (Important)
Next step in management: blood transfusion should NOT be given because she is competent to make the decision.

5. A 5-year old girl is a Jehovah™s Witness. She requires emergency blood transfusion. Her mother refuses blood transfusion. A social worker along with two physicians spoke to the mother. (Important)
Next step in management: blood transfusion should be given because the patient is not competent. Mother cannot refuse her daughter™s treatment. (LIFE saving)

6. A 2-year old boy was brought to the ER by his parents for an injury. Physicians made the diagnosis of child abuse. There are three other children living in the same household. Both parents confess to child abuse but request the physician to keep it confidential. Social worker was involved. (Important)
Next step in management: the case should be reported to Child Welfare Agency (CWA). All children should be removed from the parents.

7. A 15-year old boy with STD (sexually transmitted disease) came to see a physician. He asked the physician not to tell his parents. (VERY IMPORTANT)
Next step in management: the physician should treat the patient and notify the appropriate health authority, but should not tell his parents.

8. A 16-year old boy wants to use a condom. He comes to the clinic for free samples. He requested the physician, however, not to tell his parents. (Important)
Next step in management: condom should be given and physician should not tell his parents.

9. A 30-year old male patient is recently diagnosed with HIV. He lives with his wife and two other children but is promiscuous. He requested the physician not to tell his wife. He lost his job recently. Social worker spoke to the patient. (Very Important)
Next step in management: physician should notify the appropriate authority (e.g. department of health) for the safety of other specific persons who are engaged in unsafe sexual practices. The physician however, should ask the patient to divulge the diagnosis to his wife and other sexual partners.

10. A very small premature infant was born in the delivery room by NSVD (normal spontaneous vaginal delivery). The attending physician decided not to resuscitate the newborn. Physician spoke to the mother. Mother started to cry. Newborn expired after 30 minutes. Is the physician liable for the newborn™s death?
Answer: NO. Please remember, no physician in the USA has ever been found liable for withholding or withdrawing any life sustaining treatment from any patient for any reason.

11. A physician picked up a car accident victim from the street and brought him to the ER in his car. He did not want to wait for an ambulance because the patient™s condition was critical. Physical examination in the ER reveals quadriplegia. Is the physician liable for this consequence? (Very Important)
Answer: YES, because the physician did not protect the neck of the patient resulting in quadriplegia.

12. A policeman brought an alcoholic patient to the ER. The policeman asked the physician to give him a sample of gastric contents by putting a nasogastric tube for laboratory study. The patient refused insertion of a nasogastric tube. (Very Important)
Next step in management: nasogastric tube should not be placed. Blood alcohol level however should be preformed. The policeman should not give orders to a physician.

13. A 60-year old man with a history of myocardial infarction (MI) suddenly develops ventricular tachycardia. A physician from another department was present. The patient needs resuscitation. (Very Important)
Next step in management: the physician must resuscitate that patient. Physician should not refuse treatment because he belongs to another department.

14. A 15-year old homosexual boy wanted to change his sexual orientation. He was not successful. He needed help. He requested the physician not to tell his parents. (Important)
Next step in management: physician should help him avoid homosexual activities. Physician should not tell his parents about his homosexual activities.

15. A 15-year old homosexual boy is brought by his parents to a physician. His parents do not accept their son™s sexual orientation. The boy refuses to change. (Important)
Next step in management: physician should tell his parent that homosexual activity is considered as an alternative life style. Parents should talk to his son but should not force him to change his homosexual activities.

16. A 16-year old girl becomes pregnant. Her mother wanted her to abort this pregnancy. The girl wanted to continue her pregnancy despite difficulties. Her boyfriend is a high school drop out. (Important)
Next step in management: physician should advise to continue this pregnancy because the girl is competent to make this decision.

17. A 15-year old girl recently becomes pregnant. She went to a doctor for abortion. She told the doctor not to tell her parents about this pregnancy. (Important)
Next step in management: abortion should be done and parents should not be notified. Please remember, strict requirements for parental consent may deter many adolescents from seeking health care.

18. Can a physician provide sterile needles for intravenous drug abusers? (Important)
Answer: YES. It reduces the risk of acquiring HIV or hepatitis. The patients should be referred to appropriate health facilities.

19. A 26-year old pregnant woman went for antenatal check up. Sonogram revealed a 27-week old fetus with erythroblastosis fetalis. Doctor recommended intrauterine fetal blood transfusion. She refused the procedure. Social worker discussed the case with the mother. (Important)
Next step in management: doctor should receive a court to do the procedure which will help the fetal condition.

(A HIGHLY DEBATABLE ASPECT.MOTHER IS LIABLE FOR WITH HOLDING LIVE SAVING PROCEDURE TO VIABLE FOETUS WHICH DOESNT PUT THE MOTHER AT RISK OF SERIOUS DAMAGE.HOWEVER AMERICAN OBSTETRIC ASSOCIATION MENTIONS NOT TO DO ANY OBS PROCEDURE ON REFUSAL OF MOTHER.MOTHER SHUD BE TOLD THAT SHE IS COMMITING AN ASSAULT AND THE CASE BE TAKEN IN COURT.COURTS ARE NOW RECOGNISING THE RIGHTS OF A VIABLE FOETUS)

20. A 20-year old man tells his doctor that he is going to kill girlfriend. She lives in the university dormitory. Doctor called the university and alerted them about the threat. However, university security people did not protect her. She was killed by her boyfriend. Who should be responsible for this killing? (Important)
Answer: the 20-year old man is responsible. Doctor did the right thing by notifying the university or the police. The university is also responsible because they did not take any preventive measures.

21. A 20-year old pregnant woman refuses cesarean section for complete placenta previa. Fetus is full-term and healthy. Social worker spoke to mother. (Important)
Next step in management: doctor can go to court to get permission for cesarean section for the benefit of the fetus.

22. A 30-year old pregnant woman ingested alcohol and illicit drugs (e.g., cocaine, crack) which are harmful to the fetus. What should a physician do? (Important)
Answer: the physician should be careful in reporting this case because the pregnant woman may not come back for prenatal care, which is important for both the mother and the fetus. However, if the baby™s urine toxicology test is positive for illicit drugs, case should be reported to CWA (child welfare agency). CWA suggests separate custody for the child.

23. A physician wants to study a group of children aging from 10-12 year old. Physician already got the consent from the parents. However, he didn™t discuss the study with the children. A child refused to participate. Should the physician force the child to participate? (Important)
Answer: no, because a child can refuse to participate in a research study.

24. A 40-year old schizophrenic patient needs hernia repair. Surgeon discussed the procedure with the patient who understood the procedure. Can the patient give consent? (Important)
Answer: yes. If a psychiatric patient understands the procedure, he or she can give the consent.

25. A 65-year old schizophrenic patient needs coronary angiography because of suspected myocardial infarction. Cardiologists explained the procedure to the patient who did not understand the procedure. Who can give the consent on behalf of the patient? (Important)
Answer: the patient™s relative can give the consent. If nobody is available to give the consent, court order should be obtained. If a psychiatric patient does not understand the procedure, he or she cannot give the consent.

26. A 25-year old woman developed postpartum psychosis. The newborn developed cyanosis due to congenital heart disease. The newborn needs cardiac surgery. Surgeon discussed the procedure with the mother. She understood the procedure. Can she give the consent? (Important)
Answer: yes, because she understood the procedure.

27. A newborn is diagnosed with either trisomy 18 or 13 with TE (tracheoesophageal) fistula which requires suregery. Mother request surgeon to repair the TE-fistula. What should a surgeon do? (Important)
Answer: surgeon should refuse to do the reparative surgery because these conditions (trisomy 18 or 13) are nonviable. If the patient survives, surgeon can put a gastrostomy feeding tube for nutrition. However, please remember that a patient with trisomy 21 (Down syndrome) with TE fistula should be operated on.
(LAW OF MEDICAL FUTILITY APPLIED HERE)

28. A 45-year old terminally ill patient wanted to die. He has pancreatic cancer and has been suffering from constant pain. He asked the physician to give him some medication which can expedite his death. What should a physician do? (Very Important)
Answer: physician cannot give any medication which will expedite the death. However, physician can prescribe medication to minimize the pain. The dose should be appropriate. Physician-assisted suicide is illegal everywhere (except in the state of Oregon).

29. A 47-year old man came to a doctor for chronic low back pain and dysuria. The diagnosis of metastatic prostate cancer was made after appropriate investigation. Should the doctor tell the bad news to the patient? (Important)
Answer: yes physician must tell the truth to the patient.

30. A surgeon wanted to perform cholecystectomy on a patient. The surgeon is not sure whether the patient has decision-making capacity. What is the next appropriate step? (Important)
Answer: consultation with a psychiatrist or neurologist may be helpful. Sometimes it is necessary to discuss the case with hospital attorneys, ethic committees, or ethic consultants. In a difficult case, the ultimate judge of a patient™s competency is a court.

31. A 45-year old widow was admitted to an ICU (intensive care unit) with ruptured intracranial aneurysm. She is comatose and is placed on a mechanical ventilator. She has a 20-year old son who did not keep any relation with his mother. However, he came to see his mother. His mother made a written proxy advance directive which indicates that her 50-year old female neighbor should make the substitute decision. Who is the right person to make the substitute decision in this situation? (Important)
Answer: 50-year old neighbor should make the substitute decision. Please remember, the most appropriate person to make the substitute decision is someone designated by the patient while still competent, either orally or through a written proxy advance directive. Other substitute decision makers, in their usual order of priority, include a spouse, adult child, parent, brother or sister, relative, or concerned friend. For a patient who has no other decision maker available, a phblic official may serve as a decision maker.

32. The right of patients to refuse medical intervention: patients can refuse dialysis, cardiopulmonary resuscitation, mechanical ventilation, and artificial nutrition and hydration, even if such a decision results in the patient™s death. A patient™s decision to withdraw (discontinue) or to withheld (not to initiate) life-sustaining treatment is not considered suicide and physician participation is not considered physician-assisted suicide. Physicians do not have any legal risk.

33. Can a medical student introduce himself or herself as a ˜doctor™ to the patient? (Important)
Answer: no. a patient can refuse a medical student from performing any procedure. However, medical students are allowed to perform a procedure under appropriate supervision If the patient agrees to that.

34. Should a bus driver hide history of epilepsy from his employer? (Important)
Answer: no. He has requested his physician not to mention his epilepsy to the employer because this would result in the loss of his job. The physician is obligated not only to his patient but to the community. The patient should notify his employer and try to find a non-driving job in the company. If the patient disagrees, physician may notify the appropriate authority for the safety of the patient and the community.

35. A 50-year old make is diagnosed with stomach cancer. He requested the physician not to tell his wife. The following day, the wife calls to inquire about her husband™s diagnosis. (Important)
Answer: the physician should not divulge the husband™s diagnosis. However, the physician should encourage the patient to reveal his diagnosis to his wife.

36. A 29-year old man is diagnosed with presymptomatic Huntington™s disease. This disease is an autosomal dominant (50% chance of having the disease in each pregnancy). He requested his physician not to tell the diagnosis to his wife. The wife wants to have children. (Important)
Answer: physician should ask the patient to seek genetic counseling and to urge him to discuss the matter with his wife. Since there is a risk of harm to the future children, physician can divulge the diagnosis to protect the future children.

37. A 18-year old man is diagnosed to have suspected bacterial meningitis. He refuses therapy and returns to the college dormitory. What should a physician do in this situation? (Very Important)
Answer: physician should report to the college authority and recommend that the suspected individual should be isolated during the course of his illness.

38. A 39-year old nurse is diagnosed with hepatitis B antigen-positive. She is working in a dialysis unit. She told her doctor. However, she did not tell the hospital authority because she is afraid to lose her job. (Very Important)
Answer: physician should ask the nurse to divulge her medical condition to the hospital authority. If she refuses, physician should notify the hospital authority for the protection of patients.

39. A 20-year old man with severe head injury was admitted to a small hospital. The patient needs neurosurgical intervention which is available in a nearby university hospital. Hospital refused to accept a patient who has no medical insurance.
Answer: university hospital must accept the patient.

40. A 30-year old man needs a second prosthetic valve. He is a drug addict. Surgeon does not want to perform surgery because the patient does not take care of himself. Is this the right decision? (Important)
Answer: no. Surgery should be performed if it is medically indicated.

41. A newborn male is diagnosed with anencephaly. His 1-year old sibling needs a kidney. His parents requested the physician to remove the kidney from the anencephalic child and to transplant that kidney in the 1-year old sibling. What should a physician do?
Answer: surgeon should perform the kidney transplant.

42. A 50-year old man is in a persistent vegetative state. Physician decided to discontinue nutrition and hydration for that patient. Is this the right decision?
Answer: yes. This is an acceptable practice in most states. Few states require clear evidence that the patient would have chosen this course.

43. A medical student requested his attending to perform a pelvic examination on a patient who is anaesthetized for appendectomy. Is this ethically acceptable?
Answer: no. The patient did not give consent to perform a pelvic examination.

44. A 20-year old woman slashed her wrists and wanted to die. She was unconscious and was brought to the ER. What should a physician do? (Important)
Answer: physician should take care of the patient. Psychiatric consultation and social worker evaluation are indicated. A suicide attempt is very often a ˜cry for help™.

45. A 90-year old man was diagnosed with having Alzheimer™s disease 10 years ago. It is difficult to feed him. He cannot recognize his family members. He developed recurrent aspiration pneumonia. What should a physician do?
Answer: physician should discuss this with the family and should respect their decision.

46. A 1-day-old infant was diagnosed with hypoplastic left heart syndrome. The patient is stabilized with the use of prostaglandin. Physician discussed this case in detail with the parents. What should the parents decide in this situation?
Answer: the parents can choose a staged surgical repair of the heart, a final heart transplantation if the organ is available, or allow the infant to die.

47. A 55-year old woman with severe developmental disability recently is diagnosed with breast cancer. Her mental age is estimated at a 2-year old level. Her family members do not want any more intervention. What should physician do?
Answer: physician should discuss this case with the hospital ethics committee members. The usual consensus is ˜not to do anything™ because of her severe mental disability. (NEVER, SHE IS COMPETENT, COMPETENT PATIENT APLLY BEST INTEREST STANDARAD)

48. A 49-year old woman with cervical cancer has a history of noncompliance. She had surgery a month ago. She missed several appointments. Can a physician force her for chemotherapy? (Important)
Answer: no. Physician can talk to her regarding the importance of chemotherapy. However, the patient must make the final decision.

49. An internist has been managing a diabetic patient for the last 10 years. The patient™s condition is progressively getting worse. The patient is also not happy with the physician™s management. What should a physician do in this situation?
Answer: physician should find another physician (e.g., endocrinologist) who might be more successful with the patient in this particular circumstance.

50. An internist recently refused to see a patient who he has been seeing for the last 5 years. Internist stated that the patient was rude to him. The patient went to see another physician who requested the patient™s medical record. What should the internist do in this situation?
Answer: internist should provide the medical records of the patient to the new physician.

51. An internist refused to see a complicated hypertensive patient who he has seen for the last 10 years. Internist did not give any notice to that patient. The patient was angry with the physician. The patient was recently admitted to a hospital with the diagnosis of stroke. Is the internist responsible for the patient™s condition?
Answer: yes. The legal charge of abandonment can arise when the physician without giving timely notice, ceases to provide care for a patient who is still in need of medical attention. Internist is not obligated to find him another physician. However, patient should have sufficient time to arrange for another physician.

52. A physician went to vacation for 2 weeks. He did not find another physician to cover him. He is very sincere. One of his patients with hypertension developed severe headache. The patient has an appointment with the doctor as soon as he comes back from vacation. The patient did not look for another physician and decided to wait. The patient suddenly collapses and was diagnosed to have intracranial hemorrhage. Is the physician responsible for this patient? (Important)
Answer: yes. The physician has a legal obligation to arrange for coverage by another physician.

53. An ophthalmologist performed a cataract surgery on a patient who went home after the operation. In the evening, the patient started vomiting and complained of severe headache. The ophthalmologist refused to accept that the symptoms were due to postoperative complications. The patient wanted to see the doctor immediately but he refused to see that patient. The patient went to the nearest ER and was diagnosed to have dislocation of the lens and partial retinal detachment. Is the physician responsible for the patient™s condition?
Answer: yes ophthalmologist failed to judge the patient™s condition seriously enough to warrant attention.

54. A 70-year old Chinese man is diagnosed to have severe osteoarthritis. He told his doctor that he is using Chinese herbal medicine. He is feeling better. However, he had two episodes of dizzy spells since he started that herbal product. What should a doctor suggest to this patient? (Important)
Answer: the doctor should suggest to discontinue the herbal product which may be causing the dizzy spells.

55. A 35-year old woman is diagnosed to have chronic throat infection. She is frustrated with the conventional medicine. She told her doctor that she is using an alternative homeopathic medicine. She is feeling much better and she has no other complications. What should a doctor suggest to this patient?
Answer: the patient can continue an alternative homeopathic medicine. Alternative medicine therapy is accepted in the society and is also used along with conventional therapy.

56. A 45 year old woman is diagnosed to have UTI (urinary tract infection). She told her doctor that she could not afford to purchase antibiotics. However, she is using herbal medicine that is cheaper. She is complaining of fever and dysuria. What should a doctor suggest to this patient? (Important)
Answer: the patient should discontinue the herbal medicine immediately and should start antibiotics as soon as possible.

57. A 13-year old boy with suspected meningitis refuses therapy. His parents also support that decision because they are supposed to go on vacation the following day. What should a physician do in this situation? (Important)
Answer: the patient should be admitted and treated in the hospital. If they refuse, legal action should be taken. (MENINGITIS TREATMENT IS LIFE SAVING)

58. A 2-year old girl is admitted with the diagnosis of intestinal obstruction. Her mother has a psychiatric problem. Her mother is not capable of giving the consent. Her father died one year ago. What should a surgeon do in this situation?
Answer: legal steps may be taken to provide a surrogate decision-maker.
(IN EMERGENCY LIKE THIS ONE USE IN LOCUM PARENTIS I.E PHYSICAIN DECIDES)
59. A 67-year old widow has been using hypnotics for the last 5 years. She is addicted. Her doctor wants to withdraw her from her present medication by trial on placebos. Is the physician making a right decision? (Important)
Answer: no. The physician cannot use placebos because his decision is deceptive. The problem of addiction should be discussed directly with the patient. The use of deceptive placebo is indicated in the following conditions:
(a) the patient insists on a prescription;
(b) the patient wishes to be treated;
© the alternative to placebo is either continue illness or the use of a drug with know toxicity;
(d) high response rates to placebo (e.g., postoperative pain, mild mental depression).

60. A 50-year old man is diagnosed to have multiple sclerosis. In the morning, the surgeon asked the man his opinion on the surgical procedure and he agreed. In the evening, the man refused to give consent for the same surgical procedure. He is also disoriented to place and time. Is the patient capable of making the decision?
Answer: no the patient has impaired capacity.

61. A 55-year old woman with diabetes is diagnosed to have gangrene on both feet. She was brought to the hospital. She told the doctor the she is feeling fine and she has no medical problems. Can she give consent for the amputation of both legs?
Answer: no. The appointment of a surrogate should be sought to get the consent for the surgery.

62. A 17-year old boy came to a surgeon for bilateral vasectomy. He is the father of one child and does not want to have any more children. He does not want to tell his girlfriend and parents. He lives with his parents. What should a surgeon do in this situation? (Important)
Answer: surgeon should not perform bilateral vasectomy and should offer him less radical alternatives. Please remember, a mature minor may not comprehend the implications of this procedure.

63. A 16-year old girl came to a doctor for bilateral tubal ligations. She is a mother of one child and does not want to have any more children. She does not want to tell her boyfriend and parents. She lives with her parents. What should a doctor do in this situation? (Important)
Answer: obgyn doctor should not perform bilateral tubal ligation and should offer her less radical alternatives.

64. A 16-year old boy wants to donate one of his kidneys to his friend who is suffering from ESRD (end stage renal disease). The boy™s parents did not agree with his decision. What should a physician do in this situation? (Important)
Answer: the physician cannot accept his kidney. However, he can donate one of his kidneys if his parents agree.

65. A 15-year old boy wants to participate in a research study. He told his parents who did not agree. He lives with his parents. Can this boy participate in the research study?
Answer: no the boy needs consent from his parents to participate in a research study.

66. A 17-year old boy lives independently. He is married and has one child. He wants to participate in a research study. Does he need his parents permission? (Important)
Answer: no. He is an emancipated minor who lives independently from his parents physically and financially.

67. A 70-year old man is diagnosed with terminal esophageal cancer and requires an insertion of a gastrostomy tube. He has signed a DNR (Do Not Resuscitate) order about a month ago. Should the preexisting DNR order stand or be suspended during the surgical procedure? (Very Important)
Answer: attending physician, surgeons, and the patient or surrogate should discuss the matter and either affirm or suspend the order in anticipation of surgery. If a patient is competent and wishes a preexisting DNR order to stand, resuscitation should not be performed in the event of an intrasurgical arrest.

(BAD ANSWER, DNR ONLY REFERS TO CPR AND NOTHING ELSE IF NOT CLEARLY
MENTIONED IN DNR ORDERS) MY ANSWER IF PT COMPETENT ASK SPECIFICALLY TO GASTROSTOMY.AND PROCEDE AS WISHED BY PT.IF PT INCOMPETENT ASK FAMILY FOR PTS WISH.IF CONFLICT AMONG FAMILY ON WISH OF THE PT GO TO ETHICS COMMITEE. (REMEMBER IF CONFLICT AMONG FAMILY IS NOT BASED ON WISH OF PT.GO FOR THE WISH WHOEVER TRULY REPRESENTS IT.)

Reply
#15
68. An infant, born at 30 weeks gestation, appears to be SGA (small for gestational age) with multiple malformations. Amniocentesis study was not performed. Infant needs resuscitation at birth. What should a physician do in this situation? (Important)
Answer: physician must resuscitate the patient in the delivery room because the diagnosis is uncertain.

69. A 60-year old man is diagnosed with terminally ill colon cancer and needs resuscitation. He did not sign a DNR order. The physician has decided to perform a ˜slow code™ on his own. Is this the right decision? (Important)
Answer: no. Please remember, a performance of ˜slow code™ or ˜show code™ is not acceptable to the patient. This decision by the doctor represents the failure to come to a timely and clear decision about the patient™s resuscitation status.

SLOW CODES REFER TO WHEN FAMILY SAYS TO DO CPR BUT PHYSICAIN THINKS ITS FUTILE AND JUST TO SHOW FAMILY DOES CPR WITHOUT FULL PROTOCOL.

70. A 20-year old man is diagnosed with suspected bacteremia and meningitis. He refuses antibiotic therapy. He collapses and requires resuscitation. What should a physician do in this situation?
Answer: the physician should resuscitate the patient despite the patient™s refusal to antibiotic therapy. THOUGH ANTIBIOTICS STILL NOT USED.

71. A 50-year old woman is diagnosed with severe aortic stenosis. She collapsed in a doctor™s office and is required resuscitation. She is waiting for valve replacement surgery. What should a physician so in this situation?
Answer: this condition is called ˜physiological futility™. In severe aortic stenosis, vigorous resuscitation is highly unlikely to restore adequate cardiac output. Therefore, the physician might reasonably refrain from resuscitation.
PHYSIOLOGICAL FUTILITY IS WHEN CHANCES OF SURVIVAL WITH CPR LESS THAN 1%.
72. A 14-year old boy is diagnosed with terminally ill cancer. He is not responding to chemotherapy. His parents want to continue the treatment. However, the boy does not want to continue his suffering. The physician told the parents that chemotherapy will not be helpful. What is the appropriate decision at this point? (Important)
Answer: the boy™s decision should be respected because the treatment is futile.
(WRONG PT MINOR PARENTS REQUEST SHUD BE UPHELD)GIVING CHEMOTHERAPY DOESNT PUT PT ON LIFE AND DEATH SITUATION
73. A surrogate pregnant mother made a surrogacy contract with a couple in which she will give the baby to the couple. She developed complications in the first trimester and wanted to abort. Is she allowed to do that? (Important)
Answer: yes. If her life or health becomes threatened from continuing the pregnancy, she should retain her right to abortion.

74. A physician became sexually involved with a current patient who initiated or consented to the contact. Is it ethical for a physician to become sexually involved?
Answer: no. Sexual involvement between physicians and former patients raises concern. The physician should discuss with a college or other professional before becoming sexually involved with a former patient. The physician should terminate the physician-patient relationship before initiating a romantic or sexual relationship with a patient.
IDEALLY ON STEPS SHUD BE HER DOC BUT SUPRESS UR OWN FEELINGS
75. A physician decided to take care of his own family members and relatives. He is a very smart physician. Is this a right decision?
Answer: no. The physician should encourage all friends and family members to have their own personal physician.

76. A male patient wants to have a copy of his medical records. What should a physician do in this situation?
Answer: the physician should retain the original of the chart. Information should only be released with the written permission of the patient or the patient™s legally authorized representative (e.g., attorney).

77. A 30-year old female wants to have an abortion. Her physician objects to abortion on moral, religious, or ethical grounds. What should a physician do in this situation? (Important)
Answer: physician should not offer advice to the patient.
IF SHE IS NOT COMFORTABLE DOING ABORTION ON MORAL GROUNDS REFER THE PT TO SOMEONE ELSE.

78. A physician sees patients at a reduced fee. He spends very little time with each patient. Is the physician doing the right thing?
Answer: no. The physician is not providing optimal care.

79. A surrogate pregnant mother signed a surrogacy contract with a couple. Male partner gave sperms which were artificially inseminated to the surrogate mother. Surrogate mother has a genetic relation to the child. She wants to void the contract after the baby is born. Is she allowed to breach the contract? (Important)
Answer: yes. Surrogate contracts, while permissible, should grant the birth mother the right to void the contract within a reasonable period of time after the birth of the child.

80. A surrogate pregnant mother signed a surrogacy contract with a couple. Both male and female parents gave sperm and ovums respectively. The surrogate mother wants to void the contract and she has no genetic relation. Is she allowed to breach the contract? (Important)
Answer: no. Genetic parents have exclusive custody and parental rights.

81. A surrogate pregnant mother signed a surrogacy contract with a couple. The couple got divorced. Male partner gave sperms and the female partner gave ovums. They do not want to continue the surrogate pregnancy. What should be the decision at this point? (Important)
Answer: the couple is genetically related to the fetus. They have the right not to continue with this pregnancy.

82. A surrogate pregnant mother signed a surrogacy contract with a couple. The couple got divorced. Male partner gave sperms but the female partner could not give ovums. They do not want to continue the surrogate pregnancy. What should be the decision at this point? (Important)
Answer: female partner has no right to terminate this pregnancy because she has no genetic relation. Surrogate mother has genetic relation and she has the right to continue this pregnancy even if the male partner disagrees.

83. A 3-year old girl is diagnosed with blood cancer. She has been waiting for an umbilical cord transfusion. Her mother delivered a newborn baby girl. Umbilical cord blood was obtained and was transfused to the 3-year old sibling. What is the duty of the physician?
Answer: physician should obtain an informed consent of the risks of donation and he or she should follow the normal umbilical cord clamping protocol. Physician should protect both the children.

84. A 31-year old man has decided to donate one of his kidneys for a large amount of money. Is this the right decision?
Answer: no. However, the donor can receive some payment to cover his medical expenses. Only the potential donor not the donor™s family or another third party may accept financial incentive. Payment should occur only after the organs have been retrieved and judge medically suitable for transplantation.

85. A couple has decided to have a child through artificial insemination. They asked the physician for sex selection of the child. What should a physician advise in this situation? (Important)
Answer: physician should not participate for sex selection for reasons of gender preference. However, sex selection of sperm for the purpose of avoiding a sex-linked inheritable disease is appropriate.

86. A 30-year old man has donated his sperms which were kept frozen. He died in a car accident. He did not leave any specific instructions regarding sperm donations. His wife wants to make use of them. A woman requested her to donate his sperms. What is the appropriate decision? (Important)
Answer: the donor™s wife can use the semen for artificial insemination but not to donate it to someone else. The donor should give clear instructions at the time of donation. The donor has the power to override any decision.

87. The donor and recipient of sperms are not married. Who would be considered the sole parent of the child? (Important)
Answer: the recipient. Except in cases where both donor and recipient agree to recognize a paternity right.

88. The residents and medical students were asked by an attending to follow certain orders for a patient. The residents and medical students believe the orders reflect serious errors in clinical or ethical judgment. What is the appropriate way to handle the situation? (Important)
Answer: The residents and medical students should not follow those orders. They should discuss with the attending issuing those orders. They should also discuss the situation with a senior attending physician, a chief of staff, or a chief resident.

89. A physician used a newly prescribed drug to his patient. The patient got sick after the drug was ingested and required hospitalization. Should the physician report this drug™s side effect to FDA (Food and Drug Administration)? (Important)
Answer: yes. FDA should be notified only if the drug causes serious adverse events such as those resulting in death, hospitalization, or medical or surgical intervention.

90. A 39-year old female has been suffering from chronic cholecystitis. The surgeon advised cholecystectomy. The patient wants a second opinion. The surgeon agreed. The patient went to another surgeon and has decided to be operated by the second surgeon. What should the second surgeon do in this situation? (Important)
Answer: the second surgeon should accept the patient because the patient has the right to choose the surgeon. First surgeon should accept the patient™s decision.

91. A 45-year old male was admitted to the hospital with mild chest pain. He wants to leave the hospital before completion of therapy. How do you manage the patient? (Important)
Answer: The patient is asked to sign a statement that he is leaving against medical advice (AMA). The patient may however leave without signing that statement. This document is a legal evidence that the patient was warned by the physician about the risk of leaving. Please remember, discharge AMA does not apply to children.

92. A 55-year old man requested his physician to misrepresent his medical condition to receive disability or insurance payment. What is the appropriate response of the physician in this situation?
Answer: The physician must refuse that request.

93. What is the responsibility of a fellow physician who is aware of drug abuse, alcohol abuse, or psychiatric illness of his colleagues or of a medical condition that is harmful to patients? (Very Important)
Answer: the physician should protect the patients. The fellow physician should report to the appropriate authority (i.e., report to the hospital authority; report to the Dean for a medical student™s problem).

94. A 60-year old male has been suffering from severe pain due to terminal prostate cancer. The patient is receiving lower doses of narcotics and sedatives. How can a physician relieve his suffering? (Important)
Answer: The physician should increase the dosage of narcotics and sedatives up to the maximum recommended amount. Listening, spending more time with him, and reducing psychological distress can reduce the suffering.

95. A 25-year old female medical student or resident noticed a mistake made by a junior attending physician during rounds. She is afraid of that attending physician. What is the appropriate way to handle the situation? (Important)
Answer: She should discuss the situation with a more senior attending physician for appropriate interpretation, advice, and assistance.

96. A 26-year old male medical student or resident made a mistake during patient care. He is afraid of what might result. What is the appropriate way to handle the situation?
Answer: He should disclose the mistake to the attending physician and try to learn from that mistake. The patient should be notified as well.

97. A 63-year old female health care worker is concerned about taking care of patients with HIV infection or multidrug-resistant tuberculosis. What is the appropriate way to handle the situation? (Important)
Answer: The physician should provide appropriate care to patients despite personal risk. Institutions should reduce the risk of infection by appropriate equipment, supervision, and training. Her concern should be taken seriously.

98. A 30-year old male physician has an opportunity for financial incentive if he sees more patients and refers them unnecessarily. What is you opinion about this?
Answer: The physician should provide only care that is in the patient™s best interest.

99. Two physicians are discussing a case inside the elevator of a hospital. What is your opinion about this?
Answer: They should not do that because they have to maintain the patient™s confidentiality.

100. The patient™s confidentiality should be maintained except in the following situations:
Physicians should override third parties in case of domestic violence, child abuse, elderly abuse, gunshot wounds, syphilis, and tuberculosis. They should report these cases to appropriate governmental authorities.

101. A physician is experiencing a very difficult ethical issue regarding a complicated case. He is confused. What should be the next step?
Answer: he should discuss the matter with other faculty members in his health care team, colleagues, or hospital ethics committee.

102. What is the final plan of action in an ethical issue?
Answer: Both patient and physician should agree regarding final management. The patient should be well informed about the medical condition. The physician should be sympathetic and knowledgeable regarding the relevant medical condition.

103. DNR (do not resuscitate) order. This is appropriate if the patient or surrogate signed that order or if CPR (cardiopulmonary resuscitation) would be futile. Physicians should write DNR orders and the reason for them in the chart. Please remember, œslow or œshow codes are not acceptable. Foods and fluids are considered therapies that should be stopped. (Important)

104. (A) Brain death (adult): (Important)
(i) Definition by the President™s Committee:
Death is an irreversible cessation of circulation and respiratory functions or irreversible cessation of all functions of the entire brain and brain stem.
(ii) The criteria of brain death by the staff of Massachusetts General Hospital and the Harvard Committee:
Death occurs when there is absence of all signs of receptivity, responsivity, and all brainstem reflexes, and the EEG is isoelectric. Sometimes metabolic disorders and intoxications may mimic the above findings.
(iii) The guidelines of brain death:
(a) The diagnosis should be made also by another physician and confirmed by clinical findings and EEG.
(b) The family should be notified. They should not make the decision about discontinuing medical treatment except in a situation where the patient has directed the family to make the decision.
© The physician should discuss with another physician before removing supportive measures (e.g., ventilators).
(d) Family members may request organ donation, and in many states physicians may request the family to make an organ donation.
(B) Brain death (children):
(i) Definition: same as in adults.
(ii) Criteria: similar in children and adults, but the period of observation is longer in children.
Children 1wk-2mo of age: two separate examinations 48 hours apart
Children 2mo-1yr of age: two separate examinations 24 hours apart
Children more than 1 yr of age: two separate examinations 12 hours apart
Spontaneous movements must be absent, with the exception of spinal cord reflex withdrawal and myoclonus.
Generalized flaccidity should be present. The presence of clinical criteria for 2 days in term and 3 days in preterm infants indicates brain death in majority of asphyxiated newborns. The absence of cerebral blood flow on radionuclide scan and silence of electrical activity on EEG are not always observed in brain-dead newborns. There is no universal consensus about the definition of neonatal brain death. The decision is made after discussion with the family and health care team. If there is difference of opinion, the ethics committee should be consulted. The decision is made on the basis of what is in the best interest of the infants and children.

105. Practice guidelines for physicians:
(i) The best way to practice medicine is to select useful diagnostic techniques and therapeutic measures which are most appropriate to a particular patient and clinical condition.
(ii) Practice guidelines can reduce the health care costs, which improve health care to patients who even do not have adequate health care benefits.
(iii) Please remember, guidelines do not and should not be the only way of managing an individual patient.

106. Some important points about patients:
(i) For a patient with an incurable disease, the major goal of therapy should be the enhancement of the quality of life.
(ii) The patient care begins with a personal relationship between the patient and the physician. If a patient has confidence on the physician, reassurance may be the best therapy. The patient must understand that the physician is giving the best possible care available.

107. Patients who do not have decision-making capacity about their medical care:
The patients who do not have decision-making capacity usually arrange for surrogates who make decisions for them. Their choices depend on their values. Psychiatrists are helpful in mentally impaired patients. Family members are usually the surrogates, because they know the patients very well.
Advanced directives: statements made in advance in case patients lose their decision-making capacity in the future. These directives indicate the names of surrogates and which interventions are acceptable or not acceptable to them. These are achieved by oral conversation (most common form), living will, health care power of attorney, or physicians can ask the patient in advance.
Absence of advance directives and surrogates: physicians can make the decision using all information and should respect the patient™s values. Physicians must know the laws of the state in which they practice.
Patient preferences are known:
The decision is made with the patient™s best interest in mind.
Disagreements between potential surrogates or between the physician and surrogate: Physicians can consult with the hospital ethics committee or with other physicians. The courts should be only the last resort.

108. Down syndrome with different medical conditions: (Very Important)
(a) Duodenal atresia at birth: surgical repair is recommended as it is done regularly.
(b) VSD (ventricular septal defect) in newborn period: initial conservative medicals management is followed by surgical repair as it is done regularly.
© Cyanotic heart disease at birth: immediate medical management, which is followed by surgical repair, as it is required routinely.
(d) Cosmetic surgical condition (e.g., rhinophyma or big nose): there is no urgency to repair the underlying condition, but it can be done as it is performed regularly.
(e) Neural tube defects (e.g., meningomyelocele): surgical repair is recommended as it is done regularly.
Please remember, a patient™s management should be discussed with his/her parents and the decision made with the best interest of the patient in mind.

109. A healthy male patient with Down Syndrome lives independently. He went to a doctor for facial cosmetic surgery. Can he make his own decision? (Important)
Answer: yes. The patient can make his decision if he understands the procedure and the consequences. He lives independently which indicates that he is capable of making his own decision.

110. A patient went to the doctor due to throat pain. The doctor asked the patient what her problem was. The patient said that she woke up at six o™clock in the morning, went to the bathroom, ate breakfast, and went to drop her children at school. She then came back home, stared cooking and continues to talk about irrelevant things. What should the doctor do to stop the patient from rambling? (Important)
Answer: the doctor should ask the patient to tell him what problems she has related only to her throat.

111. A patient went to a doctor for abdominal pain but remained quiet throughout the visit. He did not tell the doctor enough about his symptoms. What should the doctor do? (Important)
Answer: the doctor must ask the patient detailed questions about his abdominal pain. It is the doctor™s obligation to find out as much as he can about the patient. Without enough information, the doctor will not be able to make an accurate diagnosis.

112. A patient walked into his doctor™s office with acute abdominal pain. He has been suffering from ulcerative colitis. The patient is noncompliant and did not visit for the past six months. What should the doctor do in this situation? (Very Important)
Answer: The doctor should find out more about the patient™s abdominal pain before making any other decision. The doctor should always be responsible with the patients.

113. A terminally ill pancreatic cancer patient with multiple metastases is admitted to the hospital.
He is in critical condition. The patient wants to know his prognosis. What should the doctor say?
Answer: the doctor should tell the patient politely that he will discuss his condition with him and his family. The doctor should never specify the longevity of the patient.? The doctor should tell the truth even when the patient is a child. The doctor should not hide any medical information from the patient.

114. A patient is recently diagnosed with cancer. He is nervous but is eager to know about his medical diagnosis. What should the doctor™s reply be? (Important)
Answer: the doctor should gently tell the patient his condition.

115. A patient is recently diagnosed with cancer. Previously, he had an episode of a nervous breakdown after hearing a family death and had to be admitted to a hospital. He loves his family members and tends to be very open with them on all issues. How should the doctor tell the patient about his current state? (Important)
Answer: the doctor should call his family members and discuss the patient™s medical condition openly and politely.
(WRONG IF DISCLOSING BAD NEWS CLEARLY PUTS PT IN NERVOUS BREAKDOWN CONDITION WAIT AND GRADUALLY TELL THE PT.NEVER BREAK CONFIDENTIALETY)

116. A male patient was admitted with severe myocardial infarction. He was admitted to the ICU and his condition is very critical. He does not know the reason for his admission. The patient is unstable. What should the doctor tell the patient? (Important)
Answer: the doctor should wait until the patient is stabilized and then gently tell him his medical condition.
(HIDING SOMETHING FROM THE PT MAKES HIM MORE SUSPICIOUS AND MAY LEAD TO ALL KIND OF MISCONCEPTION IN PTS MIND SO NEVER WAIT TO TELL PT UNLESS TELLING IS CLEARLY DETREMENTAL)
117. A mother gave birth to a premature baby who was admitted to the NICU (neonatal intensive care unit). The baby is on a mechanical ventilator. The mother wants to hold the baby. What should the doctor do in this situation? (Important)
Answer: the mother should be allowed to hold the baby.

118. A male patient is recently diagnosed with HIV. Should the doctor ask about his sexual orientation (i.e., male, female, or both)? (Important)
Answer: yes, the doctor should ask the patient directly but politely about his sexual orientation.

119. A homosexual male patient went to a doctor. The patient™s partner was recently diagnosed with HIV. Should the doctor ask the patient whether his partner is penetrating him or he penetrates his partner?
Answer: yes, because the person who is being penetrated has a higher incidence of HIV due to trauma in perianal area.

120. A 6-year old boy comes to the ER after drowning. He expired in the ER despite appropriate resuscitations. The family members became angry which is a reflection of a sense if guilt and helplessness. What is the appropriate way of giving information to the family members?
Answer: the physician should give the information clearly and compassionately when there is no hope for survival. Parents need to know that everything was done to save the child.

121. A pregnant woman who is Rh (-) ve became sensitized. She had H/O induced abortions. Her husband is not aware of his wife™s previous abortions. He wants to know from the physician how she became sensitized. (Very Important)
Answer: the physician should tell the man to ask his wife. The physician should not mention anything about the patient™s H/O abortions.

122. A mother brought her infant to the ER. The radiologist test reveals old fractures of the ribs. She did not know anything about that. (Very Important)
Answer: this is a case of child abuse. This case should be reported to child welfare agency.

123. A physician is examining a child with respiratory distress. The child™s mother became anxious during the physical examination. Please remember, a patient™s management should be discussed with his/her parents and the decision made with the best interest of the patient in mind. (Important)
Answer: child abuse.

124. A mentally retarded patient became pregnant. The patient does not want an abortion. Her mother and husband want an abortion. What should a physician do in this situation? (Important)
Answer: abortion should not be performed.
BCUZ SHE IS COMPETENT UNLESS PROVED OTHERWISE
125. A male physician is examining an adolescent or adult female patient. What should a physician do in this situation? (Important)
Answer: a chaperone should be present during the physical examination. The same rules apply when a physician is examining a patient who appears to be seductive. (Important)

126. A female physician is examining an adolescent or adult male patient. What should a physician do in this situation? (Important)
Answer: a chaperone should not be present during the physical exam.

127. A suspected HIV patient expired in a car accident. He signed for organ donations. What
should a physician do in this situation?
Answer: his HIV status is not certain. The organs can be preserved until the HIV status is confirmed. If the test for HIV is positive, organs should be discarded.

128. A patient who expired in a car accident signed in his license for organ donations. His license has expired. He always wanted to donate his organs. What should a physician do in this situation? (Important)
Answer: physician cannot accept organs because the signed consent has expired.
ASK FAMILY MEMBERS ABOUT WISHES OF THE PT IF HE WANTED TO DONATE ORGANS.JUST AHVING AN ORGAN DONOR CARD DOESNT TELL PT INTENT.MAY BE HE SHUD HAVE DONE TO SHOW OFF OR GET A BONUS OR FORCRD BY SOMEONE.EVEN IF DONOR CARD HAD NOT EXPIRED AND FAMILY UNANAMOUSLY SAID THAT PT DIDNT WANTED TO DONATE ORGANS DONOT ACCEPT ORGANS FOR TRANSPLANT

129. A male physician sexually harassed a female patient during the physical examination. The patient complained to a nurse. What should the nurse do in this situation?
Answer: the nurse should tell the patient to make an official report to the hospital authority or to an appropriate agency.

130. A chronic male smoker comes to the physician for his heart problems. The physician wanted his patient to quit smoking. What should the physician advise in this situation?
Answer: the physician should ask the patient to quit smoking immediately because patients are usually more responsive when they are ill. The physician should assist the smoker to move one step closer to quitting.

131. A terminally ill patient did not sign a DNR (do not resuscitate) order, however, he signed a DNI (do not intubate) order. What should the physician do in this situation?
Answer: the physician should follow his orders i.e., the patient should be resuscitated but should not be intubated, despite severe hypoxic condition of the patient.

132. A terminally ill patient signed a DNR order, however, he did not sign a DNI (do not intubate) order. He wants to be intubated but not resuscitated. What should a physician do in this situation?
Answer: the physician should follow his orders i.e., the patient should be intubated but should not be resuscitated.

133. An adolescent car accident victim was brought to the ER in an unconscious state. The patient needs immediate surgical interventions. The surgeon was unable to contact any family member to obtain consent. What should a surgeon do in this situation? (Important)
Answer: the surgeon should do the procedure without waiting to obtain consent for the benefit of the patient.

134. An obgyn doctor is recently diagnosed with HIV infection. He is receiving medication for HIV. His physical and mental conditions are normal. Should he tell his patients or fellow physicians about his HIV status?
Answer: no, however, the doctor should take appropriate precautions for infection control. He does not have to tell his fellow physicians about his HIV status including the physicians who are referring patients to him. The doctor is allowed to see patients if he takes appropriate precautions. However, he should notify the hospital authority.

135. A physician is scared of seeing an HIV patient with an open wound. Can a physician refuse to see a patient?
Answer: yes, however, a physician™s refusal to see a patient is unethical but is legal.

136. An elderly semi comatose patient may require surgical intervention. His family members are confused about the surgery. They asked the surgeon for his opinion. What should the surgeon™s response be?
Answer: the surgeon can give his opinion and act as a moral surrogate for the benefit of the patient. (Very Important)
137. A 12-year old boy is diagnosed with a terminal illness (e.g., malignancy). He asked the doctor about his prognosis. His parents requested the doctor not to tell him the bad news. What should the doctor do in this situation? (Very Important)
Answer: the doctor should tell the truth politely and compassionately to the patient.
YES TOTALYY CORRECT CHILD NEEDS TO BE BE TOLD OF HIS TERMINAL ILLNESS IF HE CAN UNDERSTAND THE SITUATION AND IRREVERSIBILTY OF DEATH.TRY TO CONVINCE PARENTS FIRST.

138. A 55-year old woman is recently diagnosed with right breast cancer. The doctor told the patient that she would require surgery for removal of the right breast. She started to cry. What should a doctor do in this situation?
Answer: first, the doctor should give her some tissue paper for wiping her tears. Then, the doctor should be sympathetic to her and console her. He might tell her that similar reactions are usually expected from other patients with breast cancer. Please do not mention that she will be fine with a breast implant or without a right breast because she is already 55-years old.

139. A mother is carrying a 500 gram premature fetus, which develops acute fetal distress. The physician wanted to perform a cesarean section. Mother refused cesarean section. What should the doctor do in this situation?
Answer: the physician should arrange a bedside conference with the mother along with other physicians, social worker, and administrator to discuss the matter.
Report Abuse


* Re:good for those whos exam is near
#1473520
rizowana - 09/21/08 10:01

AUTOSOMAL RECESSIVE DIS

Mnemonic: Police Commissioner of STATe Had Problem Getting PR(Permanent Residence) for Pigmented Hypothyroid Dwarf uncle Wilson

P-Phenyl Ketonuria

C-Cystic fibrosis

S-Sickle cell anemia
T-Tay Sachs( & all other lysosomal storage disorders except hunter™s & Fabry™s which r XLR)
A-Albinism
T-thalassemia

H-Hemochromatosis
Hepatolenticular degeneration
P-Pyruvate kinase deficiency
G-Galactosemia
-Glycogen storage disorders (All)
P-Polycyctic Kidney (Juvenile)
-Premature senility
R-Retinitis pigmentosa
Pigmented-Xeroderma pigmentosum
Hypthyroidism
Dwarf-Dwarfism
Uncle Wilson-Wilson™s disease

Most of the enzyme def. disorders are AR..except for few like G6PD def,Fabry's,Hunter's , which are XLR
Report Abuse

* Re:good for those whos exam is near
#1473536
rizowana - 09/21/08 10:26

#1356034
silva - 06/20/08 16:51

gojan audio
http://rapidshare.com/files/107293563/1-...njury1.m4a
http://rapidshare.com/files/107294661/1-...njury2.m4a
http://rapidshare.com/files/107295716/1-...ation1.m4a
http://rapidshare.com/files/107296980/1-...ation2.m4a
http://rapidshare.com/files/107298045/1-...emody1.m4a
http://rapidshare.com/files/107299089/1-...emody2.m4a
http://rapidshare.com/files/107300093/1-...emody3.m4a
http://rapidshare.com/files/107301220/2-...tion_1.m4a
http://rapidshare.com/files/107302151/2-...lasia1.m4a
http://rapidshare.com/files/107303101/2-...lasia2.m4a
http://rapidshare.com/files/107304261/2-...logy_1.m4a
http://rapidshare.com/files/107305332/2-...ology2.m4a
http://rapidshare.com/files/107306404/2-...ology3.m4a
http://rapidshare.com/files/107307394/2-...ology4.m4a
http://rapidshare.com/files/107308622/2-...logy_5.m4a
http://rapidshare.com/files/107309669/3-...ology6.m4a
http://rapidshare.com/files/107310682/3-...ology7.m4a
http://rapidshare.com/files/107311682/3-...ology8.m4a
http://rapidshare.com/files/107312848/3-...ular_1.m4a
http://rapidshare.com/files/107313932/3-...ular_2.m4a
http://rapidshare.com/files/107314879/3-...ular_3.m4a
http://rapidshare.com/files/107315885/3-...ular_4.m4a
http://rapidshare.com/files/107317184/3-...atory1.m4a
http://rapidshare.com/files/107318304/4-...atory2.m4a
http://rapidshare.com/files/107319491/4-...tory_3.m4a
http://rapidshare.com/files/107320581/4-...inal_1.m4a
http://rapidshare.com/files/107322237/4-...inal_2.m4a
http://rapidshare.com/files/107323808/4-...ncreas.m4a
http://rapidshare.com/files/107325299/4-...reas_2.m4a
http://rapidshare.com/files/107326853/4-7Renal_1.m4a
http://rapidshare.com/files/107328969/4-8Renal_2.m4a
http://rapidshare.com/files/107330324/5-1Gyn.m4a
http://rapidshare.com/files/107331492/5-2GYN.m4a
http://rapidshare.com/files/107333157/5-3Endocrine.m4a
http://rapidshare.com/files/107334567/5-...eletal.m4a
http://rapidshare.com/files/107335859/5-5Skin.m4a
http://rapidshare.com/files/107337110/5-...senses.m4a
Report Abuse


* Re:good for those whos exam is near
#1473551
rizowana - 09/21/08 10:41




#1403851
ultradocmansu - 07/31/08 04:49

hi rizowana.. i did nbme 5 online , but i dint do nbme 6 . for nbme 5 , i have list of saved answers by 2c2bd , and in first 2 blocks , i have some comments on answers posted by 2c2bd , (so for first 2 blocks , u have 2 opinions - mine and 2c2bd) and for other 2 blocks , they r jus 2c2bd answers alone.. if u guys wanna edit , edit it the way i did , for the benefit of everyone in the forum..

Still some doubt on answers..ask around if any doubts..2c2bd
Block 1
1 A
2D
3E
4A
5B
6A... i think eee
7B
8B
9A
10D, i think ee
11B
12B
13B
14B
15A
16A
17B
18E
19F
20C .. i think dd is the answer.(but jus for knowledge of related thing , flexor digitorum profundus inserts into proximal part of middle phalanx while flexor digitorum superficialis pierces through to insert into proximal part of distal phalanx.)abductor pollicis brevis (not mentioned here) and adductor pollicis insert into distal part of proximal phalanx.. there s no muscle as adductor pollicis brevis . its the abductor that has longus and brevis.
21B
22D
23B
24C
25A
26E
27A
28D
29E
30B
31A .. i think its bbb.. IgM binds RBCs , activates complement , membrane attack complexy -> intravascular hemolysis
32D
33E , i think f
34C
35D
36E
37B
38G
39A. i think c
40A , i think b
41E , i go for a
42A
43A
44C
45F , my answer is c
46B
47D
48D
49B
50C

Report Abuse

* Re:nbme 5 answers..
#1339271
2confused2bdoctor - 06/05/08 13:16

Block 2
1E
2D
3F
4B.. i have no idea
5D
6B
7D
8B.. i go for d
9B
10B
11E
12E
13A
14B
15B
16E
17F
18C , good question..
19C
20A
21D
22F. i think g.. i dont know if 'one month old boy' has any significance here. they wanna say he developed hydrocephalus later or something?
23F
24C , why not e?but c seems better anyway.. e has rearrangement of genome , genome , specifically means dna.. that is not rearranged anyway
25A
26E
27D
28C
29D
30E
31A
32A
33B
34E
35A
36B
37E
38E
39F
40E
41A
42D
43A
44E
45D
46E
47D
48F
49A
50E

Report Abuse

* Re:nbme 5 answers..
#1339273
2confused2bdoctor - 06/05/08 13:17

block 3
1C
2E
3D
4A..
5E
6A
7A
8D
9D
10C
11C
12D
13D
14B
15C
16A
17E
18C
19B
20C
21E
22E
23B
24E
25B
26C
27A
28C
29C
30B
31D
32D
33D
34E
35E
36C
37B
38C
39D
40E
41D
42B
43B
44E
45D
46H
47A
48D
49E
50C

block 4
1A
2C
3E
4C
5D
6F
7D
8B
9C
10B
11E
12E
13F
14E
15C
16B
17D
18A
19A
20D
21A
22D
23C
24C
25A
26D
27C
28F
29F
30A
31C
32C
33C
34C
35G
36B
37C
38E
39D
40A
41E
42D
43D
44D
45C
46D
47A
48D
49E
50C

Reply
#16
68. An infant, born at 30 weeks gestation, appears to be SGA (small for gestational age) with multiple malformations. Amniocentesis study was not performed. Infant needs resuscitation at birth. What should a physician do in this situation? (Important)
Answer: physician must resuscitate the patient in the delivery room because the diagnosis is uncertain.

69. A 60-year old man is diagnosed with terminally ill colon cancer and needs resuscitation. He did not sign a DNR order. The physician has decided to perform a ˜slow code™ on his own. Is this the right decision? (Important)
Answer: no. Please remember, a performance of ˜slow code™ or ˜show code™ is not acceptable to the patient. This decision by the doctor represents the failure to come to a timely and clear decision about the patient™s resuscitation status.

SLOW CODES REFER TO WHEN FAMILY SAYS TO DO CPR BUT PHYSICAIN THINKS ITS FUTILE AND JUST TO SHOW FAMILY DOES CPR WITHOUT FULL PROTOCOL.

70. A 20-year old man is diagnosed with suspected bacteremia and meningitis. He refuses antibiotic therapy. He collapses and requires resuscitation. What should a physician do in this situation?
Answer: the physician should resuscitate the patient despite the patient™s refusal to antibiotic therapy. THOUGH ANTIBIOTICS STILL NOT USED.

71. A 50-year old woman is diagnosed with severe aortic stenosis. She collapsed in a doctor™s office and is required resuscitation. She is waiting for valve replacement surgery. What should a physician so in this situation?
Answer: this condition is called ˜physiological futility™. In severe aortic stenosis, vigorous resuscitation is highly unlikely to restore adequate cardiac output. Therefore, the physician might reasonably refrain from resuscitation.
PHYSIOLOGICAL FUTILITY IS WHEN CHANCES OF SURVIVAL WITH CPR LESS THAN 1%.
72. A 14-year old boy is diagnosed with terminally ill cancer. He is not responding to chemotherapy. His parents want to continue the treatment. However, the boy does not want to continue his suffering. The physician told the parents that chemotherapy will not be helpful. What is the appropriate decision at this point? (Important)
Answer: the boy™s decision should be respected because the treatment is futile.
(WRONG PT MINOR PARENTS REQUEST SHUD BE UPHELD)GIVING CHEMOTHERAPY DOESNT PUT PT ON LIFE AND DEATH SITUATION
73. A surrogate pregnant mother made a surrogacy contract with a couple in which she will give the baby to the couple. She developed complications in the first trimester and wanted to abort. Is she allowed to do that? (Important)
Answer: yes. If her life or health becomes threatened from continuing the pregnancy, she should retain her right to abortion.

74. A physician became sexually involved with a current patient who initiated or consented to the contact. Is it ethical for a physician to become sexually involved?
Answer: no. Sexual involvement between physicians and former patients raises concern. The physician should discuss with a college or other professional before becoming sexually involved with a former patient. The physician should terminate the physician-patient relationship before initiating a romantic or sexual relationship with a patient.
IDEALLY ON STEPS SHUD BE HER DOC BUT SUPRESS UR OWN FEELINGS
75. A physician decided to take care of his own family members and relatives. He is a very smart physician. Is this a right decision?
Answer: no. The physician should encourage all friends and family members to have their own personal physician.

76. A male patient wants to have a copy of his medical records. What should a physician do in this situation?
Answer: the physician should retain the original of the chart. Information should only be released with the written permission of the patient or the patient™s legally authorized representative (e.g., attorney).

77. A 30-year old female wants to have an abortion. Her physician objects to abortion on moral, religious, or ethical grounds. What should a physician do in this situation? (Important)
Answer: physician should not offer advice to the patient.
IF SHE IS NOT COMFORTABLE DOING ABORTION ON MORAL GROUNDS REFER THE PT TO SOMEONE ELSE.

78. A physician sees patients at a reduced fee. He spends very little time with each patient. Is the physician doing the right thing?
Answer: no. The physician is not providing optimal care.

79. A surrogate pregnant mother signed a surrogacy contract with a couple. Male partner gave sperms which were artificially inseminated to the surrogate mother. Surrogate mother has a genetic relation to the child. She wants to void the contract after the baby is born. Is she allowed to breach the contract? (Important)
Answer: yes. Surrogate contracts, while permissible, should grant the birth mother the right to void the contract within a reasonable period of time after the birth of the child.

80. A surrogate pregnant mother signed a surrogacy contract with a couple. Both male and female parents gave sperm and ovums respectively. The surrogate mother wants to void the contract and she has no genetic relation. Is she allowed to breach the contract? (Important)
Answer: no. Genetic parents have exclusive custody and parental rights.

81. A surrogate pregnant mother signed a surrogacy contract with a couple. The couple got divorced. Male partner gave sperms and the female partner gave ovums. They do not want to continue the surrogate pregnancy. What should be the decision at this point? (Important)
Answer: the couple is genetically related to the fetus. They have the right not to continue with this pregnancy.

82. A surrogate pregnant mother signed a surrogacy contract with a couple. The couple got divorced. Male partner gave sperms but the female partner could not give ovums. They do not want to continue the surrogate pregnancy. What should be the decision at this point? (Important)
Answer: female partner has no right to terminate this pregnancy because she has no genetic relation. Surrogate mother has genetic relation and she has the right to continue this pregnancy even if the male partner disagrees.

83. A 3-year old girl is diagnosed with blood cancer. She has been waiting for an umbilical cord transfusion. Her mother delivered a newborn baby girl. Umbilical cord blood was obtained and was transfused to the 3-year old sibling. What is the duty of the physician?
Answer: physician should obtain an informed consent of the risks of donation and he or she should follow the normal umbilical cord clamping protocol. Physician should protect both the children.

84. A 31-year old man has decided to donate one of his kidneys for a large amount of money. Is this the right decision?
Answer: no. However, the donor can receive some payment to cover his medical expenses. Only the potential donor not the donor™s family or another third party may accept financial incentive. Payment should occur only after the organs have been retrieved and judge medically suitable for transplantation.

85. A couple has decided to have a child through artificial insemination. They asked the physician for sex selection of the child. What should a physician advise in this situation? (Important)
Answer: physician should not participate for sex selection for reasons of gender preference. However, sex selection of sperm for the purpose of avoiding a sex-linked inheritable disease is appropriate.

86. A 30-year old man has donated his sperms which were kept frozen. He died in a car accident. He did not leave any specific instructions regarding sperm donations. His wife wants to make use of them. A woman requested her to donate his sperms. What is the appropriate decision? (Important)
Answer: the donor™s wife can use the semen for artificial insemination but not to donate it to someone else. The donor should give clear instructions at the time of donation. The donor has the power to override any decision.

87. The donor and recipient of sperms are not married. Who would be considered the sole parent of the child? (Important)
Answer: the recipient. Except in cases where both donor and recipient agree to recognize a paternity right.

88. The residents and medical students were asked by an attending to follow certain orders for a patient. The residents and medical students believe the orders reflect serious errors in clinical or ethical judgment. What is the appropriate way to handle the situation? (Important)
Answer: The residents and medical students should not follow those orders. They should discuss with the attending issuing those orders. They should also discuss the situation with a senior attending physician, a chief of staff, or a chief resident.

89. A physician used a newly prescribed drug to his patient. The patient got sick after the drug was ingested and required hospitalization. Should the physician report this drug™s side effect to FDA (Food and Drug Administration)? (Important)
Answer: yes. FDA should be notified only if the drug causes serious adverse events such as those resulting in death, hospitalization, or medical or surgical intervention.

90. A 39-year old female has been suffering from chronic cholecystitis. The surgeon advised cholecystectomy. The patient wants a second opinion. The surgeon agreed. The patient went to another surgeon and has decided to be operated by the second surgeon. What should the second surgeon do in this situation? (Important)
Answer: the second surgeon should accept the patient because the patient has the right to choose the surgeon. First surgeon should accept the patient™s decision.

91. A 45-year old male was admitted to the hospital with mild chest pain. He wants to leave the hospital before completion of therapy. How do you manage the patient? (Important)
Answer: The patient is asked to sign a statement that he is leaving against medical advice (AMA). The patient may however leave without signing that statement. This document is a legal evidence that the patient was warned by the physician about the risk of leaving. Please remember, discharge AMA does not apply to children.

92. A 55-year old man requested his physician to misrepresent his medical condition to receive disability or insurance payment. What is the appropriate response of the physician in this situation?
Answer: The physician must refuse that request.

93. What is the responsibility of a fellow physician who is aware of drug abuse, alcohol abuse, or psychiatric illness of his colleagues or of a medical condition that is harmful to patients? (Very Important)
Answer: the physician should protect the patients. The fellow physician should report to the appropriate authority (i.e., report to the hospital authority; report to the Dean for a medical student™s problem).

94. A 60-year old male has been suffering from severe pain due to terminal prostate cancer. The patient is receiving lower doses of narcotics and sedatives. How can a physician relieve his suffering? (Important)
Answer: The physician should increase the dosage of narcotics and sedatives up to the maximum recommended amount. Listening, spending more time with him, and reducing psychological distress can reduce the suffering.

95. A 25-year old female medical student or resident noticed a mistake made by a junior attending physician during rounds. She is afraid of that attending physician. What is the appropriate way to handle the situation? (Important)
Answer: She should discuss the situation with a more senior attending physician for appropriate interpretation, advice, and assistance.

96. A 26-year old male medical student or resident made a mistake during patient care. He is afraid of what might result. What is the appropriate way to handle the situation?
Answer: He should disclose the mistake to the attending physician and try to learn from that mistake. The patient should be notified as well.

97. A 63-year old female health care worker is concerned about taking care of patients with HIV infection or multidrug-resistant tuberculosis. What is the appropriate way to handle the situation? (Important)
Answer: The physician should provide appropriate care to patients despite personal risk. Institutions should reduce the risk of infection by appropriate equipment, supervision, and training. Her concern should be taken seriously.

98. A 30-year old male physician has an opportunity for financial incentive if he sees more patients and refers them unnecessarily. What is you opinion about this?
Answer: The physician should provide only care that is in the patient™s best interest.

99. Two physicians are discussing a case inside the elevator of a hospital. What is your opinion about this?
Answer: They should not do that because they have to maintain the patient™s confidentiality.

100. The patient™s confidentiality should be maintained except in the following situations:
Physicians should override third parties in case of domestic violence, child abuse, elderly abuse, gunshot wounds, syphilis, and tuberculosis. They should report these cases to appropriate governmental authorities.

101. A physician is experiencing a very difficult ethical issue regarding a complicated case. He is confused. What should be the next step?
Answer: he should discuss the matter with other faculty members in his health care team, colleagues, or hospital ethics committee.

102. What is the final plan of action in an ethical issue?
Answer: Both patient and physician should agree regarding final management. The patient should be well informed about the medical condition. The physician should be sympathetic and knowledgeable regarding the relevant medical condition.

103. DNR (do not resuscitate) order. This is appropriate if the patient or surrogate signed that order or if CPR (cardiopulmonary resuscitation) would be futile. Physicians should write DNR orders and the reason for them in the chart. Please remember, œslow or œshow codes are not acceptable. Foods and fluids are considered therapies that should be stopped. (Important)

104. (A) Brain death (adult): (Important)
(i) Definition by the President™s Committee:
Death is an irreversible cessation of circulation and respiratory functions or irreversible cessation of all functions of the entire brain and brain stem.
(ii) The criteria of brain death by the staff of Massachusetts General Hospital and the Harvard Committee:
Death occurs when there is absence of all signs of receptivity, responsivity, and all brainstem reflexes, and the EEG is isoelectric. Sometimes metabolic disorders and intoxications may mimic the above findings.
(iii) The guidelines of brain death:
(a) The diagnosis should be made also by another physician and confirmed by clinical findings and EEG.
(b) The family should be notified. They should not make the decision about discontinuing medical treatment except in a situation where the patient has directed the family to make the decision.
© The physician should discuss with another physician before removing supportive measures (e.g., ventilators).
(d) Family members may request organ donation, and in many states physicians may request the family to make an organ donation.
(B) Brain death (children):
(i) Definition: same as in adults.
(ii) Criteria: similar in children and adults, but the period of observation is longer in children.
Children 1wk-2mo of age: two separate examinations 48 hours apart
Children 2mo-1yr of age: two separate examinations 24 hours apart
Children more than 1 yr of age: two separate examinations 12 hours apart
Spontaneous movements must be absent, with the exception of spinal cord reflex withdrawal and myoclonus.
Generalized flaccidity should be present. The presence of clinical criteria for 2 days in term and 3 days in preterm infants indicates brain death in majority of asphyxiated newborns. The absence of cerebral blood flow on radionuclide scan and silence of electrical activity on EEG are not always observed in brain-dead newborns. There is no universal consensus about the definition of neonatal brain death. The decision is made after discussion with the family and health care team. If there is difference of opinion, the ethics committee should be consulted. The decision is made on the basis of what is in the best interest of the infants and children.

105. Practice guidelines for physicians:
(i) The best way to practice medicine is to select useful diagnostic techniques and therapeutic measures which are most appropriate to a particular patient and clinical condition.
(ii) Practice guidelines can reduce the health care costs, which improve health care to patients who even do not have adequate health care benefits.
(iii) Please remember, guidelines do not and should not be the only way of managing an individual patient.

106. Some important points about patients:
(i) For a patient with an incurable disease, the major goal of therapy should be the enhancement of the quality of life.
(ii) The patient care begins with a personal relationship between the patient and the physician. If a patient has confidence on the physician, reassurance may be the best therapy. The patient must understand that the physician is giving the best possible care available.

107. Patients who do not have decision-making capacity about their medical care:
The patients who do not have decision-making capacity usually arrange for surrogates who make decisions for them. Their choices depend on their values. Psychiatrists are helpful in mentally impaired patients. Family members are usually the surrogates, because they know the patients very well.
Advanced directives: statements made in advance in case patients lose their decision-making capacity in the future. These directives indicate the names of surrogates and which interventions are acceptable or not acceptable to them. These are achieved by oral conversation (most common form), living will, health care power of attorney, or physicians can ask the patient in advance.
Absence of advance directives and surrogates: physicians can make the decision using all information and should respect the patient™s values. Physicians must know the laws of the state in which they practice.
Patient preferences are known:
The decision is made with the patient™s best interest in mind.
Disagreements between potential surrogates or between the physician and surrogate: Physicians can consult with the hospital ethics committee or with other physicians. The courts should be only the last resort.

108. Down syndrome with different medical conditions: (Very Important)
(a) Duodenal atresia at birth: surgical repair is recommended as it is done regularly.
(b) VSD (ventricular septal defect) in newborn period: initial conservative medicals management is followed by surgical repair as it is done regularly.
© Cyanotic heart disease at birth: immediate medical management, which is followed by surgical repair, as it is required routinely.
(d) Cosmetic surgical condition (e.g., rhinophyma or big nose): there is no urgency to repair the underlying condition, but it can be done as it is performed regularly.
(e) Neural tube defects (e.g., meningomyelocele): surgical repair is recommended as it is done regularly.
Please remember, a patient™s management should be discussed with his/her parents and the decision made with the best interest of the patient in mind.

109. A healthy male patient with Down Syndrome lives independently. He went to a doctor for facial cosmetic surgery. Can he make his own decision? (Important)
Answer: yes. The patient can make his decision if he understands the procedure and the consequences. He lives independently which indicates that he is capable of making his own decision.

110. A patient went to the doctor due to throat pain. The doctor asked the patient what her problem was. The patient said that she woke up at six o™clock in the morning, went to the bathroom, ate breakfast, and went to drop her children at school. She then came back home, stared cooking and continues to talk about irrelevant things. What should the doctor do to stop the patient from rambling? (Important)
Answer: the doctor should ask the patient to tell him what problems she has related only to her throat.

111. A patient went to a doctor for abdominal pain but remained quiet throughout the visit. He did not tell the doctor enough about his symptoms. What should the doctor do? (Important)
Answer: the doctor must ask the patient detailed questions about his abdominal pain. It is the doctor™s obligation to find out as much as he can about the patient. Without enough information, the doctor will not be able to make an accurate diagnosis.

112. A patient walked into his doctor™s office with acute abdominal pain. He has been suffering from ulcerative colitis. The patient is noncompliant and did not visit for the past six months. What should the doctor do in this situation? (Very Important)
Answer: The doctor should find out more about the patient™s abdominal pain before making any other decision. The doctor should always be responsible with the patients.

113. A terminally ill pancreatic cancer patient with multiple metastases is admitted to the hospital.
He is in critical condition. The patient wants to know his prognosis. What should the doctor say?
Answer: the doctor should tell the patient politely that he will discuss his condition with him and his family. The doctor should never specify the longevity of the patient.? The doctor should tell the truth even when the patient is a child. The doctor should not hide any medical information from the patient.

114. A patient is recently diagnosed with cancer. He is nervous but is eager to know about his medical diagnosis. What should the doctor™s reply be? (Important)
Answer: the doctor should gently tell the patient his condition.

115. A patient is recently diagnosed with cancer. Previously, he had an episode of a nervous breakdown after hearing a family death and had to be admitted to a hospital. He loves his family members and tends to be very open with them on all issues. How should the doctor tell the patient about his current state? (Important)
Answer: the doctor should call his family members and discuss the patient™s medical condition openly and politely.
(WRONG IF DISCLOSING BAD NEWS CLEARLY PUTS PT IN NERVOUS BREAKDOWN CONDITION WAIT AND GRADUALLY TELL THE PT.NEVER BREAK CONFIDENTIALETY)

116. A male patient was admitted with severe myocardial infarction. He was admitted to the ICU and his condition is very critical. He does not know the reason for his admission. The patient is unstable. What should the doctor tell the patient? (Important)
Answer: the doctor should wait until the patient is stabilized and then gently tell him his medical condition.
(HIDING SOMETHING FROM THE PT MAKES HIM MORE SUSPICIOUS AND MAY LEAD TO ALL KIND OF MISCONCEPTION IN PTS MIND SO NEVER WAIT TO TELL PT UNLESS TELLING IS CLEARLY DETREMENTAL)
117. A mother gave birth to a premature baby who was admitted to the NICU (neonatal intensive care unit). The baby is on a mechanical ventilator. The mother wants to hold the baby. What should the doctor do in this situation? (Important)
Answer: the mother should be allowed to hold the baby.

118. A male patient is recently diagnosed with HIV. Should the doctor ask about his sexual orientation (i.e., male, female, or both)? (Important)
Answer: yes, the doctor should ask the patient directly but politely about his sexual orientation.

119. A homosexual male patient went to a doctor. The patient™s partner was recently diagnosed with HIV. Should the doctor ask the patient whether his partner is penetrating him or he penetrates his partner?
Answer: yes, because the person who is being penetrated has a higher incidence of HIV due to trauma in perianal area.

120. A 6-year old boy comes to the ER after drowning. He expired in the ER despite appropriate resuscitations. The family members became angry which is a reflection of a sense if guilt and helplessness. What is the appropriate way of giving information to the family members?
Answer: the physician should give the information clearly and compassionately when there is no hope for survival. Parents need to know that everything was done to save the child.

121. A pregnant woman who is Rh (-) ve became sensitized. She had H/O induced abortions. Her husband is not aware of his wife™s previous abortions. He wants to know from the physician how she became sensitized. (Very Important)
Answer: the physician should tell the man to ask his wife. The physician should not mention anything about the patient™s H/O abortions.

122. A mother brought her infant to the ER. The radiologist test reveals old fractures of the ribs. She did not know anything about that. (Very Important)
Answer: this is a case of child abuse. This case should be reported to child welfare agency.

123. A physician is examining a child with respiratory distress. The child™s mother became anxious during the physical examination. Please remember, a patient™s management should be discussed with his/her parents and the decision made with the best interest of the patient in mind. (Important)
Answer: child abuse.

124. A mentally retarded patient became pregnant. The patient does not want an abortion. Her mother and husband want an abortion. What should a physician do in this situation? (Important)
Answer: abortion should not be performed.
BCUZ SHE IS COMPETENT UNLESS PROVED OTHERWISE
125. A male physician is examining an adolescent or adult female patient. What should a physician do in this situation? (Important)
Answer: a chaperone should be present during the physical examination. The same rules apply when a physician is examining a patient who appears to be seductive. (Important)

126. A female physician is examining an adolescent or adult male patient. What should a physician do in this situation? (Important)
Answer: a chaperone should not be present during the physical exam.

127. A suspected HIV patient expired in a car accident. He signed for organ donations. What
should a physician do in this situation?
Answer: his HIV status is not certain. The organs can be preserved until the HIV status is confirmed. If the test for HIV is positive, organs should be discarded.

128. A patient who expired in a car accident signed in his license for organ donations. His license has expired. He always wanted to donate his organs. What should a physician do in this situation? (Important)
Answer: physician cannot accept organs because the signed consent has expired.
ASK FAMILY MEMBERS ABOUT WISHES OF THE PT IF HE WANTED TO DONATE ORGANS.JUST AHVING AN ORGAN DONOR CARD DOESNT TELL PT INTENT.MAY BE HE SHUD HAVE DONE TO SHOW OFF OR GET A BONUS OR FORCRD BY SOMEONE.EVEN IF DONOR CARD HAD NOT EXPIRED AND FAMILY UNANAMOUSLY SAID THAT PT DIDNT WANTED TO DONATE ORGANS DONOT ACCEPT ORGANS FOR TRANSPLANT

129. A male physician sexually harassed a female patient during the physical examination. The patient complained to a nurse. What should the nurse do in this situation?
Answer: the nurse should tell the patient to make an official report to the hospital authority or to an appropriate agency.

130. A chronic male smoker comes to the physician for his heart problems. The physician wanted his patient to quit smoking. What should the physician advise in this situation?
Answer: the physician should ask the patient to quit smoking immediately because patients are usually more responsive when they are ill. The physician should assist the smoker to move one step closer to quitting.

131. A terminally ill patient did not sign a DNR (do not resuscitate) order, however, he signed a DNI (do not intubate) order. What should the physician do in this situation?
Answer: the physician should follow his orders i.e., the patient should be resuscitated but should not be intubated, despite severe hypoxic condition of the patient.

132. A terminally ill patient signed a DNR order, however, he did not sign a DNI (do not intubate) order. He wants to be intubated but not resuscitated. What should a physician do in this situation?
Answer: the physician should follow his orders i.e., the patient should be intubated but should not be resuscitated.

133. An adolescent car accident victim was brought to the ER in an unconscious state. The patient needs immediate surgical interventions. The surgeon was unable to contact any family member to obtain consent. What should a surgeon do in this situation? (Important)
Answer: the surgeon should do the procedure without waiting to obtain consent for the benefit of the patient.

134. An obgyn doctor is recently diagnosed with HIV infection. He is receiving medication for HIV. His physical and mental conditions are normal. Should he tell his patients or fellow physicians about his HIV status?
Answer: no, however, the doctor should take appropriate precautions for infection control. He does not have to tell his fellow physicians about his HIV status including the physicians who are referring patients to him. The doctor is allowed to see patients if he takes appropriate precautions. However, he should notify the hospital authority.

135. A physician is scared of seeing an HIV patient with an open wound. Can a physician refuse to see a patient?
Answer: yes, however, a physician™s refusal to see a patient is unethical but is legal.

136. An elderly semi comatose patient may require surgical intervention. His family members are confused about the surgery. They asked the surgeon for his opinion. What should the surgeon™s response be?
Answer: the surgeon can give his opinion and act as a moral surrogate for the benefit of the patient. (Very Important)
137. A 12-year old boy is diagnosed with a terminal illness (e.g., malignancy). He asked the doctor about his prognosis. His parents requested the doctor not to tell him the bad news. What should the doctor do in this situation? (Very Important)
Answer: the doctor should tell the truth politely and compassionately to the patient.
YES TOTALYY CORRECT CHILD NEEDS TO BE BE TOLD OF HIS TERMINAL ILLNESS IF HE CAN UNDERSTAND THE SITUATION AND IRREVERSIBILTY OF DEATH.TRY TO CONVINCE PARENTS FIRST.

138. A 55-year old woman is recently diagnosed with right breast cancer. The doctor told the patient that she would require surgery for removal of the right breast. She started to cry. What should a doctor do in this situation?
Answer: first, the doctor should give her some tissue paper for wiping her tears. Then, the doctor should be sympathetic to her and console her. He might tell her that similar reactions are usually expected from other patients with breast cancer. Please do not mention that she will be fine with a breast implant or without a right breast because she is already 55-years old.

139. A mother is carrying a 500 gram premature fetus, which develops acute fetal distress. The physician wanted to perform a cesarean section. Mother refused cesarean section. What should the doctor do in this situation?
Answer: the physician should arrange a bedside conference with the mother along with other physicians, social worker, and administrator to discuss the matter.
Report Abuse


* Re:good for those whos exam is near
#1473520
rizowana - 09/21/08 10:01

AUTOSOMAL RECESSIVE DIS

Mnemonic: Police Commissioner of STATe Had Problem Getting PR(Permanent Residence) for Pigmented Hypothyroid Dwarf uncle Wilson

P-Phenyl Ketonuria

C-Cystic fibrosis

S-Sickle cell anemia
T-Tay Sachs( & all other lysosomal storage disorders except hunter™s & Fabry™s which r XLR)
A-Albinism
T-thalassemia

H-Hemochromatosis
Hepatolenticular degeneration
P-Pyruvate kinase deficiency
G-Galactosemia
-Glycogen storage disorders (All)
P-Polycyctic Kidney (Juvenile)
-Premature senility
R-Retinitis pigmentosa
Pigmented-Xeroderma pigmentosum
Hypthyroidism
Dwarf-Dwarfism
Uncle Wilson-Wilson™s disease

Most of the enzyme def. disorders are AR..except for few like G6PD def,Fabry's,Hunter's , which are XLR
Report Abuse

* Re:good for those whos exam is near
#1473536
rizowana - 09/21/08 10:26

#1356034
silva - 06/20/08 16:51

gojan audio
http://rapidshare.com/files/107293563/1-...njury1.m4a
http://rapidshare.com/files/107294661/1-...njury2.m4a
http://rapidshare.com/files/107295716/1-...ation1.m4a
http://rapidshare.com/files/107296980/1-...ation2.m4a
http://rapidshare.com/files/107298045/1-...emody1.m4a
http://rapidshare.com/files/107299089/1-...emody2.m4a
http://rapidshare.com/files/107300093/1-...emody3.m4a
http://rapidshare.com/files/107301220/2-...tion_1.m4a
http://rapidshare.com/files/107302151/2-...lasia1.m4a
http://rapidshare.com/files/107303101/2-...lasia2.m4a
http://rapidshare.com/files/107304261/2-...logy_1.m4a
http://rapidshare.com/files/107305332/2-...ology2.m4a
http://rapidshare.com/files/107306404/2-...ology3.m4a
http://rapidshare.com/files/107307394/2-...ology4.m4a
http://rapidshare.com/files/107308622/2-...logy_5.m4a
http://rapidshare.com/files/107309669/3-...ology6.m4a
http://rapidshare.com/files/107310682/3-...ology7.m4a
http://rapidshare.com/files/107311682/3-...ology8.m4a
http://rapidshare.com/files/107312848/3-...ular_1.m4a
http://rapidshare.com/files/107313932/3-...ular_2.m4a
http://rapidshare.com/files/107314879/3-...ular_3.m4a
http://rapidshare.com/files/107315885/3-...ular_4.m4a
http://rapidshare.com/files/107317184/3-...atory1.m4a
http://rapidshare.com/files/107318304/4-...atory2.m4a
http://rapidshare.com/files/107319491/4-...tory_3.m4a
http://rapidshare.com/files/107320581/4-...inal_1.m4a
http://rapidshare.com/files/107322237/4-...inal_2.m4a
http://rapidshare.com/files/107323808/4-...ncreas.m4a
http://rapidshare.com/files/107325299/4-...reas_2.m4a
http://rapidshare.com/files/107326853/4-7Renal_1.m4a
http://rapidshare.com/files/107328969/4-8Renal_2.m4a
http://rapidshare.com/files/107330324/5-1Gyn.m4a
http://rapidshare.com/files/107331492/5-2GYN.m4a
http://rapidshare.com/files/107333157/5-3Endocrine.m4a
http://rapidshare.com/files/107334567/5-...eletal.m4a
http://rapidshare.com/files/107335859/5-5Skin.m4a
http://rapidshare.com/files/107337110/5-...senses.m4a
Report Abuse


* Re:good for those whos exam is near
#1473551
rizowana - 09/21/08 10:41




#1403851
ultradocmansu - 07/31/08 04:49

hi rizowana.. i did nbme 5 online , but i dint do nbme 6 . for nbme 5 , i have list of saved answers by 2c2bd , and in first 2 blocks , i have some comments on answers posted by 2c2bd , (so for first 2 blocks , u have 2 opinions - mine and 2c2bd) and for other 2 blocks , they r jus 2c2bd answers alone.. if u guys wanna edit , edit it the way i did , for the benefit of everyone in the forum..

Still some doubt on answers..ask around if any doubts..2c2bd
Block 1
1 A
2D
3E
4A
5B
6A... i think eee
7B
8B
9A
10D, i think ee
11B
12B
13B
14B
15A
16A
17B
18E
19F
20C .. i think dd is the answer.(but jus for knowledge of related thing , flexor digitorum profundus inserts into proximal part of middle phalanx while flexor digitorum superficialis pierces through to insert into proximal part of distal phalanx.)abductor pollicis brevis (not mentioned here) and adductor pollicis insert into distal part of proximal phalanx.. there s no muscle as adductor pollicis brevis . its the abductor that has longus and brevis.
21B
22D
23B
24C
25A
26E
27A
28D
29E
30B
31A .. i think its bbb.. IgM binds RBCs , activates complement , membrane attack complexy -> intravascular hemolysis
32D
33E , i think f
34C
35D
36E
37B
38G
39A. i think c
40A , i think b
41E , i go for a
42A
43A
44C
45F , my answer is c
46B
47D
48D
49B
50C

Report Abuse

* Re:nbme 5 answers..
#1339271
2confused2bdoctor - 06/05/08 13:16

Block 2
1E
2D
3F
4B.. i have no idea
5D
6B
7D
8B.. i go for d
9B
10B
11E
12E
13A
14B
15B
16E
17F
18C , good question..
19C
20A
21D
22F. i think g.. i dont know if 'one month old boy' has any significance here. they wanna say he developed hydrocephalus later or something?
23F
24C , why not e?but c seems better anyway.. e has rearrangement of genome , genome , specifically means dna.. that is not rearranged anyway
25A
26E
27D
28C
29D
30E
31A
32A
33B
34E
35A
36B
37E
38E
39F
40E
41A
42D
43A
44E
45D
46E
47D
48F
49A
50E

Report Abuse

* Re:nbme 5 answers..
#1339273
2confused2bdoctor - 06/05/08 13:17

block 3
1C
2E
3D
4A..
5E
6A
7A
8D
9D
10C
11C
12D
13D
14B
15C
16A
17E
18C
19B
20C
21E
22E
23B
24E
25B
26C
27A
28C
29C
30B
31D
32D
33D
34E
35E
36C
37B
38C
39D
40E
41D
42B
43B
44E
45D
46H
47A
48D
49E
50C

block 4
1A
2C
3E
4C
5D
6F
7D
8B
9C
10B
11E
12E
13F
14E
15C
16B
17D
18A
19A
20D
21A
22D
23C
24C
25A
26D
27C
28F
29F
30A
31C
32C
33C
34C
35G
36B
37C
38E
39D
40A
41E
42D
43D
44D
45C
46D
47A
48D
49E
50C

Reply
#17
rosacea
acne
seborrheic dermamatitis
seborrheic keratitis
porphyria cutanea tarda

Corticosteroid induced mental changes
--Dementia Vs normal age related changes
--Calculate negative predictive
--Interpret Odd ratio
--Investigate primary ameno
--Investigate ambiguos genitalia
--Erosive esophagitis - Mx
--Sjogren syn
--Scleroderma - which ANA is raised
--Serum sickness
--Mx of intermittent claudication
--Mx of pressure sores
--Mx of status epilepticus
--Mx of Alzheimer
--Diagnose Parkinson

Malignant hyperthermia, Malignant neuroleptic syn, heat stroke, heat exhaustion - how each presents

--Pathophysiology of all neurological diseases eg GBS, syringomyelia, myasthenia

--How to investigate hypertension

--Diffuse eso spasm - Tx

--Lots of cardio - MI, unstable angina, stable angina - ASD, VCD, TGA, Coart - arrhy and Mx

--Schizophrenia, brief psychotic disorder, acute stress disorder, PTSD, Adjustment disorder, delusional disorder

--Substance abuse - cocaine, cannabis, opoid, marijuana

--Trauma - aortic transection


first of all exam is completely based on concepts
exam is tougher than nbme 3
usmle world gives u orientation
i dint get any video clips
there were 4 ecgs
dermatology
dx of rosacea
dx of seborrheic keratitis
acne prevention - options were avoiding fatty foods oily based creams washing with soap water
psychiatry
many qs on dementia almost every type of dementia
delirium
alcoholo withdrawl treatment
panic disorder rx
panic disorder with agoraphobia rx
and dx
dysthymia
mdd rx
generalised anxiety disorder rx
depression due to medical condition hypothyroidism
immunology
brutons
degoeorge
scid
aids
haematology
dd of lymphadenitis
fe def folate def
mgus
multiple meloma
ttp
hus
cardilogy
ekgs on acute pericarditis svt afib mi
rx of svt
dx res cardiomyopathy
dx of ar
rx of as
resp
asthma mx
excercise induced asthma rx
volumes in asthma
ards
copd developing into corpulmonale
asbestos worker non smoker cancer bronchogenic or mesothelioma
musculosketletal
myasthenia pathophysiology
polymyalgia rhematica with gaint cell arteritis developing blindness
sjogren syndrome dx
tension headache cluter headache dx
multiple sclerosis dx
gynecolgy qs were eassy
very few paeds and surgery qs
i will post some more topics as i remember
i dint do well in the exam waiting for results

1) Ascities tap finding : hemooragic/atypical cells/ ----?D ------OP Malignancy , TB
2) AScities findings: wbc> 750 ------?d ---------sbp
3)32 y m h/o whip lash car accident injury 2 year age came with b/l wasting of small muscles of hands and senory changes ------?d---------op: Amotrophic lat sc/syringomylia/ MS/cervical spondylosis/disc prolapse
4)HIV pt female cd4
Report Abuse


* Re:good for those whos exam is near
#1473665
raju2008 - 09/21/08 12:04

We know ANGEL-MAN Syndrome aGenomic Imprint.But the gene expression you are
showing is true angel charecteristics.I wish my daughter fatimah could be like you.
May ALLAH help you.
Report Abuse

* Re:good for those whos exam is near
#1473673
raju2008 - 09/21/08 12:07

I wanted to mention Angel woman syndrome instead angel man.
Report Abuse


* Re:good for those whos exam is near
#1473694
rizowana - 09/21/08 12:19

remembered BS

a daughter comes with her elderly mother, she does all the talking and mother can hardly say three words before she takes overbest thing for the physician to do is to ask the daughter to leave. (b) parents bring 15 year old boy..they privately ask the physian to tell him how bad smoking is and ask him to talk the boyphysians best response to that. © 14 year old girl tells physian that she is under lot of pressure from her friends to have sex.what should the physians do next talk to her about STD and contraceptive. (d) concern mother brings daughtershe looks expressionless and talks little..denies that she is depressedwhat else the physician should inquire for or what waning signthere were more can™t remember now.
5.There was one case series question, one calculation for Relative risk, graph saying if moving upper limit affect sensitivity and specificityhow does it effect PPV and NPV, one confidence interval question, lot of questions on how to advice patients.either by quiting smoking, or by exerciseblah blah depending on senerio.I just hope I choose the right option.
6.One questions was a woman wants to donate $1 million to some charity after her husband dies of heart attack to lower heart disease problem among her communitywhat will give her the fastest result in 2 yrsBest option there I thought was to donate to a arobic exercise program..free for all.not sure!!
7.Ego defense question: Policeman works 24hrs shiftfor how longcan™t recalland then when a co-worker get sick he offers to do her duty as wellwhat defense mechanismthere wasn™t much else in the historybut only two best option was undoing and acting out..so if you ask me I got a lot of questions from Bev science.there were more like this.
8.one drug abuse questionmy only saver was papillary constrictionopoid or heroin?? Patient did have track marks


stats.... most of it was simple calculations but i didnot get few of them. mostly sensitivity, specificity, PPV, NPV. confidence intervals, type of study.
i

*
2 cohort study questions
a confounding bias question
sampling bias question
sensitivity question
attributable risk question
Type 1 error question
Child abuse question
A WHOLE BUNCH OF WHAT WOULD YOU DO IN THIS SITUATION QUESTIONS
variable ratio question
BEHAVIORAL SCIENCE : In every block i got appr.3-4 Q on ethics.Even had few Q from stats.but not too much calculations..sensitivity and PPV,NPV etc.had 2Q from defense mechanisms and sleep disturbances also.



BEHAVIORAL SCIENCE

Lots of computations on odds ratio, relative risk, Hardy-Weinberg, values for sensitivity and specificity of two studies and you will be asked to compare. May questions also about biases. The study will be described and you have to determine what is the bias of study. Questions on alpha and beta errors. Same thing as before, the study will be described and you'll be asked what error is it. Questions on what is the best thing to say to a patient given a particular situation. For this you have to study ethics. Kaplan is OK



exam..Very simple biostats-case control,corelations,just 2 calculations on PPV,sensittivity.


Q on gene population- a=0.6 b=0.4 what is the % of heterozygotesà 2pq=48%

Q on a karyotype (picture#7) shows the chromosome 21 + various description of the disease- what is the diagnosis? à Down syndrome

Q on a woman comes to the doc, she tells the doc that she is motivated w/ her job, but she doesn™t socialize w/ other people @ her work, she feels uneasy w/ them. What™s the Dx? à Avoidant Personality

Q on a woman had several episodes of Depression in the past, for which she has used medication. She has discontinued her medication for the past six months and didn™t have any episodes of depression. She is thinking of becoming pregnant. What should you as a doc tell her?

Q on a pt. is in the hospital. The prognosis of his disease is vague. Although he never designated any one as power of attorney or written consent but his wife tells you that he had mentioned on several occasions not to resuscitate his life if in case he is at the verge of dying. What should you as a doc do, if this pt. is suddenly about to die? àListen to the wife and do not resuscitate his life

x
Q asks about two groups of females, one smoker and the other non-smoker, are identified and followed for ten years to see the risk of developing breast CA. What type of study is it? à Cohort (Prospective)

Q to Calculate the power when Type I (Alpha) error=0.5, Type II (Beta) error=0.20

Q on a clinical trial is being conducted to see the risk of developing cancer in children who live in households near chemical factory. Will ask about whom should the control group be? àchildren living w/ their parents near the factory who do not develop the cancer

Q on a pt. presents w/ the severe chest pain. How should you (doc) initiate the patient™s history so that can learn the most about the Pt™s problem... in other words, what should you (doc) ask the pt. to learn most about the pt.™s problem? à œTell me more about your pain Ans


Q describes a person is to have a surgery done and he was told about the operation, side effects and other t
hings... signature of patient is required so how should this contract be worded?- here you can have different ways to write the consent, you would easily pick it up”its all common sense.
Q on a woman who has an argument with her boss, she comes home and yell at her children àdisplacement

Q on odds ratio. .formula- always always there get it straight you won™t miss it if you know the formula.

Q on a study is being done about the effects of contraceptive on reduction on breast cancer.. .what is the prerequisite à inclusion of group patients who shouldn™t have family history of breast cancer.

Q on a high school football player who smokes come to the doctor . . . what should doc say to make him quitting smoking? àœsmoking can and could affect your sports activity

Q about a mom brings in 12 year old son who has type 1 diabetes and he doesn™t comply with the medications mother keeps complaining to the doc and doesn™t let the kid talk- what should the doc say.... à œmam, can you please let me hear his side of the story

Q on prevalence....a specialty clinic has a 12% prevalence of a disease but when the docs go to a community clinic it decrease to 2% y?. . . àbecause specialty clinic only sees particular disease

Q on a case scenario presents a guy has a problem but there are no physical findings. à conversion disorder

narcissistic personality

absence seizure “ Rx ethosuccamide

erection problem in pt, how to Rx

function of diff parts of brain (Fist aid)


Antipsychotic causes EPS; how to treat it- antimuscarinic- Benztropine

S/S mania -what drug to treat”lithium

M/A of TCA drugs

gingival hypertrophy”phenytoin

What other drug act on beta- lactamases


Odd ratio~ used is with case control studies

Relative used with cohort study

Null hypothesis, x 35. How do u calculate sensitivity?
>36. How do u calculate positive predictive value?
>37. How do u calculate attributable risk?
>38. What is a cohort study?
>39. What is a double blind study?
>40. How do u calculate prevalance?
>41. In what situation is prevalance greater than incidence?
>Chronic diseases
x>43. Lineweaver Burk plot asking what happens with competitive
>inhibitor?
>44. Michaelis Menton curve asking what happens with a competitive
>inbibitor?
>45. What kind of mutation happens when there is change from long

1. How old is child when they don't have stranger anxiety?
>52. Characteristics of person suffering from bipolar disorder?
>53. Characteristics of person suffering form schizophrenia?
>54. Parent bring kid in who was unconcious, he has nystagmus and
>has been withdrawn from parents, what drug has he been abusing?

70. Person comes in with a magazine ad or a new drug which you as a
>doctor know won't help them in their condition and patient asks if
>you can give them the drug, what is first thing you tell the
>patient?
>71. 13 yr old kid comes in who admits to being sexually abused by
>stepfather, what is the first step you take? Notify child services
>or call police or talk to mom or talk to step father
>

83. know your vaccines, and active and passive immunizations
>84. cohort study, case series
>85. I had 5 q's on epidem. (ppv, spec, sens)
>86. p=id
Delirium (patient not oriented as compared to dementia in which patient is oriented)
Conversion disorder
Post traumatic stress disorder
Personality disorders: Avoidant and Schizotypal
Narcolepsy
Borderline patient using splitting as a defence mechanism (either everything is good or everything is bad)
Lots of questions on shizophrenia (at least 4)
Thought disorder, Loose associations, etc.


BEHAVIORAL
-lots of "how would you respond as the physician" questions, WITH lengthy clinical descriptions. read the actual question first, it will allow you to skip a lot of the data they give
-graph asking how you would maximize sensitivity
-odds ratio
-types of studies (cohort, case-control)
-speech development (what the hell?)
-developmental milestones
-narcolepsy
-sleep apena (central vs. obstructive) X2

Behavioral.. I had the smoking and the yellow teeth! I think that™s usmles favorite questions************************
Behavioral science: the statistic and epidemiology wasn't that bad, PPV, NPV, Types of studies, odds ratio, etc
They asked about one Down's Synd. patient 26, wants to get married with a 40 yo woman, lives alone and work , his aprents doesm't want him to get married, what the dr. should do, I don't know I chose let him married, I don't know. teh patien cdr relationship qs where difficult.


BEHAVIOUR
1.CHILD DEVELOPMENT
2.PERSONALITY DISORDERS
3.DEFENSE MECHANISMS
4.BEHAVIOUR PHARMA---V IMP
5.ETHICS RULES---
4.CLUSTER DESIGNATION

BIOSTATS
ITS THE SUBJECT I HATED MOST
BE SURE ABT SENS.SPEC,+VE AND -VE PREDICT VALUE



behvioural
factitious disorder by proxy-- 2q almost similar
developmental milestone in a 7 month old
separation anxiety disorder
SSRIs-- increased latency in ejaculation


¢
@BIOSTAT.:

*Calculation of RR(relative risk)-very easy.
*90%CI=0.9-2.1,asked its interpretation about that study.
*Cohort study
*p=0.01”its significance-described one study of dietary fiber and colon CA.
*Effect of one one new drug curing that particular disease-on incidence & prevalence.

¢
A large group of people in some study on risk factors are followed over 20 years; this type of study is known as what? Ans: cohort study.
¢ Know how to tell difference between screening and diagnostic test
¢ Diagnostic test is done in a sick population while screening is healthy
Know how to calculate PPV, NPV, Relative Risk, Sensitivity, Specificity...etc.
¢ Don't just memorize it as a / (a+b) as they teach in books because on the boards
they may switch the table so that instead of "Disease" on top, they will have "Test
Results" instead....this will throw off your entire calculation.
A boy gets a trauma and can no longer look down and medially; what nerve was injured?
Know the definition of a type I or alpha error
An elderly pt has disseminated prostatic CA and family says that
it will only hurt him more if you tell him the diagnosis, because there is no treatment available. What should the dr. do? Ans: Should tell the pt anyway b/c Dr cannot be asked to w/hold info from pt. Femoral nerve lies outside the femoral sheath
has tissue sample of Meckel's diverticulum - what are the two types of epithelia the pathologist will see is? Ans: Pancreatic and Gastric
Elderly female has lost spouse. Feels guilty, has shock, often feels like husband is still in room, has somatic symptoms, but no suicidal ideations. Has been occurring for
Report Abuse


* Re:good for those whos exam is near
#1473714
rizowana - 09/21/08 12:30

remembered biochem and xenetics


9.There were few mol biosome weird and some I can™t remember the exact wording.some went over my headso guessed. Enhancer question.
10.Competitive inhibition question, oncogene, one questions clearly asking about what happens in a mice if dynein was mutatedoption I choose that made sense was movement of ovum from F tube to uterus.but I tell you they made the wording so complicated to confuse you.
11.Some channel related questions.
12.3-4 second messenger questionswording it defferent ways
116.PKU, Maple syrup disease ( what enzyme), vegan mother .kid with presentation. ( sorry can™t recall the detail).urine shows methamalonic acidvoilaB12 deficiency, Alcoholic with thiamine deficiency, Few questions on different biochemical pathwaywith glucagon and Insulin effect.most questions were presented in kids.I thought I was giving the exam on Padiatrics17.Many questions basically will give you all clues about the expected disease they even give the dX.and ask you a completely different thing.
18.Glucose-6 phosphatse deficiency, Carnitine deficiency, MCAD deficiency
19.Lesch_Nyhan syndromewhat enzyme, Tay-Sachs disease

22.I had quite a few genetic question with pedigree asking what type of inheritance, or where it is masaicism, Pleiotropy , incomplete penetrance or variable expression.
23.One hardy weiberg calculation
2t, turner syndrome (2-3 questions), Fragile X (triple repeat), Trisomy 21 pt when dies at 45 age.what do you expect to find in autopsy, DeGeorge syndrome, Vitamin A def, Vit c defcollagen problem, I think I mention before B12 def, Rat poisoing what to give ptonly vit K wasn™t an optionbut there was fresh frozen plasma,
syndrome, muscle spindle question, Neural tube defectpatient saying her niece has NTD so what should the physician say or do nextwasn,t sure whether to do AFT or to say it™s not a hereditary disease (I choose the second..comment??), MS, ALS, MLF syndrome, Neurotransmitter decrease in Huntington ..only they didn™t mentio in was Huntingtonyou have to figure that outI choose GABA, SuccinylCholinsaying aneasthetic drug causing flaccid paralysis..what drug?

rRNA is the most abundant question
Exons are expressed question
where on tRNA does the anticodon bind
Restriction enzyme question
a Lac Operon question
Competitive inhibitor question
2 Cell Cycle questions
Golgi is where O-Oligosaccarides are added
microtubule question
Collagen is seen in granulation tissue question
immunohistochemical stain of cytokeratin question
Glycolysis is metabolized in the cytoplasm
NADPH oxidase def. question
citrate regulates glycolysis
rate limiting enzyme in heme synthesis
what enzyme is in gluconeogenesis question
2 G6PD questions
Lactase def. question
cystathione cynthase def question
maple syrup urine disease question
SCID question
where is insulin uptake question
Pompes dx. question
Ketone bodies in DKA
Medium Chain Acetyl CoA question
2 LDL questions
how does lead poisoning affect heme
CO2 transport in blood
Imprinting question
Anticipation question
Hardy Weinberg question
Neurofibromatosis question
2 cystic fibrosis questions (1. what transporter, and what dx can you get)
myotonic dystrophy is trinucleotide repeat
LHON is mitochondrial
Vitamin B6 question
Vitamin B12 question
monopygotic twin question
)r
...........
out of glycoge storage...by far mostly asked is Mcardle(somebody exhausts after physical work,recall mus.phosphorylase) lyso.stor-gaucher/tay(*)/neimanpick. ********************************************************************

****************************************************************************

what is the main difference between DNA and cDNA?????


but there was this question on measles and RFLP segments 5 different types.............. and they asked wich one belong to the virus I had NO IDEA on this one so, I read it twice guessed and moved along ???????


what is the mechanism of action of 9-(2-hydroxyethoxymethyl)guanine????

ok so the question said which of the following is a intercalated sequence of pro and Gly of course it had other things but that is what it wanted.

1.-CCTGGGCCCGGACUUGGTCCGGGG
2.-CCGGGTCCAGGCCCAGGACTGGGG
3.-AGTCATGGTCCATTGAAAACAATTGT
4.-CCTGGGCCCGGACCAGGTCCGGGG
5.-TCTTCUGAAGAGAGCATTTTTTGCGA
what is the mech of action of Sparfloxacin???

a) inhibit protein synthesis
b) inhibit cell wall synthesis
c) inhibit DNA polimerase V
d) inhibit DNA Gyrase
e) Competes with para-aminobenzoic acid





****************************************************


biochem...my weak area was thankfully easy. mostly rate limiting enzymes , insulin- glucagon world, enzyme defeciencies.
molecular biology ws the toughest part for me. there were qs i had no clue. i cant even remember the qs properly now. some were ok too like anticipation, hardy weinberg calculation, and simple punnet square calculations.
t Q
BIOCHEM : Had few Q from pathways...but they were pretty straight forward...no need to memorize all big pathways..got many Q from abnormal diseases like PKU,maple syrup urine dz.ochronosis etc
Got 3 pedigree chart from genetics,1 Q regarding pleiotropy and genetic imprinting
2Q regarding hardy-weindberg equation...but i was not able to figure out ans.of one of them..was little tricky and time was running...so couldn't get the correct ans.i guess
2
1. Drawing of urea cycle. Point to area where NH4 enters the cycle.
2. What causes RBCs to form aggregates in Sickle Cell Disease?
3. A gene product is thought to produce tRNA. What characteristic tells you that it is tRNA?
1. presence of many modified bases
2. 7 methyl guanine cap
3. polyA tail

4. Drawing of glycolytic, gluconeogenic, glycogen synthesis pathways. Point to the part of the drawing which is influenced by insulin.
5. G proteins and second messengers
6. Nitric oxide works thru which second messenger?
- cGMP
7. How would you distinguish the gene product translated from cDNA from that translated from ordinary DNA?

Some questions on glycogen storage diseases, lysosomal diseases. They were pretty straightforward. Study the pathways well.

Post-translational modifications, like capping and N-linked phosphorylation
Study also about lac operon, transcription, translation.


**********************************************************************************

Q on a pedigree à all offspring of affected females show signs à A classic MITOCHODRIAL inheritance (picture#2)

Q on why anticipation occurs? à More trinuleotide repeats.

Q always describes a child with malnutrition à which AA deficient? à look for essential ones (remember PriVaTe TIM HALL in First Aid page 166)

Q must know about mech of action of Nitric Oxide à Nitric oxide stimulates soluble guanylate cyclase (sGC) to convert GTP to the intracellular second messenger cGMP.

Q always arises on how Insulin acts on GLUT4 à Translocates to membrane.

Qs on vitamin B12 à they always ask many questions here à the most popular one is its relationship to intrinsic factor and pernicious anemiaà know everything about B12.

.

Q on DNA replication à What DNA Topoisomerase does? à Makes a nick in the Helix to remove supercoils.

Q on the Lac Operon- what happens in the absence of lactose à the repressor protein encoded by the I gene binds to the lac operator and prevents transcription.

Q about Lesch Nyhan Syndrome- What Accumulatesà uric acid.


Q on where RNA is made? à Nucleolus.

Q on how mutation in splicing affects protein synthesis.

Q about protein folding à is not random.

Q about the affect of no hydroxylation of proline à what happens? à The triple Helix can™t be formed.

Q Describes Scurvy à What Vitamin Def. à Vitamin C.

Q Describes the regulation of eukaryotic gene à what is location of the enhancer related to the exon?

Q about what occurs many times in a protein life span? à Can be phosphorylated many times.

Q describes a fat soluble vitamin à cranial nerve problem à which and why?à Vitamin A causes bone deposition at the skill foramina which causes CN compression.also know that vitamin A is very toxic if taken orally.

Q on Hypoglycemia à Factitious Insulin induced à look for serum C-peptide à C-Peptide only made by endogenous Insulin.

Q on Melatonin à relationship to which AA à Tryptophan.

Q related to PKUà SHOULD be screened AT BIRTH.

Q About pseudohurler à defect in n-acetylglucosamine causes what? à failure to add a marker to manose phosphate (lysosomal problem)

Q must know relationship between Insulin and glucagons in the fasting and fed states- Know everything they always ask questions here.

Q on carnitine def. à what happens? à carnitine deficiency impairs the entry of long-chain fatty acids into the mitochondrial matrixà production of ketone bodies impaired

Q usually describes Popes disease à which molecule abnormal? à Glycogen. (defect in lysosomal alpha-1,4-glucosidase)

Q always asked about Glucose 6-phosphatse à what happens in fasting? à Increased.

Q asks about a child S. Aureus bacterial infection and describes a phagocyte problemà NADPH oxidase def.

Q about which enzyme in bacteria help it survive in lysosomesà Catalase.

Q describes hypercholesterolemia à where is the defect? à LDL receptor

Q describes the initial lesion in atherosclerosis à fatty streaks.

Q drug of choice for increased VLDLà Niacin.

Q about a homeless man in a poisoning case that caused metabolic acidosisà what is the poison? àethylene glycol
.
Q about potassium levels in DKA à low intracellular and high extracellular.

Q Describes maple syrup disease- avoid which AA? à Branched chain Ile Leu Valine

Reply
#18
Q describes Cystathione synthase deficiency à which AA accumulates? àmethionine


)
Q on Oocyte before ovulation and ask about the egg arrested at what stage? à Prophase


x
Q Describes a woman who overcooks her food, what s vitamin def. will she present with? à Folate (folic acid is heat labile)

Q about Vit C and ask it is a cofactor in the conversion of what? à Dopamine to NE.

Q describes a patient with vit B12 and ask what test used to detect def.? à Schilling test (In the exam the name might not be there but the procedure will be)


Q describes a patient has gastrectomy, later anemia and neurological problems à B 12 deficiency




Q on a pt who has renal failure, what other deficiency will you see? Inability to form active Vit. D



Q on a Karyotype of Trisomy and will ask what would you see? àFlat facies and simian crease

Q will give a Description of Turner™s and will ask what another finding is associated w/it? à Infertility


Q on an alcoholic pt. WI easily bruisibility. ecchymoses. etc. What™s the defect? à Defective Hydroxyproline (require Vit. C)

Q
Q ASKS what vitamin could be supplemented to prevent neural tube defects. à Folic Acid

Q describes of Lesh Nyhan was given and asked to identify the purines? Guanine & Hypoxanthine

Q about PKU- a girl w/ blond hair, musty odor etc. What will you see? àPhenylactate

QQ asks about aecond messenger of Nitroglycerin? à CGMP (FA 324)


Q about the Defect in CGD? à NADPH oxidase

Q about the Defect-in Chediak Higashi? à Microtubule and lysosomal defect


Q on a pt. who has mild fasting hypoglycemia (might not be severe) and enlarged heart.. .what™s the defect? àDebranching enzyme def. (Glycogen Storage Disease)

Q on a pt is heterozygote for Familial Hypercholesteremia. What™s the defect? apo B-100 receptors (are aka LDL receptor)


Q on a Hematology slide (picture#39) of megaloblstic anemia (hypersegment PMN™s might not be there ). . Ask for the mech or what is associated w/? à Decrease conversion of N-methyl tetrahydrofolate to Tetrahydro folic acid
x


Q on baby girl with MR and inappropriate laughter à you have to have to ID imprinting

Q describes Huntingtonsà you have to ID trinuclcotide base pair repeats.

Q on which amino acid is ketogenic à mot likely lysine

Q on a boy with violent behavior coarse facial features who bites off his own fingers à ID the missing enzyme in a drawing of the Uric Acid pathway.

Q about a boy who uses Gowers sign to stand who has adipocytes in his calf muscle biopsy à X-linked inheritence

Q on diagnosis of Fabrys diseaseà ID an accumulation of ceramide trihexose.

Qss on G-protein-linked second messengers, must know which ones release calcium, inhibit and stimulate cAMP-à know the pathways for all the G proteins---Always asked in the exam “ALWAYS.

Q activators of the P450 system and one question on inhibition of P450. FA 336 DRUG reaction table is a MUST KNOW.

Q on the activator of lipogenesis à citrate

Q on the rate limiting step of the TCA cycleàisocitrate

Q on the metabolism of methanol à ID formic acid via alcohol and acetaldehyde dehydrogenase

Q on collagen synthesis and which process occurs immediately after the procollegen leave the ER àID peptidases cleave terminal sequences

Q to ID which blood cell lineage requires Vit Eà Vit E

Q on Huntington™s disease (anticipation), what is responsible for Anticipation? à Trinucleotide repeat expansion





Q on Anemia, neurologic abnormality, gave Folate which treated anemia but not nuero abnormality àShould be treated with both Folate & B12

Q on mosaicism in autosomal cells

Q on PKU, no symptoms when in hospital, when home symptoms appear, reason àhome food contained Phenylalanine

Q Describes of galactosema, cataract, reason for It à osmotic damage by formation galactilol


DNA formation from RNA, by a retrovirus, in which direction does it make DNA


Main RNA formation occurs when- histology slide point to nucleolus

If a pro, is reformed & structure is changed from a helix- to b-plated sheath, what bonds are reformed “hydrogen bond

I-cell dz, que was what is the problem”mannose phosphorylation lysosmal enz is deficient

Glucagon acts by increasing cAMP, which then acts on what? CREB- Lemcine Zippen

As in pathology. determining risk of a child to get dz, when one or both parents are having AD dz

If transgenic mice don™t have normal gene, which was present before, what this study conferes?

how proto-oncoggene gets converted to oncogene, --translocation

Tumor & deletion /mutation of P53 supressor gene?

Which AA is used in biosynthetic pathways?

Km values of glucokinase & hexokinase, Km value of glucokinase is higher etc.

competitive inhibitors, effect on Km & Vmax

Pertusis toxin & action thru Gi protein, on increasing cAMP

If absorption prob occurs due to bile def, what of the following gets depleted-look for fat soluble vits

Relation of HMP shunt, NADPH, & glutathione reductase

B12 def & methylmalonyl

Pt of diarrhea comes to ER. Only dextrose solution is given, what will be found blood of pt in high conc

Allopurinol & action in gout by decreasing uric acid formation



n

Valine and it is avoided in which disease? Maple Syrup patient

Which enzyme m the kidney breaks down ammonia?

Mechanism of AMP?

Where does citrate acts on the fatty acid synthesis?

Cell cycle phases, know phases?

A man with blindness with skin deformities and a headache, what is the? Retinol deficiency

Know about vitamin K


Marfan™s mode of inheritence

pedigree - mitochondrial inheritence,

Glucose-6-phosphatace deficiency

Rh (”) mom gives birth to Rh(+) baby- what do u do so that mom™s baby won™t get jacked? Rogham or its action will be listed

know your sedative antihistamines

know which specific hormones have intracellular receptors
What muscles used for the valsalva maneuver?




Given length of cDNA and original DNA. Difference Is due to what? (Introns.)

Match the given length of nucleotides. E.g., ACGA in DNA matches to UGCU in RNA, remember polarity.

Polyadenylation occurs on which end?

snURPs fxn where?

Hardy Weinberg genetics law. Given the frequency of recessive gene, calc of heterozygous.

Most important fxn of HMP in RBC™s? (keep glutathione reduced)

Basement membrane fxn is dependant on what? Between collagen IV and proteoglycan

Glycine is the most conserved a.a. because?

arachidonic acid pathway.

curve; efficacy vs potency

Comparing mm Tx effect vs mm toxic effect of two drugs.

Enzyme kinetics. E.g., choose most likely effect of noncompetitive inhibitor

During metaphase, ER and golgi are not visible because...

PKU-affected patient must avoid Nutrasweet



.

Know your DNA replication and RNA transcription

Know RNA subtypes very well; mRNA splicing and processing sequences; recognize tRNA sites

Avoid answers that tell U to refer the patients to other physicians or other hospital units

Know Genetic terms by definition (IMPRINTING, ANTICIPATION, REASORTMENT, MIXING)

Know everything about G6PD deficiency and its links to hemolytic anemia

Lysosomal storage diseases (mainly Fabry™s, Tay-Sachs, and (Gaucher™s)

Cystic Fibrosis --- always asked

Duchenne™s muscular dystrophy --- always asked (know presentation, X-linked recessive)

the role of BRCA gene in DNA repair

Location of RNA splicing? Nucleus

tRNA structure with labels, which site amino acids bind?

Protein (chaperons) function? Assist protein folding in ER before going to Golgi

Defects in what part of the lac operon results in functional irregularity?

Association of X inactivation (transcriptionally inactivated gene, Barr body) with IMPRINTING

G6PD deficiency question and association with hemolytic anemia

Case of Fragile X syndrome by history, likely explanation? Expansion Trinucleotide Repeat

Case of Menkes disease by history, asked cause? Defect in the lysyl oxidase gene

Which enzyme is reduced in function after a meal? Glycogen phosphorylase

Which enzyme is inactive in their dephosphorylated state? Fructose bisphosphate phosphatase-2 (remember there are 2 more; Glycogen phosphorylase and Hormone sensitive Iipase)

A diagram of Citric acid cycle with labels, name the labeled

A diagram of Urea cycle with labels- name the labled

Heme synthesis cycle- know the rate limiting step
Report Abuse


* Re:good for those whos exam is near
#1473765
rizowana - 09/21/08 13:04





Heme synthesis cycle- know the rate limiting step

Cholesterol synthesis - know the rate limiting step

Enzyme kinetic graph with increased Vmax. Likely cause? Overexpression of of Kinase gene


Vitamin.A deficiency (skin problems and dry eyes and blurred vision at night)

Thiamine deficiency in HMP shunt (causes reduction in RBC transketolase activity)



*************************************************************************************

1. Biochemsitry: not many, but know ALL the enzyme-related dz, DNA/ RNA stx and enz. working mechanism, enzyme-kinetics.

7.
1

************************************************************************

, one q on HOX genes--had no idea,

3-4 questions on identification of inheritance patterns-one was mitochondrial pattern













***************************************************************************************************

o16.PKU, Maple syrup disease ( what enzyme), vegan mother .kid with presentation. ( sorry can™t recall the detail).urine shows methamalonic acidvoilaB12 deficiency, Alcoholic with thiamine deficiency, Few questions on different biochemical pathwaywith glucagon and Insulin effect.18.Glucose-6 phosphatse deficiency, Carnitine deficiency, MCAD deficiency
19.Lesch_Nyhan syndromewhat enzyme, Tay-Sachs disease
20.I had atleast 3-4 questions on different dyslipidemias
21.How Hydroxyurea is used to make HbFto help in sickle cell disease.2


Biochemistry/genetics:
1: HGPRT deficiency with typical case scenario
2: HGPRT defieicency, which substance accumulated .. GMP
3: Glucose to gluose 6 phophate, reaction requiring .. ATP to ADP
4: Diabetic mother, child born with Hypoglycemia, which pathway affected .. confusing answers, i checked Hepatic gluconeogenesis ?
5: Child born with biliary atresia n severe jaundice, which vitamin deficeint after 1 month .. all were water soluble vit in answers, no fat soluble mentioned .. folic acid, vit B6, Vit B12, vit C .. i checked folic acid ??
6: Vit C deficiency
7: Ch lead poisoning with histo slide, asked which diagnostic test confirmatory .. raised delta aminolevulenic acid
8: Glucose 6 phosphatase deficiency with confusing scenario mixing with Galactose metablism disorder ?
9: Vit D deficiency linked to chronic renal failure
10: Few questions ab chances of homozygous/ heterozygous with diease identificaiton in stem
11: Rickets presentation



Z**************************************************************************************

* Pic ofDNA replication, where is helicase? A.. at the fork unwinding. * Few cases of modes of inheritance.
*.
* Case of pt taking anti-depressive eats cheese and wine having headacheA.. MOA inhibitors + tyramineA.. hypertensive crisis.
* Case of G6PD deficiency A.. decrease in NADPH leading to hemolytic anema. * Case ofMcArdled£TMs dis. What is deficient? Glycolosis or Gluconeogenisis?
.
A~ Case of PKU
A ~ Case of Alkaptonuria ..dark urine.
A~ Case of Lesch-Nyhan syndrome ..hyperurisemia, def. In HGPRT. ~
A ~r.
A ~ Case ofDown Synd.A.. meiotic DISJUNCTION ofHOMOLOGOUS chromosomes. A ~ Case of DuchennedTMs Ms
Dystorphy A ..defect in dystrophin gene which results in hypertrophy in calf muscle, replaces with fat. Dx with biopsy. A ~

* ~ choice. -,
* Mech. Of action of AllopurinolA.. inhibits xanthine oxidase. * Side effect ofPTUA.. agranulocytosis.
* One half life problem. J * Pt has anemia after taking anti malarial drug, what dei Does pt have?A.. G6PD dei
* One case with cardiac cycle.
* Case of new born with bleedingA..def. Vit K.
* Case of child with Necator americanus infection and anemia why? A.. dei of iron.
* Case ofD. latum infection and anemia why?A.. BI2 dei that leads to megalogblastic anemia.
14.
19. vitamin b12 def 20. thiamine def in alcoholics 21.

Biochem

frameshift mutation
topoisomerase (what is their function)
promoter sequesnces
intron vs, exon
3' polyadenylation
tRNA charging
Northern Blot
several pedigrees
several Hardy Weinberg questions
Pompe's Disease
Hunter's syndrome
Ehlers-Danlos syndrome
Osteogenesis Imperfecta (picture of kid with blue sclerae)
carnitine shuttle for fatty acids
many, many questions on G-protein linked 2nd messengers (make sure you know all the ANS receptor types and what type of 2nd messenger it is linked to---it was SO MONEY to know that)
many insulin/glucagon questions
several questions on arachidonic acid pathway
kwashiokor vs. marasmus
enzyme kinetics
CO2 transport in blood (which way does the Hb curve shift)
which vitamins are fat soluble
vitamin A deficiency (can cause alopecia)
>g
>>47. Where is mRNA made in the cell?
>48.>?
>>69. Know steps involved in doing a PCR.
>

>1
>1
>16. mitochondrial inheritance -2 q's
>17. >18. pku
>19>
.
Hardy Weinberg population genetics calculations
Glucose-6-phosphate dehydrogenase deficiency and reaction to sulfa drugs
Which a.a. become essential in a patient with PKU with phenylalanine restricted diet:
Answer: tyrosine
PPatient presents with muscle weakness and hypertrophy of calf muscles, which chromosome is defective?
Answer: X chromosome (DMD)
Uncoupling agents and on which part of mitochondria they work?
Anwer: dissipate proton gradient across the inner mitochondrial membrane
Urea cycle
Arachidonic Acid products (very heavily tested, at least 5 questions)
Collogen Synthesis and structure
Vitamins (heavily tested):

B1 (alcoholics)
B2 (cofactor in oxidation and reduction-FADH2)
B6 (given along with isonizid to prevent peripheral neuropathy)
Vitamin C (hydroxylation of proline and lysine in collogen)
Vitamin D (which organ hydroxylates 25 hydroxy D3 at alpha1 position: kidney)
2 questions on Huntington Disease:
trinuclear repeat pattern
point to Caudate Nuclei on MRI



BIOCHEM
-DNA mutations
-where in cell introns are removed
-tRNA attachemnt to ribosomes
-modes of inheritence (I actually had one ask me the chance of a MAN passing an X-linked recessive gene to a son, read CAREFULLY)
-urea cycle
-carnitine
-glucokinase vs. hexokinase kinetics
-enzyme kinetics
-prednisone

biochem...my weak area was thankfully easy. mostly rate limiting enzymes , insulin- glucagon world, enzyme defeciencies.
molecular biology ws the toughest part for me. there were qs i had no clue. i cant even remember the qs properly now. some were ok too like anticipation, hardy weinberg calculation, and simple punnet square calculations.


*******************************

knock out mouse with defect in ubiqutin gene...result-->
knock out mouse with defect in special integrin gene, what causes in lymphocyte action--->

yup, ubiquitin ar sevral classes of protein like e3 ligase. it causes polyuiquitination. proteasome will come and degrade the protein that are ubiquitinated. most imp example is nf-kb. this is th most imp transcription factor in immunity.in the absence of ubiquitin, u cant activate nf -kb as there will not be any degradation of ikb associated with nf-kb, which are inhibitory to nf-kb. so u mus remember that apart from decreased protein turnover, loss of ubiquitin will affect inflammatory response in cells as ubiquitin is must for nf-kb activation.
**********************************
.

****************************************
was one question on HOX on asso of HOX with uterine anomalies, s.. MCAD deficiency, McArdles, Achondroplasia and Huntingtons and Neurofibromatosis were all there

Biochemistry: a lot of qs about transcription, say what kind o mutations, a lot related with alcohol, tehy were ones really odds, vitamins,
Genetics:a lot of qs with q2, what is the amount of carriers in a population.
****************************************

¢
BIOCHEM N GENETICS
1.AS GOLJAN UNCLE SAYS ALL PATHWAY REGULATING ENZYMES WTH THEIR RESPECTIVE DEF DISEASES--V IMP
2.GLYCOGEN STORAGE DISEASES
3.AMINO ACIDS N FATS RELATED TO VARIOUS DISEASES
4.JUST TRY TO TAKE "THE BIG PICTURE"FRST N THEN U WILL BE ABLE TO INTEGRATE
5.GENETICS BE SURE ABT THE GENETIC DIAGNOSIS
METHODS N THEIR PARTICULAR DIFFERENCES

bichemistry

nieman picks
carnitine deificiency
pyruvate kinse def
g6pd def -- 4/5 q--caused by drugs -- primaquine, sulfonamides
vit b12-- 4/5 q
thiamine-- alcoholic
folate-- pregnancy 3q

mol bio
a-t c-g bonds
dna gyrase
tata box
7 methyl guanosine cap
restriction endonuclease-- 4q
2 transgenic mice exp -- answerable
unhybridised loops on mRNA
dna recombinant technology

@BEH.SCI.:

*phobia case-in patient with artificial pacemaker.
*factitious disorder-path.lab worker with hematuria.
*one girl stopped going to school after an vehicular accident.
*Informed concent-mentally retarded patient for hernia repair.
*old patient-came with 2 daufgters-how will you startetc.
*Teen comes with STD,asks not to tell her parents
*female worker with nuchal rigidity-onlt at work,no pain at weekends-what will you suggest?stetching exercise/ask more about her posture/work depression.

*MCAD deficiency-dicarboxylic aciduria
*Maple syrup urine disease-valine was in choices.
*Ac.porphyria-with alcohol
*patient with malabsorption-fat soluble vit.supplementation is needed.
*MOA of brown fat in newborn-uncoupling agent
*pilot on 3days fasting-action of glucagon
*arginino succinate in urea cycle
*patient on pure veg.diet-what deficiency?..CHO/protein/fat/minerals
*deficiency of tyrosine-effect on which neurotransmitter?Ach/NE/GABA/dopamine

@MOL.BIO.&GENETICS:

*2 pedigree qs
*photo of a girl with blue eyes-defect in?..precollagen-1
*why some diseases are common in A.jews?-vague options.
* DNA synthesis in nucleus &RNA synthesis in cytoplasm-how is it possible?
Nucleus/nuclear pores/chromatin.
*2qs on southern blot,northern blot &other immunoassays.
*mutation at TATA box
*given numbers to codons-asked how many codons are translated?you have to identify stop codons and answer accordingly.
*some one liner qs on transcription/translation
*mutation in glucuronyl transferase enz.in a case of Gilbert`s syndrome.
*adopted kid-wilson`s disease-keyser fleisher ring
*I-cell disease
*anticipation
*no Hardy-wienberg!!!!!!!!


¢ Aldose reductase: converts galactose to galactitol. What is harmful effect of this?
¢ Ans: Galactitol can cause cataracts.
¢ Pt's description classic for diarrhea, dementia, dermatitis - I think the answer is Hartnup's which can cause the pellagra
¢ Adenosine deaminase defic (ADA defic) can cause what? Ans: SCID
Some bacteria make Vit. K, thus if you give an ABX, would expect to see Vit. K levels to decrease. DZZ'A-r rich DNA has low .tpeWng~polnt than_ rich DNA due to different numbers of hydrogen bonds formed between the bases~~
KNOW WELL: Potymerase chain reaction- why you do It, how you do it, materials needed, etc.(4 questions at least)
Know electrophoresis- what it is, why you do it. how to read a drawing of one (3 questions)
Know restriction endonudeases -what they are, why you use them, palindrome sequences are usually cut (be able to recognize a palindrome)
Transition, transverslon, deletion, frame-shift mutation, nonsense mutation, missense mutation, silent mutation, Introns, exons, cap and tall on mRNA (what are they and when are they put on)
Remeber thymine in DNA, uradi in RNA
Understand the E.cdi In opernn (operon, promoter. repressor, co-repressor, Inducer, etc.) Understand transcription and translation VERY WELL (primers, sigma factor. 3as and 50s
ribosomes, translocation, DNA and RNA polymerises, elongation, TATA box) both in eukaryotes and prokaryotes
WA light damages DNA by indudng formation of thymine dimers (dinical--xemderma pigmentosa)
Upid storage disease/mucopolysaccharidoseslgiycagen storage diseases- thXKNOW WELL¢ Receptor signal transdudion. In other words, second messengers for each receptor subtype (CAMP, IP3&DAG, ion gated channels, etc.)
)
Biochemistry:
1) Alkaptonuria, ?parents asked u about long term prognosis: ?arthritis*, ?renal calculi
2) Pathogenesis of CGD of childhood
3) Menkes disease, MOA
4) In which part of the cell removal of the introns from mRNA take place?: _nu_c_le_i*?, RER? Golgi? Etc
5) CF, MOA receptor, CF transmembrane conductance regulator gene ---+ defect in CI-channels* (CF is HY, many qs)
6) SCD (but not in q stem), they said disease caused by substitution val for glu, question about which type of bond of bond is affected ....
7) Mother Hx of myelomeningocoele, son with MMC, which vitamin decreases risk for this abnormality? Folate*
8) Hypercholesterolemia, ?pathogenesis: ans description of LDL receptor (Receptors and second messengers r extremely HYI)
9) tRNA wobble diagram, asked to show site of different functions (2 qs .. similar q in
retired or self test)
10) Fetal alcohol sy: 2 qs
11) Tx child with hyperammonemia, etc
12) ?Reason some pts get lactic acidosis post alcohol ingestion .... know everything u can
about OH
13) McArdle's described, ? enzyme: muscle phosphorilase
14) Von Gierke's*, description disease
15) Drawing IG, where does the Ag bind?
16) Multiple skin ca (xeroderma pigmentosum), ?what's wrong; DNA repair defect*
17) Mother has 2 y old son wi MPS wi decr iduronidase and wiN chromosomes, now she is pregnant again, amniocentesis is done, demonstrated decr iduronidase, what else is necessary to be done: ?check for MPS in amniotic fluid*, ?chromosomal analysis, ?fetal biopsy of the liver, ?mothers liver biopsy
18) Prader Willi sy, paternal imprinting, microdeletion 15q*
-WHICH ANTINEOPLASTIC DRUG REACTS WITH DNA DIRECTLY A)MTX B)CYCLOSPORINE

3-5-FU D)VINCA ALKALOIDS

4-. USE OF GABAPENTIN

5-an anti tb drug hepatotoxic,hyperurecemia acting against only intracellular bact..pyrazinamide

6- Lots of MOA, few side efeects

7-HIV pt on drug present w/anemia, what cause it

8-How Hydroxyurea is used to make HbFto help in sickle cell disease

9-Rat poisoing what to give ptonly vit K wasn™t an optionbut there was fresh frozen plasma,

10-What to give in CMV infection if resistant to ganciclovir.foscarnet, One question on HIV medsHAART regiminforgot what it was.

11- Legionella what drug to use, Staph Aureus what drug to use, N Gonnorhea what drug to use,

12- Drug for helminth, S/E of malaria drug

13-.Question on CVS, ANS drugswith graphsI made my best guessnot sure if I did right or wrong, Renal drugs, M/O of phenytoin and carbimazapine, Haloperdol, M/O buripion ( FA says still unknownI had to guess), S/E of TCA, Some CTY P450 related questions, 2-3 picture asking where the drug works

14-Aspirinblockinglowers what, TXA2

15-??), Neurotransmitter decrease in Huntington ..only they didn™t mentio in was Huntingtonyou have to figure that outI choose GABA, SuccinylCholinsaying aneasthetic drug causing flaccid paralysis..what drug?

16-Pt taking already two drug for hypertension and saying you add 3rd drug to control pressure but causes hyperkalamia.what drug..ACE inhibitor

17-Sidenfil and nitroprissidewhat happens, Birth control pill associated break through bleeding in contex of CTY P450 drugs.

18-pharma.... easy.mostly MOA and resistance. antiviral and chemotherapy drugr tested heavily

19-know minoxidil also give a B-blocker with it
Zero order elimination question
bethanachol question
physostigmine question
2 alpha-1 blocker questions
what drug pheochromocytoma
Erythromyocin cyp inhibitor


20-Na channel blocker that increases QT

2 warfarin questions
Thrombolytic question
Methotrexate and leucovorin
Bleomycin question
Vincristine and neurotoxicity


21-.they love to ask MOA of warfarin-Rx for AF,bleeding diathesis c cimitidine(gyn.mastia) monitor PT,preg CI(*) 2,7,9,10/anti vitK.heparin-safe in preg/antithrombin3/???INR, stk/tpr plasmin-plasminogen etc etc.BPH/ca.prost.......

22-pharmacology
1 doxorubicin : intercalatred dna or break strand DNA?
2 mech of colchicine :inhibit microtubule
3 rupture appendicites : atb cefotaxime+genta or pipercilin/tazobactam?
4 s/e of dramamine : urinary tention
5 old people need to use digoxin need to check liver or renal function or both
6 orlistat SE steatorrhea

23-A father brings his child to the ER, he was fixing the sink that is usually locked cozz they keep different poisons there but he got careless and his kid drank one of the poisons. they have fumigators, sprays and rat poisons there, what is the teatment for this pt. I forgot the patient is bleeding from the gums.

a) deferoxamide
b) Atropine
c) fresh frozen plasma
d) EDTA
e) Naloxone

24-what is the mech of action of Sparfloxacin???

a) inhibit protein synthesis
b) inhibit cell wall synthesis
c) inhibit DNA polimerase V
d) inhibit DNA Gyrase
e) Competes with para-aminobenzoic acid

25-PHARMAC : Many many Q from this ...but they were easy and simple..need to memorise all tiny tiny side effects for some Q.Had 2 graphs from pharmaco dynamics regarding potency and efficacy and also from ANS with drug X and norepinephrine etc

26-For example, how does tetracycline work? Ans: by inhibiting attachment of tRNA to the ribosome. Something like that. Study also the AIDS drugs. As usual, what action of drug at the molecular level.

27-Drug for methotrexate
1. What drug is usually given as pre-treatment for leukemia chemotherapy?
- allopurinol
2. Drawing of nephron. Where does this diuretic act?
3. Which of the following anti-diuretics works by inhibiting the Na/K/2Cl transporter?
- furosemide



28-1. 25 y.o male who went hiking in the woods 2 wks ago. He was treated with a penicillin drug for throat infection. PE shows linear erythematous rashes over arms, legs, hands. What is the cause?
- photosensitivity
- Varicella-Zoster
- Hypersensitivity
(I answered photosensitivity

29- Px with greenish ear discharge, with a fruity smell. What is the antibiotic of choice?


30-focus on antimicrobials with emphasis on pneumonias,some complicated drug interactions,MOA of neoplastic{favourite was doxorubicin and vincristine

31-on acyclovir- they want you to know that for it to function it needs àthymidine kinase (phosphorylates DNA polymerase

32-Q on how NSAIDS affect the kidney in general à INHIBITS prostaglandins. What does that mean? Prostaglandins œDilate the afferent arteriole and if inhibited GFR will dec- watch if for patients with renal failure.


33-NE release from adrenal gland medulla is mediated by? à Ach

34-Q on site of action of drugs tricyclic antidepressantsà inhibits reuptake of NE


35-Q describes Reye™s syndrome and ask what is the mech of action of the drug that causes it à Irreversibly inhibits cyclooxygenase(COX-I and COX-2) that is Aspirin


36-Q on what is the advantage of COX-II inhibitors à It only plays a role in inflammation ONLY and does not give any GI side effects (GI lining which is protected by COX-I inhibited by Aspirin

37-Q on which drug to add in a patient on NSAIDs and GI upset à add misoprostol (Note that misoprostol should NOT be given to pregnant women because it is abortifacient

38-Q you must know the Mech of action of streptokinaseà conversion of plasminogen to plasmin to aid in thrombolysis.

39-Q about the efficacy and potency curves for drugsknow the diagrams in FA p299 p300. This is ALWAYS in the exam.


40-Q about the most common side effect of Calcium channel blockersà Flushing


41-Q about the substitute to ACEI in case of its coughing side effect à LOSARTAN (AgII receptor antagonist)

Q about the drug (s) that cause SLE like symptoms à Procainamide

Q about the P450 inhibitorsà Cimetidine is almost always in the answer choices

Q about a drug that blocks gastric acid secretions even in the presence of Ach à omeprazole

Q on accidental injection of NE into tissues what to give to prevent necrosis à Alpha blocker

Q on patient who has mania and seizures whch drug to give to treat both à valproate

Q case decribes hemorrhagic cystitis what drug causes it à cyclophasphamide

Q on what is used to treat the hemorrhagic cystitis of cyclophasphamide à mesna¢


42-Q on a pt. is using 3 or 4 different drugs names listed) and present w/ some symptoms. Which drug can be substituted to prevent the cough? Losartan


43-Q shows a pic where drug X is stimulating muscular contractions while Drug Y is inhibiting muscular contractions. ID? à Most of the time they have Atropine and Succinylcholine as X and Y.


44-
Q on the MOA of Ketoconazole? à Inhibit ergosterol synthesis

Q on the MOA of tetracyclines? à Prevents the attachment of amino-acyl tRNA to the ribosome

Q on a Pt. who has Herpes, what™s the tx? à Acyclovir

Q on Parathion (organophosphate) poisoning along w/ it™s symptoms how ou treat it? àAtropine

Q on a Pt w/ Wilson™s disease. Tx? à Penicillamine

Q describes a Pt who has a prolactin secreting tumor.Tx? à us Dopanine Agonist

Q asks about aecond messenger of Nitroglycerin? à CGMP (FA 324)

Q describes a Mother who is pregnant, what anticoagulant should be used? à Heparin

Q on the MOA of 5-FU? à Bioactivation of FdUMP which leads to decrease dTMP

Q Tell you about hemorrhagic cystitis caused by cyclophosphamide and asked the MOA? àAlkylating agent which covalently cross link DNA at guanine N-7.

Q asks why penicillin not effective against Mycoplasma. Has cholesterol in its bacterial membraneNOTE - œno cell wall might not be an answer choice.


45-Q ask about 5-fu mode of action àinhibit thymidylate synthase.

Q to trt Teenager with acne àtx is tetracycline

Q on which hormone has affect on renal vasculature àdopamine

Q where you must know the mech of action of Sertralineà inhibit the neuronal reuptake of serotonin

Q on how you treat influnza A àneuranunidase..


46-on a patient taking an antiarhythmic has pulmonary fibrosis. àamiodarone.

47-Q on a guy who just turned positive for a TB testà Give INH with Vit. B6.


48-Q on Allopurinol MOA....diagram will be given. Locate where it acts àxanthine oxidase inhibitor


49-Q on colchicines MOA à inhibit microtublules

50- about a kid who has asthma, what should be given during an acute episodeàalbuterol

51-on a women who was put on warfarin- what allergy medicine should be avoidedàcimetidine


52- describes a woman who has an oral candidiasis... what drug should be used. ànystatin


53-describes a patient on ahypertensive medication and now he is allergic to sulfa- what to give? àEthacrynic acid

54-Q on which Beta-blocker not to give an asthmatic à propranolol

55-Questions Prophylaxis for accidental healthcare worker exposure to TB (Rif for a month)


56-on which antibiotic would you not prescribe an eight year old. à Tetracycline


57- activators of the P450 system and one question on inhibition of P450. FA 336 DRUG reaction table is a MUST KNOW.

58-Qss always asked in the exam arediuretics “ know the MOA and Site of action of Acetozolamide, MOA and site of action of Furosamide


59-Q on Which diuretic will you prescribe to a diabetic with a GFR of

Q on a schizophrenic patient who has diabetes develops facial and tongue movements, what is the mechanism of action of the medication that you will prescribe next? àID clozapine MOA


60- Describes of galactosema, cataract, reason for It à osmotic damage by formation galactilol

61- on antiCA -Bloomycin- it causes à interstitial pulmonary fibrosis

Q ask If tumor produced in rats, what is deleted àtumor superssor gene, p53 etc.


62-Drug induced interstitial nephritis- methicillin

63-breast tumor Esgn Rc positive, how to treat- tamoxifen


64-Hyperprolactinemia assoc. with Dpamine


65-absence seizure “ Rx ethosuccamide

erection problem in pt, how to Rx


66-Benzo antagonism”flumazenil

67-Pt after operation not recovering from relaxation, gave neostygmine, condition didn™t improve, which was relaxant- succinylycholine

Antipsychotic causes EPS; how to treat it- antimuscarinic- Benztropine

S/S mania -what drug to treat”lithium

M/A of TCA drugs

gingival hypertrophy”phenytoin

What other drug act on beta- lactamases

Pencicilin elimination blocked where by probeneied- renal tubular secretion,

toxoplasma- treatment trimethoprim”sulfo

M/R to INH”by change in catalase

Rx of herpes zoster-acyclovir

Pulmonary fibrosis as an ADR-bleomycin

Pt on Rx hodkin™s dz, ADR, describing CHF, drug was doxorubicin

Breast Ca, Rc +ve,-- Rx with tamioxifen

Drug acting on IL-2 , INF, blocking their synthesis

Reply
#19
68-Drug given in peptic ulcer, coates mucosa”sucralfate
M/A of omeprazol- inhibit H+/K+ ATPhase

Hx of bipolar, which is the DOC? Lithium

69-A man with an absence seizure, what drug you use?


70-Penicillin hypersensitivity

Tetracycline mechanism

Patient given Clindamycin, what is likely to results = overgrowth of bacteria in his gut

TB patient with red-orange body fluid, what drug? (: Rifampin)

Amphotericin B mechanism

Hs of HSV, what does it required? Thymidine kinase

Picture: Where does Cocaine acts on NE pathway FA page311

Picture: Where alpha-2 acts on NE pathway FA page311

Patient with diarrhea, who is salivating and sweating. which drug reversesthese symptoms = Atropine

A farmer who sprayed and inhaled parathion, what treat = Atropine

Treat for asthma = beta-2 agonists

Which drug causes metabolic acidosis = Acetazolamide


71-
Amiloride acts on the collecting ducts

Which drug decreases QT interval?

Which drug reduces cholesterol = HMG-CoA reductase

Which drug lowers VLDL = gemfibrizol

What do you monitor with Bleomycin LFTs

What is the disadvantage of giving streptokinase

A man with schizophrenia has been given antipsychitcs and present with Parkinson symptoms, which drug you instead that wouldn™t have similar effects? atypicals

Sildenifil (Viagra): 2nd messenger = cGMP

Isosorbide given with Viagra (Selenfield) = cause hypotension

Antagonizing TXA2 cause what

Colcbicine mechanism

Diabetic drug something to do with glipizidine

Diabetes: Contraindication for Metformin?

cyclosporins

A pituitary tumor patient has been given bromocriptine, which hormone is inhibited = Prolactin


72-Most commonly used glucocorticoid used in chemotherapy? (prednisone

73-Warfarin poisoning

74-know everything about digoxin

75-moa of vancomycin

moa of neostigmine

76-Treatment for person with Wilson™s

77-What antipsychotic is absolutely contraindicated for a person with a heart condition

78-Patient on griseofulvin is now sterile. Problem with sperm is that...(motility)


79-Teratogenic effect of tetracycline.

Experiment where basically ask the order of receptor affinity with Dobutamine.

Drug of choice for DM complaining of neuralgia of one leg.

Mech of action of opiods in cough suppression.

Woman walks into ER with swelling of upper lip, dyspnea with cyanosis. (angioedema) What is her antihypertensive med? (Captopril)

Woman with vasospastic angina by history. Most appropriate med?


80-Phospholipase A2, AA, PG. and Leukotriene cascade, COX I and II, drugs that work here


82-Aldosterone acts via which receptors? Intracellular receptors

Angiotension II acts via which system? 1P3 2nd messenger system

83-Patient was on azithromycin develops resistance over the course of several days, what is the mechanism? Methylation of ribosomal RNA (via methyl-transferases)

Patient has been on a drug for few weeks develop interstitial nephritis and other mild symptoms; what drug was he likely taken? Methicillin

Patient with chlamydia infection was given ceftriaxone, which agent should also be given?

Mechanism of Cilastatin? Inhibits renal dihydropeptidases

Mechanism of Trimethoprim?

Mechanism of Ciprofloxacin? lnhbits DNA gyrase

Mechanism of Mebendazole?

Patient develops hemolytic anemia after taking an X drug, what is the mechanism? G6PD deficiency (Recall: INH causes bemolysis in G6PD deficient people)

A patient is taking Fluconazole for esophageal condiasis, what is the drug™s mechanism? Inhibits synthesis of fungal ergosterol

a diagram of a nucleus and asked where does Ribavarin acts?

An AIDS patient on AZT and Lamivudine, which agent is complementary for the patient regimen? Ritonavir (lndinavir was not a choice) --- must know triple therapy

A diagram of a skeletal muscle and a synaptic vesicle, asked for transmitter? Ach

Therapy for Atropine overdose? Physiostigmine

Malignant hyperthermia and Dantrolene was given, what is D mechanism? Blocks the release of Ca ions and prevents muscle contraction

ID Therapy of pheochromocytoma and ID mechanism?

Dobutamine --- order of receptor™ Beta 1 > Beta 2 and no alpha effect

Mechanism of Nortriptyline?


84-Carbmazapin for a patient with partial seizure, what is C mechanism? Decrease Na~ influx and prevents axonal conduction

Bradykinin role in AceI and cough

Patient on Furosemide develops hypakalemia, which agent can be used to correct the problem? Amiloride --- K-sparing

Hypertensive patient on a diuretic X develops Hypercalcemia, ID X and the likely mechanism of this finding?

Odonserton mechanism? Antagonist of 5-HT

Anti-lipid agents, ID most likely drug for mechanism? Cloflbrate (Gemflbrozil) --- stimulates lipoprotein lipase and chylomicrons

Most common complication with Methotrexate? Bone morrow suppression

Drug used to open and maintain a patent ductus arteriosus (PDA)? A prostaglandin

Asked about Heparin mechanism on treatment of Deep Vein Thrombosis (DVT)?

a structural diagram of arachidonic acid pathway synthesis, know it all.

Gout therapy

A Diabetic treated with Troglitazone- adverse effect? Hepatotoxicity

Leuprolide --- (GnRH agonist)- know it they always ask

Lady has been taking Iodine during pregnancy, likely fetal abnormality?


85-Before a colon surgery what antibiotic should give? Neomycin

86-A patient who was taking Codeine for pain and was complaining of constipation, why was his medicine causing the constipation?

A college girl who had overdosed on something and had pinpoint pupils- what should she be given? naloxone


87-A patient that had hypocalcaemia - what diuretic would be good for her? Furosemide


88-Man taking drug for acne and has sun burn over his sun exposed areas, what drug is he taking? Tetracycline

binds to 30S and prevents attachment or aminoacyl-tRNA.- Tetracycline


89-What should be given as prophylaxis to contacts of someone with meningitis? Rifampin


90-
What receptor Sumatriptan works on? 5-HT 1D agonist

Pt. with positive ANA. Procainamide.

Gemfibrozil what is the side effect? Myositis

Methotrexate works. -- Inhibits dihydrofolate reductase.

What do you have to check in patient on Warfarin? PT

How streptokinase (specifically) works.


91-
1. few were like what is the MOA of HAART (HIV) drugs and lots of antimicrobial drug's MOA...also rifampin and isoniazide

2 they told me pt had gray baby synd and gave cyclophosphamide....what would u give for toxicity...MESNA...

2. then lots of graphs and Drug x and Drug Y and what is the interaction or what is the name of drug X

92-.Pregnant patients with DVTfamily history of DVTwhat to giveheparin

93-What to give in CMV infection if resistant to ganciclovir.foscarnet, One question on HIV medsHAART regiminforgot what it was

94-Question on CVS, ANS drugswith graphsI made my best guessnot sure if I did right or wrong, Renal drugs, M/O of phenytoin and carbimazapine, Haloperdol, M/O buripion ( FA says still unknownI had to guess), S/E of TCA, Some CTY P450 related questions, 2-3 picture asking where the drug works

95-M/O of Diabetic drug.

96-what to give if patient is allergic to aspirinclopidogrel

97- Birth control pill associated break through bleeding in contex of CTY P450 drugs.

98-1: Volume of distribution can be increased by .. HTN ?? or obesity
2: Drug respose graph of adding drug noncompetetive antagonist
3: 3 Questions with long stem having 2 or more drugs additon and affecting different aspects of CVS, all were time consuming, n i did all wrong
4: Antiphychtic drug poisoning
5: Dextroamphetamine poisoning
6: NO source .. arginine
7: Sildenafil mech of action .. phosdiesteaarse inhibitor raising cGMP
8: Benzodiazipine acting on which receptor with diagram
9: Amphotericin B main side affect .. its nephrotixic .. damn main ne theek kr ke wapis ja ke change kr dya Tongue itna gusa aya baad main
10: Pt allergic to penicillin .. which drug to give for UTI .. 5 options, 2 penicillin, 1 cepaholosprin, clindamycin, gentamicin .. i checked gentamicin
11: Cyclophosphamide, hemorragic cystitis .. mesna
12: Pic showing cell in different phases of cell divison n spindle formation, which dug affective .. Vincristine
13: AIDS pt taking Indinavir, which other drug can replace it .. ritinavir
14: Affect of asparte inhibitor on cellular function in virus .. inhibiton of proteins for virus core
15: Women wants to have prophylaxis for malaria, which drug .. Mefloquiine
16: Pt with diarrhea, stool has entameba and giardia, drug of choice .. Metronidazole
17: Drug covering ulcer surface .. sucralfate


99-Pt presented w/hepatosplenomegaly, mild retardation, jaundice and cataract, asking what is the MOA of cataract? Galactitol

100-. A young girl had unprotected sex, and present w/cervical motion tenderness (key word) and ask for MOA of Rx? Easy on I hope

101-Questions asking what is the mechanism of resistance of Cipro

102-What is the mechanism of sulfonureas causing hyperglycemia? Straight forward

103-Pt was brought in to the ER in COMA, history of diabetes, how do you RX?

104-Parents notice there is some kind of brownish substance in the baby diaper (something like that, Can™t recall exactly) and asking for Rx MOA

105-Graph of chemical synapse. Included receptors alpha and beta

106-Dopamine, Acelyacholine, Norepi and Epinephrine. What drug would activate beta I receptor? -

107-Case of pt having psychotic symptoms, weight loss, agitation, dilated pupils, what drug abused? Amphetamine

108-What neurotransmitter does cocaine effect?

109- What does haloperidol act on? A ..dopamine receptors

110-Case of pt taking anti-depressive eats cheese and wine having headacheA.. MOA inhibitors + tyramineA.. hypertensive

111-What can give most lasting treatment for ulcerA.. antimicrobials.

112- Case of girl getting sick after drinking alcohol, what drug is she
(on?A.. metronidozole, disalfuram like effect.
* Griseofulvin.A.. interferes with microtubual function, distrupts mitosis.
Mechanism of action of Acyclovir ,
* DOC for CMV A ..ganciclovir, F oscarnet is gancyclovir fails.
* Case of pt taking nasal decongestant, what receptor is activated? A .. alpha 1.
* Side effects ofLithiumA.. hypothyroidism and nephrogenic Dl. ,
* ACE inhibitors will INCREASE renin due to reflex inhibition.
* Man with high lipids and hypertension having muscle pains and
myalgias, which drugs are causing symptoms? A.. lovastatin and loop diuretic. * Acetazlamide will causeA.. metabolic acidosis.
* Pt on warfarin has infarct, what other drug is he taking? A .. ehenobarbitol, induces P450 systep.
* Case of gout what drugs to useA.. colchicine not answer, probenecid was
~ choice. -,
* Mech. Of action of AllopurinolA.. inhibits xanthine oxidase. * Side effect ofPTUA.. agranulocytosis.
* One half life problem. J * Pt has anemia after taking anti malarial drug, what dei Does pt have?A.. G6PD dei

113- Pt taking pills develops acute hepatic reaction based on enzyme levels, AST>AL T, what did she take? A.. acetaminophen

114-Case of hypertensive having orthostatic hypotension, what drug? A .. Prazosin.A.. alpha I antogonist.
* Where does anti-cancer effect of Vincristine and Vinblastine work? DNA or RNA ?
* Case of hypertensive develops Lupus-Like symptoms, what drug is she taking?A.. Hydralazine. Other drugs include: procainamided£:. *

115- cimetidine effect on p450 and drug metabolism

116-benzodiazepines- 3-4 ques on mechanism of action/metabolism/changes in dosage for elderly 35. PCO- history given- had to

117-at least 5 kinetics questions
clavulanic acid
tetracycline as used for acne
gentamicin assoc. with nephrotoxicity
trimethoprim for PCP
flouroquinolones contraindicated in kids
anti TB drugs
fluconazole--inhibits steroid hormone synthesis
HIV therapy
mebendazole
ANS was big!
bethanechol, cocaine, antimuscarinics, clonidine, B blockers, succinylcholine, phenylyephrine (rebound nasal congestion), epi, norepi, prazosin, glaucoma drugs...just know ANS real well
several antipsychotic drugs
opioid analgesics
carbamazepine--causes aplastic anemia
nitrous oxide as anesthetic
ACE inhibitors
acid/base balance with diuretic use
hydrochorothiazide causes hyperglycemia
K+ sparing diuretics--spirinolactone can cause gynecomastia
quinidine
several questions on lipid-lowering agents
anti-cancer drugs were big too: 5 fluorouracil, doxorubicin, prednisone, raloxifene (make sure you know all their side effects)
Magnesium antacid abuse
heparin/warfarin
many NSAID questions
glucocorticoids
propylthiouracil

118-
1. MOA of rifampin (darn question showed up 2 times)
>2. MOA of erythromycin
>3. MOA of cephlosporins
>4. What do u give first to unconcious alcholic who comes in to ER?
>IV thiame first


119-Side effects of combo drugs of person being treated with
>Hodgkins

120-25. If person has megaloblastic anemia with folate and neuro
>problems and after Rx u only have neuro problems what did u give
>patient?

121- MOA of erythromycin?

122- ACE inhibitors cause renal damage.

123--SE of person taking aminoglycosides

124-. Person with H.Pylori given medication that covers ulcer, what
>is MOA of that drug?

125-SE of person taking H2 blockers

126- Give heparin to person but no change in PT or PTT what is
>deficient?

127-Best treatment of person who is Black suffering from CHF

128-. know the cyto p450 inducers and inhibitors - 6 q's

129-. moa of omeprazole
>52. moa of vancomycin
>53. moa of neostigmine

130-Very long intimidating question about drug given to young versus old. Young person™s VD and CL are twice that of the old™s. T1/2 for young person is 24 hours. What™s the T1/2 for the old person? (Without computing, I just answered 24 hours; should it have been higher or lower? I mean, the VD and CL are half of the young person™s and the formula is 0.7 Vd/Cl anyway?)
6.Glyceraldehyde 6 P ßà DHAP, reaction in favor of DHAP. What™s the G of this reaction? (I didn™t cover this in my review so I forgot about choices.)

131-Cyclosporine™s MOA

132-Mechanism of Tetracycline
Nephrotoxicity associated with Aminoglycosides and also ototoxicity.
Isoniazid and B6 which can prevent neurotoxicity
HIV triple therapy (two nucleotide reverse transcriptase inhibitors with a protease inhibitor.
Mechanism of Dantrolene
Isoprotrenol effect and reflex tachycardia associated with it
Mechanisms of action of sympathetic receptors, Dopamine receptors (eg. cAMP, IP3, etc.)
Antipsychotics and Tardive dyskinesia
Lithium and it™s effect on thyroid (may cause hypothyroidism)
Sumatriptan and it™s mechanism of action
ACE inhibitors and it™s effect on blood pressure and it™s mechanism of action
Cardiac glycosides and it™s effect on intracellular and extracellular Na and K
Busulfan associated with pulmonary fibrosis
Cisplastin associated with Nephrotoxicity
Lead poisoning and it™s features (eg. lead lines) and treatment
H2 Blockers and their mode of action and their effects on gastrin and stomach acid
Arachidonic acid products and their effects on vascular tone and platelet action
Leuprolide and it™s indications (also mechanism of action).


133--maintanence dose
-first order elim.
-effect of antagonist in general
-tetracycline mechanism (MORE than knowing 30S)
-aminoglycosides=ototoxicity
-INH mechanism
-cocaine additction mech.
-PAM
-Dantrolene mech. (Ca2+ from SR) (yes, got it right Kelly)
-Beta blockers
-Alcohol and Benzodiazipenes
-Sumatriptan mech
-Diuretics X5-6
-Quinidein efx X3
-Bleomycin side efx (cancer bear will be your friend)
-warfarin mech
-misoprostol (w/NSAIDs)
-H2 blockers
-Arachidonic acid pathway X3
-diabetic drugs (increasing insulin receptor sens)
-prednisone
VEINS DRAINING CAVERNOUS SINUS......

2-HIGHEST NERVE IN POSTERIOR TRIANGLE



5-abt 2 brainstem crosssectns,1 spinal cord...easy ones..1 mri showing sup sagittal thrombosis(curved line in sagittal section)




. pt present with knee injury w/posterior posterior sign, ask for defect & MRI ,X-RAY

8-.about 5-6 CT/MRI, one angiogram pointing to Posterior cerebral artery giving a small senerio asking what other associated symptom the patient will have.

. Bicornuate uterus what problemincomplete fusion of paramesonephric ducts, few

10-4 fracture questions what nerve involved, brachial plexus..identify radial nerve, rotator cuff.one tendon tornXray pointing asking what muscleI thought it was supraspinatus as it was on the top,

11-Buccinator pushes cheek up against your molars

12-allantois is median umbilical ligament

13-DAMN question- CN 7 via solitary nucleus (how do I forget that)


14-Double cervix caused by Double Uterus caused by

15-Testes are paraaortic

16-what artery does testes get blood supply from

17-picture of a compression fracture

18-section of the spine labeled, and they asked a lesion where would be equivalent to the SX of Horner's????

19-picture of a subarachnoid hemorrhage, question what is the etiology??
a) rupture of the bridging veins
b) coup
c) contercoup
d) sacular aneurism
e) HTA




20-gross anat.was mostly on upperlimb nerve injuries ,had 3CTSCAN total of neuroanat.and few Q from neuroanat

21-. origin of mandible/external auditory meatus

22-. Man burn his tongue when he sipped coffee, what CN?


23- 1 year old with weakness, vomiting and abd distention- revealed volvulus- what is the developmental anomaly that lead to these? (it was meckels diverticuum that lead to volvulus but the answer that they want is viteline duct.

24- child with bladder extrophy- associated with epispadia

25- Bicornuate uterus “cause

26- ANomalies of a singe umbilical artery

27- ANomalies of a singe umbilical artery

28-surgical repair of hiatal hernia - most likely to be injured by the surgeon ?
13. Most posterior part of the heart
14. 58 yr old man with abd hernia -landmark to dtermine the tpe of hernia
15. vessels to connect to bypass portosystemic shunt “

29-. Px with inattention, loss of drive. Point to the lesion. (Drawing of brain)
2. Px with injury to the knee. Point to the structure in the MRI that prevents excessive sliding of the tibia anteriorly.

3. Hypothenar atrophy and impaired adduction of fingers. What nerve is affected?
4. Px with difficulty doing push ups, doing wrist flexion, extension of fingers. What nerve is affected?

30-4-5 Cts on lower limb lesions-knee joint hit posteriorly,ankle sprains and abdomen{which i think i messed up thoroughly--i knew the answers but couldn't place them on the CT

32-
Q CT of the pelvis of a man with BPH describe à ID prostate (picture#16)

Q CT of the pelvis ID which muscle helps in valsalva à rectus abdominis (picture#16)



34-about the base of bladder “ what is the structure closely associated with it? à vas deferens

35- will mention cancer from descending colon spreads to which lymph nodesà inferior mesenteric identify on x-ray--- maxillary sinus (picture#19)

36-describe homer™s syndrome (ptosis, meiosis and anhidrosis) -where is lesion? à cervical sympathetic plexus

37- patient has a tachycardia; carotid massage stops it, mechanism? à Pressure on carotid sinus à CN IX send the signal to the brainstemà CN X (vagus) returns the signal to slow down the heart. (this is my explanation in plain English picture#20) Easy ha..

38-mentions that lateral and third ventricles enlarged, 4th is normal where is the obstruction? à Aqueduct of sylvius

39- describes subacute combined degeneration of cord, which vitamin deficient? à Vit. B12

40-
Q description of UMN corticospinal tracts- ID lesion in cordà (picture#21)- try to memorize the big ones from High Yield Neuroanatomy by Fix (very good source for neuro for step 1)


41-of a patient that has unilateral hearing loss- vertigo diagnosis? à meniere™s disease

42-Qsss about visual field defects STRIGHT from First Aid page 111- KNOW ALL of them. Most famous question on this topic will be Bitemporal Hemianopsia- whats causes it? à Optic chiasm compression by a pituitary adenoma.



43- describes internal capsule infarct, which artery? à lenticulostriate branches of middle cerebral artery.


44-in a picture (Picture#25) must diagnoseà intussuuception
Reply
#20
45-on Pt who has a severe headache. Subarachnoid hemorrhage was noticed on the CT. What are the other findings? - à Out pouching at the junction of the Ant. Communicating Artery and Ant. Cerebral Art. àBerry Aneurysm

46-shows a picture (picture#28) of an arteriogram of the cerebral arteries and Ant. Cerebral A. will be labeled and asked what functions will be lost if you severe this artery? Pt. can climb the stairs


47-shows a picture (picture#28) of an arteriogram of the cerebral arteries and Ant. Cerebral A. will be labeled and asked what functions will be lost if you severe this artery? Pt. can climb the stairs

48- will show a pic (picture#34) of supratentarial herniation occurs, what CN will be affected mostly?

49-of a young male gets a stab wound in his neck right above the clavicle and lateral to the manubrium sternum. What structures most likely damaged? àPleurae

50- describes a child who has absence of both kidneys but the ureters are present. What™s was the defect?

51- describes a pt. has Adenexal mass is going through a surgery, which ligament would you ligate to prevent the bleeding? àSuspensory Ligament (has ovarian vessels)

52- shows a Head CT (picture#38) w/ an arrow in the temporal lobe lesion (around the Meyer™s Loop) and asked the visual field defect?- à Left upper quadrantic anopsia (œpie in the sky)

53-on an athlete who fell on his knee- injury describes the tibia moving interiorly to the femur what is the injury? àAnterior cruciate ligament injury

54- on women fallen on an outstretched hand what™s signs? àMedia n. compression or damage.

55- on a women off of a ladder down and bit her bead on the table..but she is ok, 2 or 3 days after she falls unconscious... àsubdural hematoma.

56- on a guy who has a stroke and has aphasia and paresis what structure is involved? à anterior capsule

57-on papillary light reflex....what is involvedà pretectal area

58- on diaphragmatic hernia (xray shown)(picture#46)- dignosis

59- on peroneal nerve injury... àloss of dorsiflexion


60-on a young boy with stridor and fuzzy X-Ray- thumb sign (picture#47) must ID à H. influenza

61- on TB delineates the difference of the Ghon complex from the Ghon focus.


62- about a boy who uses Gowers sign to stand who has adipocytes in his calf muscle biopsy à X-linked inheritance

63- which foramen the maxillary (V3) branch of the trigeminal pass through. à foramen ovale

64- on CATs-- subdural hematomas (picture#50) vs. Epidural hematoma (picture#51). à Make sure you know that the subdural is crecent shaped and epidural is bi- convex shaped- knowing this fact always helped me pick it up.


65-on a fish bone was stuck in a kid™s œPeriformis fossa, the nerve Damaged will delay what function? à gag reflex

Q on an infant dies suddenly due to pulmonary hypoplesia

66-on a child has to go thru Tracheaectomy, which structure does it passes thru? à Skin and cricothyroid membrane most likely the answer will be.

67- on Loss of pain and temperature on one side of the face, what is the nerve lesion? àCN V


68- on a child who was extremely HOT, which structure is damaged? àAnterior hypothalamus

69-on what controls the Autononuic nervous system? àHypothalamus

70-on an alcoholic who is losing memory, what structure is damaged? àmammary bodies


71-on child with craniopharyngima, what visual field is defect à heteronymous hemianopsia

72-on a child with temporal lobe lesion, what visual field is defect? àLeft Upper qudrantic anopia (pie in the sky) “ Know the picture of Visual defects page 111- here there are alwaysss questions.

Q on a patient with paralysis of the upper arm, which structure is lesioned? àMCA

Q shows a c-section (picture#52) of the cord in a lady who losses pressure and touch on her legs, which lesion? à Gracillis nucleus area (1. Nucleus gracilis )

Q on a Man with subararachnid bleeding, which area is most likely source? à circle of Willis (berry aneurism)


73- mentions an Occlusion of the Superior Mesenteric Artery, what does it affect? àAscending colon most likely to be in the answer choices

74-brachial plexus

75-Which structure must be preserved during hysterectomy, after the uterine artery was ligated. (Ureter?) Also which nerve must be preserved during repair of a hernia


76-Kid who choked on a peanut with signs of SVC syndrome

77-Cervical rib, ID nerve by cutaneous innervation.


78-Broca™s vs. Wernicke™s aphasias and their locations


79-Carotid angiographies, Identify (ID) ACA & MCA and their related deficits

80-ID damaged structures in a) Parkinson™s disease = ID substantia Niagara b) Huntington™s disease = ID caudate nucleusc) Alzheimer™s ID hippocampusd) Kluver Bucy syndrome = ID amygdale) Wernicke Korsakoffs encephalopathy ID mamillary bodies


81-ID fiisciculus cuneate vs fasciculus gracilis and related deficits
82-structures in Circle of Willis and related deficits


83-MRI of eye muscles, ID muscles and related structures



84-Derivation of Mylohyoid muscle? 1st brachial arch


85-Pt presents with RIGHT side hoarseness, which structure is related to which structure?


86-A tumor located in Ascending colon, most likely route of spread? Superior mesentery artery

MRI of eye muscles, damage to which muscle (ID) causes the eye to deviate inward and down?

History of pt with acromegaly and giant bone formation, likely association? Bitemporal hemonopsia

Loss of sensation and movement in lower limps, ID damaged structure in carotid angiography? ACA

Description of pt with Huntington™s disease (jerky movements, etc), ID damaged structure? (picture#78)

Loss of vibration & touch in lower extremities, ID structure in Spinal cord section? Gracilis nucleus

CTs ID GI and Pelvis structures


87-A view of the intervertebral discs and pointed to the nucleus pulposis and, what was it derived from? Notochord


88-Bicornuate uterus, what caused this? Incomplete fusion of the paramesonephric ducts.

A very athletic woman who was having pain in her leg when she ran, her x-ray

89-Athlete with positive anterior drawer test, asked what was torn? AC


90-showed a faint line on one of her tibia? Stress fracture

91-Embryo of heart--sinus venosus,omphalocele vs gastrochisis-very well worded..again hard to choose between 2 options

92-GT CT FOR EPIDURAL HEMRRHAGE WTH TYPICAL TALK N DIE PRESENTATION

93-GT BRAIN SECTION HAD TO IDENTIFY SUBSATNTIA NIGRA


GT MIDBRAIN SPECIMEN WTH PARKINSONS RESTING TREMOR GVEN HAD TO IDENTIFY
SUBSTANTIA NIGRA AGAIN

GT A BRAIN SPECIMEB ,HAD TO IDENTIFY PREMOTOR AREA


HAD TO IDENTIFY MEDIAL LONGITUDINAL FASICULUS WTH SYMPTOMS GVEN OF ITS
LESION


WAS A MRI OF THIGH HAD TO IDENTIFY AN ANTEROLATERAL MUSCLE WTH LIPOMA,WAT
WAS THAT
RECTIS FEMORIS,SARTORIUS,VASTES LATERALIS



MRI OF HIP JOINT WTH OSTEOATHRITIS I GUESS BUT WAS VERY POORLY PRESENTED


94-GT A2 QS REGARDING FIELD OF VISION
1 WAS BINASAL HEMIANOPIA
1 WAS LEFT HOMONYMOUS HEMOANOPIA

95-FRACTURE OF HUMERUS WCH NERVE AFFECTED?

HAD TO IDENTIFY AXILLARY NERVE ON A BRACHIAL PLEXUS DRAWING

GT A BRANCHIAL FISTULA QS

96-about 5-6 CT/MRI, one angiogram pointing to Posterior cerebral artery giving a small senerio asking what other associated symptom the patient will have.

97-.One Brachial arch questionwhere do mandible derive fromArch1. Tip of the tongue burned when drinking hot coffee.pain transmitted byV nerve., Bicornuate uterus what problemincomplete fusion of paramesonephric ducts, few questions on developmental anomalies

98-3-4 fracture questions what nerve involved, brachial plexus..identify radial nerve, rotator cuff.one tendon tornXray pointing asking what muscleI thought it was supraspinatus as it was on the top,

99-1: Membranes of heart which contain pericarial fluid .. visceral n parietal layer of pericardium
2: Portal hypertension, stent passed to releieve from which structures .. portal vein to right hepatic vein
3: Injection in gluteal region which nerve damaged .. Schiatic N
4: Fracture of upper part of shaft of femur with Xray, which artery damaged .. Profunda femori??
4: Baby with cessation of breathing on right side also brachial plexus injury, cause of cessation .. Phrenic N damage
5: Young man having pain in temoromandibular joint, touching which muscle illicit pain .. Masseter??
6: Sucide attempt by cutting wrist, Radial N damage, which function affected .. oppostion of thumb

Neuroscience/anatomy:
1: CT scan showing arrow head to lateral rectus muscle, which funtion affected and on which side ..
2: Angiogram of cerebral artries, arrow head on some artery, n asked which fucntion affected after its blockage .. Middle cerebral artery ??causing spastic paresis of upper limbs
3: CT scan showing tumor obsructing 4th ventricle, cause .. obstructive hydrocephalus
4: Pt in vegetative state, not responding to painful stimuli, site of damage .. brainstem or internal capsule ?? right answer is brainstem, i checked internal capsule
5: Convergence of eyes intact with accomodation, but when light is shown eyes do not respond, also pt has severe myopia .. Optic N damage ( hina knows better Tongue )
6: also few questions with syptom described in stem and then shown 4-5 sections of spinal cord with labels to localise whr is the damage .. simple but time consuming
7: Section of lower part of medulla with damaged anterior part, lesion .. Hypoglosal N, tongue deviation
8: Suprachiasmatic N damage, abnormality .. melatonin secretion


100-Question described some kind of mass remnant from the tongue below cricothyroid muscle ? thyroglossal duct

101-cleft palate: ."failure of fusion of LATERAL PALENTINE, NASAL SEPTUM
* Cleft lip: Failure of fusion of MAXILLARY, MEDIAL NASAL.
101-Ct point to the body of the pancreas and asking what structure like retroperoeal? Celiac (I hope)

102-Case of g!rl who has loss of eversion and lifting whole leg to walk: what nerve? A.. common peroneal

103- Loss of sensation of lateral digitsA ..median n.
* Tumor compressing renal vein, what is result? A.. edema in lower leg, varicocele, esophageal varicies were the choices.
Blood supply to descending colon ..Inferior mesenteric artery.

104- Visual field defect of homonymous hemianopsia, where is the lesion? A .. East the optic chiasm.
* Knee jerk reflex arc.
* Where do the cerebral veins drain into : ..find on angiogram.

105- Gross photo ofbrainstem, where is : cranial nerve 5?
*

106- Case of loss of sensation of face, what nerve is affected? A ..Maxillary branch ofCN V.

107- CT of abdomen-at the level of pancreas

108-Rathke's pouch
neurohypophysis derived from ectoderm
what causes increased left atrial pressure and decreased right atrial pressure after birth
several branchial apparatus questions
question about remnant of thyroglossal duct
Meckel's Diverticulum
mesonephric vs. paramesonephric ducts
Bartholin's glands
function of sertoli cells
Potter's sequence
common peroneal nerve lesion
PCL injury
Congenital megacolon


109-.Cross section diagram of superior part of chest. What pointed structure is responsible for mydriasis, eye opening, sweating/what structure if lesioned will produce Horner™s? (I chose this bilateral structure very near and just lateral to the vertebral bodies, other choices were several nerve like sections albeit unilateral just posterior & lateral to esophagus, and the trachea) I know I™m supposed to look for the superior cervical ganglion but I haven™t encountered this diagram before.
110-.Cross section diagram of superior part of chest. What pointed structure is responsible for mydriasis, eye opening, sweating/what structure if lesioned will produce Horner™s? (I chose this bilateral structure very near and just lateral to the vertebral bodies, other choices were several nerve like sections albeit unilateral just posterior & lateral to esophagus, and the trachea) I know I™m supposed to look for the superior cervical ganglion but I haven™t encountered this diagram before.

111-.Nerve deficits, and two q about severe combined degeneration

112-Which of the abdominal aorta branches supply the foregut Answer: Celiac trunk

113-Patient presents with inflammation and necrosis of small intestine, ascending colon and proximal 2/3 of transverse colon, which blood vessel is involved:
Answer: superior mesenteric A.

114-Which branchial arch makes up Mandible and Mylohyoid, etc.
Answer: first arch

115-Picture of endochondral bone formation (similar to picture in first aid) and asks which of the following bones are formed by the shown process. Choices were clavicle, skull, femor, etc.
Answer: Femur (long bones)
Picture of a child being able to bring her shoulders together, which bone is she missing:
Answer: clavicle

116-Patient presents with deviation of uvula and shoulder droop, where is the lesion?
Answer: jugular foramen

117-Point to the muscle that depresses the eye from the adducted position
Answer: Superior Oblique muscle (same diagram as in page 108 first aid, 2001 edition).
Marcus Gunn Pupil
Lesion in which blood vessel causes left homonymous hemianopsia (angiography similar to circle of willis pictures in High Yield Neuro)
Answer: right post. Cerebral
Brown Sequard lesion
Dorsal Column pathway and lesions of Gracilis fibers and Cuneatus fibers and how they affect sensation on upper and lower limbs.
Radial nerve injury
Cervical rib and what it affects
Answer: inf. Trunk of brachial plexus

118-Patient presenting with upper and lower motor neuron problems
Answer: ALS

119- brachial plexus- I had ulnar nerve b/c interossei

120- lat rectus on mri

121- know your fractures- what muscles pull them

122- know your lesions in the CNS- whats ipsi whats
> contra

123--mesonephric ducts=male spermatochord
-inguinal fascia and saddle injury (kids starts to pee, where will urine go inside body cavity, NOT down thigh)
-diaphragmatic hernia
-GI blood supply (celiac trunk=forgeut)
-ureters and obstructions
-macula adherens
-ciliar dynein (Kartagener's syn)
-picture of lesion in Broca's area
-spianl cord lesions
-braichial plexus (of course, mine was axillary n.)

125- Q .Anatomical common injuries are super duper HY.

A football player comes in with an injury in the shaft of his humerus. What nerve is crushed (choices: radial, median, ulnar, axillary) and what prob. does he have?
Radial nerve is damaged. He is lose his triceps, brachioradialis, and have wrist drop.
This is a must know...

The aforementioned athlete breaks his supracondyle of humerus. What nerve is crushed?
Reply
« Next Oldest | Next Newest »


Forum Jump: